Rosh Cardio

Ace your homework & exams now with Quizwiz!

Which of the following is an indication for elective operative repair of an abdominal aortic aneurysm in asymptomatic patients? A) Aneurysm diameter more than 4.4 centimeters B) Aneurysm diameter more than 5.5 centimeters C) Aneurysm expansion of 0.4 centimeters in six months D) Aneurysm expansion of 0.5 centimeters in 12 months

Explanation: An abdominal aortic aneurysm (AAA) is an abnormal full thickness focal dilation of the abdominal aorta with the potential for significant morbidity and mortality. Important risk factors for abdominal aortic aneurysm include male sex, smoking, advanced age, atherosclerosis, and a family history of abdominal aortic aneurysm. Screening for an abdominal aortic aneurysm is recommended in men 65-75 years who have ever smoked cigarettes. One-time screening is also suggested in men 65-75 years of age who have a first-degree relative who required abdominal aortic aneurysm repair or died from a abdominal aortic aneurysm rupture as well as women who have a family history of either abdominal aortic aneurysm repair or death due to abdominal aortic aneurysm rupture. The benefits of screening include possibly preventing death from an abdominal aortic aneurysm rupture, but the risks include psychological harm or adverse outcomes from the management of the aneurysm. Abdominal aortic aneurysms are most commonly asymptomatic prior to rupture but come to medical attention by palpation of a pulsatile mass physical examination, incidental identification on abdominal imaging studies, or through screening programs. A widened abdominal aortic pulse may also be palpable on exam in patients with an abdominal aortic aneurysm. When symptoms do occur, the most common symptoms are abdominal, back, or flank pain. The diagnosis of an abdominal aortic aneurysm requires imaging confirmation. Abdominal ultrasound is the imaging modality used to screen asymptomatic patients based on risk factors and history or physical examination findings. Bedside abdominal ultrasound is the imaging modality used in hemodynamically unstable (hypotensive and tachycardic) patients who present to the emergency department with suspected abdominal aortic aneurysm but without a previously known abdominal aortic aneurysm. Computed tomography (CT) with IV contrast is another sensitive and specific imaging modality that provides additional anatomic detail in symptomatic patients. It is the imaging modality used when hemodynamically stable patients present to the emergency department with suspected abdominal aortic aneurysm. The management of asymptomatic patients with identified abdominal aortic aneurysm depends on the size of the aneurysm and the rate of expansion. Most patients with an asymptomatic abdominal aortic aneurysm of less than 5.5 centimeters should undergo conservative management (watchful waiting) rather than elective abdominal aortic aneurysm repair. Conservative management is preferred in these patients because the risk of rupture does not exceed the risk of repair. Conservative management involves periodic surveillance of aneurysm diameter with abdominal ultrasound and addressing modifiable cardiovascular risk factors, such as smoking, dyslipidemia, and hypertension. The frequency of surveillance with abdominal ultrasound depends on the size of the aneurysm, but it is usually recommended every six or 12 months. Patients with abdominal aortic aneurysm should be counseled on smoking cessation as smoking is the most important modifiable risk factor. Elective abdominal aortic aneurysm repair is recommended in patients with an asymptomatic abdominal aortic aneurysm more than 5.5 centimeters in diameter. Surgical repair is recommended for patients with symptomatic, nonruptured abdominal aortic aneurysm of any size. Ruptured abdominal aortic aneurysms may present as a patient with a known history of abdominal aortic aneurysm who presents in shock (hypotension, tachycardia, cool skin). However, some patients with ruptured aneurysm present with flank pain, abdominal pain, or back pain. Patients presenting with a ruptured abdominal aortic aneurysm should also undergo surgical repair. However, 50% of patients with a ruptured abdominal aortic aneurysm die prior to arriving at a hospital. Aneurysm diameter more than 4.4 centimeters (A) is not a surgical repair indication in asymptomatic patients. At this size, the risk of surgical repair is still considered greater than the risk of rupture. The perioperative mortality for elective surgical repair of an abdominal aortic aneurysm is 1-2% with elective endovascular repair and 3-5% with elective open repair. Aneurysm expansion of 0.4 centimeters in six months (C) is not a surgical indication in an asymptomatic patient. Although rapid expansion is a risk factor for rupture, expansion by more than 0.5 centimeters in six months is the recommended threshold for considering an elective repair. Aneurysm expansion of 0.5 centimeters in 12 months (D) is not a surgical indication in an asymptomatic patient. Expansion by more than 1 centimeter in 12 months is considered an indication to recommend an elective surgical repair.

A 53-year old homeless man is brought to the emergency department via ambulance after being found unresponsive. On physical exam, the patient is not breathing and his pulse is absent. Vitals reveal a T 77.3°F and absent blood pressure measurements. Findings on ECG reveal absence of electrical cardiac activity. Which of the following is the most likely diagnosis? A) Asystole B) Pulseless electrical activity C) Third-degree atrioventricular block D) Ventricular fibrillation

Explanation: Asystole, identified as the absence of electrical activity without pulse or blood pressure, is an emergent dysrhythmia. Reversible secondary causes of asystole include hypoxia, hypovolemia, hypothermia, hypo/hyperkalemia, hydrogen ion excess (acidosis), tension pneumothorax, tamponade (cardiac), toxins, and thrombosis (either pulmonary or coronary). These etiologies, sometimes called the "Hs and Ts," should be investigated and appropriately managed in the cardiac arrest patient. Jointly produced by the American Heart Association and European Resuscitation Council, the advanced cardiac life support protocol places asystole in the nonshockable rhythm category and advises immediate initiation of excellent cardiopulmonary resuscitation. As opposed to other dysrhythmias, such as ventricular tachycardia, asystole is not able to be electrically cardioverted, and therefore, treatment lies in the successful reperfusion of the heart and major organs via cardiopulmonary resuscitation and the rapid reversal of the underlying etiology. Epinephrine 1 mg IV every three to five minutes should be administered as soon as possible as well as high-quality chest compressions followed by checking for a shockable rhythm or pulse every two minutes. Following return of spontaneous circulation, postresuscitation care should begin with attention to continued management of underlying pathology and goals of minimizing brain and organ injury. Pulseless electrical activity (B) refers to a group of dysrhythmias in which organized electrical activity is found on ECG but mechanical contraction is insufficient or absent. Both pulseless electrical activity and asystole fail to perfuse the myocardium and other vital organs and require emergent activation of advanced cardiac life support with cardiopulmonary resuscitation. Third-degree atrioventricular block (C) occurs with uncoordinated contraction of the atria and ventricles. It is an emergent condition and requires pacing. Ventricular fibrillation (D) is a nonperfusing rhythm identified by the rapid, irregular, and uncoordinated contraction of the ventricles seen on ECG. It is treated via advanced cardiac life support guidelines by electrical cardioversion using unsynchronized defibrillation and cardiopulmonary resuscitation with additional epinephrine and amiodarone administered following unsuccessful rhythm conversion.

A 75-year-old woman presents to the clinic with fatigue, progressive dyspnea, and recent 15-lbs weight gain. Physical exam reveals a third heart sound, crackles in bases bilaterally, and 2+ pitting edema in ankles and pretibial region bilaterally. Which of the following is the most likely diagnosis? A) Acute decompensated heart failure B) Asthma exacerbation C) Pneumonia D) Pulmonary embolism

Explanation: Heart failure is a chronic condition characterized by impaired cardiac muscle function. In systolic heart failure, the left ventricle does not contract normally, resulting in a low ejection fraction. In diastolic heart failure, the ejection fraction is normal, however, the left ventricle does not relax normally which leads to increased pressure within the left ventricle during filling. Acute decompensated heart failure is a syndrome characterized by acute worsening of symptoms related to underlying heart failure. These symptoms include dyspnea (initially on exertion which progresses to dyspnea at rest), orthopnea, fatigue, weight gain, and edema. Physical exam shows decreased breath sounds or crackles in the lungs, elevated jugular venous pressure, and lower extremity edema most common in ankles and pretibial region. Cardiac examination may be normal or reveal a third heart sound with a laterally displaced point of maximal impulse (PMI). ECG is nonspecific, but may show low voltage or left ventricular hypertrophy. Diagnosis is suspected based on clinical presentation and is confirmed with an echocardiogram. A B-type natriuretic peptide laboratory test is typically available before echocardiogram results, and an elevated level is also supportive of diagnosis. Initial treatment of acute decompensated heart failure includes supplemental oxygen and diuretics. An asthma exacerbation (B) is acute bronchospasm triggered by a stimulus such as allergies, infection, or medications that occurs in patients with chronic asthma. Signs and symptoms include dyspnea, wheezing, cough, chest tightness, and fatigue. Although wheezing may be present in patients with acute decompensated heart failure, weight gain and edema are not associated with an asthma exacerbation. Pneumonia (C) is an infectious infiltrative process of the alveoli resulting from a bacterial, viral, or fungal etiology. Fever and productive cough are the most common presenting symptoms. Cardiac exam is normal while lung exam may reveal dullness to percussion, decreased bronchial breath sounds, rhonchi, or wheezing. Lower extremity edema and weight gain are not associated with pneumonia. Pulmonary embolism (D) is a blockage of a pulmonary artery that may be caused by thrombus, air, tumor, or fat. The most commonly associated symptoms include chest pain, shortness of breath, and cough. Shortness of breath is more commonly acute onset rather than progressive. Physical exam often reveals tachypnea, tachycardia, and leg swelling that is typically unilateral. It is not associated with lung crackles.

A two-week-old male newborn is evaluated by echocardiogram for a murmur detected on exam. The echocardiogram is notable for a hemodynamically significant patent ductus arteriosus. What is the circulatory pathology noted to make this diagnosis? A) Left-to-right shunting from the aorta to the pulmonary arteries B) Left-to-right shunting from the left atrium to the right atrium C) Left-to-right shunting from the left ventricle to the right ventricle D) Right-to-left shunting from the pulmonary arteries to the aorta

Explanation: In a fetus, the ductus arteriosus connects the pulmonary artery to the aorta and is an important element of normal fetal circulation that enables blood to bypass the lungs and go straight to the aorta. Prostaglandins and low oxygen maintain the ductus arteriosus during fetal life. In the birth transition, multiple physiologic changes occur triggered by lung expansion, alveolar fluid clearance, and abrupt increases in pulmonary perfusion and systemic pressure resulting in a change from a right-to-left shunt to a left-to-right shunt, with subsequent closure of the ductus arteriosus and foramen ovale. In some infants, this transition does not result in complete closure of the ductus arteriosus, and the resulting patent ductus arteriosus (PDA) can have hemodynamic consequences due to a persistent left-to-right shunt from the aorta to the pulmonary arteries causing volume overload, pulmonary HTN, and right-sided heart failure. The risk of PDA is increased in premature and low-birth-weight infants. Depending on the degree of the left-to-right shunt in an infant with PDA, clinical presentation in the first few days to weeks of life can vary from a murmur, to respiratory findings, to heart failure. The murmur of PDA is typically a systolic ejection murmur, though cases that have progressed further may present with a continuous "machinery murmur" best heard at the left second intercostal space. Other clinical findings may include bounding peripheral pulses, widened pulse pressure, a prominent left ventricular impulse, and respiratory findings like tachypnea, apnea, and hypercapnia. Evaluation of the size and significance of a PDA requires evaluation by echocardiography. Based on these findings, cardiologists may take a conservative/supportive approach, may opt for pharmacologic closure using cyclooxygenase inhibitors like indomethacin or ibuprofen, or may, in more severe cases, opt for surgical ligation. Left-to-right shunting from the left atrium to the right atrium (B) describes the physiology of an atrial septal defect, usually a result of a failed closure of the foramen ovale, known as a patent foramen ovale (PFO). Likewise, left-to-right shunting from the left ventricle to the right ventricle (C) describes a ventricular septal defect. Right-to-left shunting from the pulmonary arteries to the aorta (D) describes the normal fetal circulatory flow before the birth transition alters the infant's physiology leading to closure of the ductus arteriosus and reversal of the shunt to a right-to-left direction.

A 42-year-old woman with a history of depression and hypothyroid presents to her primary care physician for her annual exam. She currently takes synthroid to correct her thyroid and fluoxetine to manage her depression. She has two children, has never smoked, and drinks one to two glasses of wine per week. While updating her history and physical exam, you note she has gained 20 pounds in the last year and now has a BMI of 32. She also reports her grandmother died of ovarian cancer earlier this year. Which element of her history puts her at greatest risk for metabolic syndrome? A) Alcohol consumption B) Family history of cancer C) Use of antidepressants D) Weight

Explanation: "Metabolic syndrome" is a term applied to a group of findings that lead to increased risk for concurrent development of diabetes and cardiovascular disease. These findings typically include abdominal obesity (waist circumference in men ≥102 cm (40 in) and in women ≥88 cm (35 in)), dyslipidemia (serum triglycerides ≥150 mg/dL, serum high-density lipoprotein (HDL) cholesterol <40 mg/dL in men and <50 mg/dL in women), hypertension (blood pressure ≥130/85 mmHg), and elevated blood glucose (fasting plasma glucose (FPG) ≥100 mg/dL). Behavioral components like lack of exercise, poor cardiovascular fitness, and genetic predisposition are also influential. The most significant risk factor for development of these metabolic risks is increased body weight. Additional risk factors include high carbohydrate intake, low household income, no alcohol consumption, postmenopausal status, and smoking. Use of clozapine, an atypical antipsychotic medication, is also an independent risk factor. Individuals with one or more risk factors should be routinely assessed for risk of metabolic syndrome at least every three years. Routine assessment includes measurement of blood pressure, fasting glucose, fasting lipid profile, and waist circumference. Aggressive lifestyle interventions including physical activity and weight reduction should be recommended for patients with metabolic syndrome. Assessment tools like the Framingham Risk Score or the systematic coronary risk evaluation (SCORE) are useful for generating a risk metric that can be followed serially and can direct medical management of metabolic syndrome. Alcohol consumption (A) is associated with a decreased risk of metabolic syndrome. Family history of cancer (B) has no known risk, positive or negative, associated with development of metabolic syndrome. Use of antidepressants (C) is generally not associated with metabolic syndrome, however, use of certain atypical antipsychotic medications, especially clozapine, does significantly increase risk of metabolic syndrome. Some antidepressants are associated with suppressed satiety and weight gain, so these undesirable side effects are worth managing in patients with other risk factors for metabolic syndrome.

Which of the following is first-line intervention for treating hypercholesterolemia in a patient who has never had a cardiovascular event? A) Antiplatelet therapy B) Lifestyle modifications C) Nonstatin lipid-lowering therapy D) Statin therapy

Explanation: Elevated low density lipoprotein-cholesterol is a risk factor for the development of atherosclerotic cardiovascular disease. Primary prevention is the process used to decrease or manage risk factors in an individual who has never experienced a cardiovascular event, such as a myocardial infarction. Reduction of low density lipoprotein-cholesterol has been shown to have a positive relationship to a reduction in cardiovascular events in patients both with and without cardiovascular disease. Cardiovascular disease causes approximately one-third of deaths worldwide, which is why reducing low density lipoprotein-cholesterol is an important component of preventive medicine. Hyperlipidemia is the general medical term used to describe an increase in one or more plasma lipids, including total cholesterol, triglycerides, phospholipids, high density lipoprotein cholesterol, and low density lipoprotein-cholesterol. There are two main categories of hyperlipidemia. Primary hyperlipidemia, also referred to as familial hyperlipidemia, is caused by a genetic defect. Comorbid diseases, such as diabetes mellitus, kidney disease, hypothyroidism, and chronic alcohol use disorder, cause secondary hyperlipidemia. A number of cardiovascular risk calculators are available to guide treatment, and risk evaluations should begin at 20 years of age. Lifestyle modifications are recommended for all patients but especially those with elevated low density lipoprotein-cholesterol. These modifications include smoking cessation, weight loss for patients who are overweight or obese, regular aerobic exercise, and diets that are low in saturated fats. Patients with hyperlipidemia may also be taking antiplatelet therapy (A). This therapy is used to prevent myocardial infarction or cerebrovascular events in patients who have a history of deep vein thrombosis or atrial fibrillation. Antiplatelet therapy is also sometimes used in high-risk patients with very high low density lipoprotein-cholesterol who are intolerant of statin therapy. Nonstatin lipid-lowering therapy (C) is not recommended as primary prevention in the treatment of hyperlipidemia when medication is recommended and statin therapy is not tolerated. These patients should be counseled on lifestyle modifications. Exceptions to this recommendation are sometimes made when the low density lipoprotein-cholesterol level is higher than 190 mg/dL. When medication is indicated based on the 10-year cardiovascular risk being 7.5-10.0% or higher, statin therapy (D) is first-line pharmacotherapy. Side effects can include nausea, constipation, diarrhea, mild headache, and muscle aches. Serious side effects are rare, but when they do occur, hepatotoxicity and severe muscle pain are most commonly seen.

Which of the following is the most likely clinical presentation of Dressler's syndrome? A) Chest pressure B) Dizziness and presyncope C) Hypotension D) Pleuritic chest pain and pericardial friction rub

Explanation: Following a myocardial infarction (MI), patients are at risk for a variety of cardiac complications, including pericardial complications, mechanical complications, and conduction abnormalities. One of these complications is Dressler's syndrome, also called post-cardiac injury syndrome. Dressler's syndrome is a pericarditis with or without effusion that results from injury to the pericardium, which may occur due to myocardial infarction, pericardiotomy, or traumatic pericarditis. The initial injury is thought to release cardiac antigens that stimulate an immune response. The clinical findings of Dressler's syndrome include pleuritic chest pain, pericardial friction rub, and fever that occurs one week to three months after the pericardial injury. Leukocytosis and an elevated erythrocyte sedimentation rate (ESR) are common laboratory findings. All patients with suspected Dressler's syndrome should be evaluated with an electrocardiogram (ECG), chest radiograph, and echocardiogram. The classic electrocardiogram finding is diffuse ST-segment elevation in association with PR depression. The presence of pericardial effusion helps to support the diagnosis but is not always present. Treatment of Dressler's syndrome is similar to other types of acute pericarditis. First-line treatment consists of the combination of nonsteroidal anti-inflammatory drugs (NSAIDs), such as ibuprofen, and colchicine. Colchicine is also administered prophylactically pre- and post-cardiac surgery to prevent Dressler's syndrome. Mechanical complications can also occur following a myocardial infarction. Three life-threatening mechanical complications include rupture of the left ventricular free wall, rupture of the interventricular septum, and the development of severe mitral regurgitation. These should be suspected in patients with hypoperfusion or severe decompensated heart failure despite preserved left ventricular systolic function. The diagnosis can be confirmed using echocardiography. Early surgical intervention is the definitive treatment of severe mechanical complications following myocardial infarction, but the patient may be supported prior to surgery by insertion of an intra-aortic balloon pump. Conduction abnormalities following acute myocardial infarction include bradyarrhythmias (most common) such as atrioventricular blocks, which can be first degree, second degree or third degree. Electrocardiogram is used to diagnose conduction abnormalities. Treatment is not indicated for conduction abnormalities in asymptomatic patients. Patients with symptoms, such as dizziness, syncope, or confusion, may be treated with atropine. If atropine does not work, then temporary transvenous pacing can be used. Tachyarrhythmias following acute myocardial infarction may include ventricular tachycardia or ventricular fibrillation. The management of these patients should consist of standard care for myocardial infarction, including reperfusion therapy. Most of these patients will already be taking beta-blockers. Amiodarone can be used for control of refractory symptoms. Unstable patients (hypotension and sometimes pulseless) with ventricular tachycardia or ventricular fibrillation are managed according to advanced cardiac life support (ACLS) protocols. Chest pressure (A) is often used to describe the chest sensation during a myocardial infarction. Patients with Dressler's syndrome classically have pleuritic chest pain. Pleuritic chest pain is chest pain that is worse with deep inspiration. Dizziness and presyncope (B) can occur following a myocardial infarction when cardiac output is decreased. Conduction abnormalities, such as atrioventricular block, may cause dizziness and presyncope. Dressler's syndrome does not classically cause dizziness or presyncope. Hypotension (C) can occur following a myocardial infarction with left ventricular failure. It may also occur following myocardial infarction due to rupture of the ventricular free wall, rupture of the ventricular septum, or acute mitral regurgitation. Cardiogenic shock is not consistent with Dressler's syndrome.

A 67-year-old woman is brought to the emergency department after complaining of lightheadedness, recurrent episodes of presyncope, and shortness of breath. She has no known past medical history. Her vital signs show a BP 89/52, HR 49, RR 18, and T 98.3°F. Exam reveals an obtunded woman with diaphoresis and pale, clammy skin. Which of the following is the next step in the treatment of her bradycardia? A) Adenosine B) Atropine C) Electrical cardioversion D) Pharmacologic cardioversion

Explanation: Sinus bradycardia is a cardiac rhythm with a heart rate less than the normal expected rate produced by the sinoatrial node in the right atrium. This natural pacemaker has spontaneous automaticity, which means it does not require impulse input to generate and propagate impulses. The intrinsic firing rate of the sinoatrial node ranges from 60-100 beats per minute with sinus bradycardia being defined as a rate of less than 60 beats per minute. Sinus rate varies based on multiple factors, including age and physical conditioning, and bradycardia may not be pathologic in well-conditioned individuals. Pathophysiologic etiologies for sinus bradycardia include sinus node dysfunction, acute myocardial infarction, obstructive sleep apnea, exaggerated vagal activity, increased intracranial pressure, infection, or other conditions, such as hypothyroidism, anorexia nervosa, and hypothermia. Medications that depress sinoatrial node function include parasympathomimetic drugs (e.g., acetylcholine), sympatholytic drugs (e.g., beta-blockers, clonidine), opioids and sedatives, cimetidine, digitalis, nondihydropyridine calcium channel blockers (e.g., verapamil, diltiazem), ivabradine, amiodarone and other antiarrhythmics, lithium, chemotherapeutic agents, and organophosphate compounds. Sinus bradycardia is commonly found in healthy adults and children, particularly during sleep. Increased incidence is related to increasing age and is likely due to the increased sinus node dysfunction found in the elderly patient population. Asymptomatic in many individuals, sinus bradycardia can present with lightheadedness, presyncope or syncope, worsening angina or heart failure, cognitive slowing, fatigue, and exercise intolerance. Patients with hypotension, ischemic chest pain, dyspnea, syncope, altered mental status, and physical signs, such as diaphoresis and cool, clammy skin, are hemodynamically unstable and require immediate intervention. Findings of sinus bradycardia on ECG share many of the same features of normal sinus rhythm, including regular rhythm with a narrow QRS complex, uniform P wave present prior to all QRS complexes, and nondeviated ST segment. The principal difference between the two conditions is the heart rate. Evaluation of the P wave in leads I, II, and aVL should reveal a positive deflection with a negative deflection in lead aVR. This pattern indicates a sinus origin of bradycardia. It is differentiated from other bradyarrhythmias (e.g., atrioventricular blocks) by the 1:1 ratio between P waves and QRS complexes. Asymptomatic patients with bradycardia do not require treatment. In symptomatic patients, hemodynamic stability must first be determined. In the hemodynamically unstable patient, advanced cardiac life support guidelines, as provided by the American Heart Association and the European Resuscitation Council, recommend atropine 0.5 mg IV push be administered with repeat administration every three to five minutes for 3 mg total. If the symptoms and heart rate do not respond to atropine, dopamine 2-10 mcg/kg/minute IV infusion or epinephrine 2-10 mcg/minute IV infusion can be considered. Temporary pacing is required if medication management fails to return the rate to normal and alleviate symptoms. In hemodynamically stable patients, acute myocardial infarction should first be ruled out. Further etiologies should be considered and treated accordingly. Removal of offending medications may serve to effectively terminate sinus bradycardia. The definitive treatment for symptomatic sinus bradycardia is permanent pacemaker placement. Adenosine (A) is an antiarrhythmic agent used primarily in terminating paroxysmal supraventricular tachycardia. It slows conduction through the atrioventricular node to restore normal sinus rhythm. This drug is not a treatment for bradycardia. Electrical cardioversion (C) is used to treat ventricular fibrillation and unstable ventricular tachycardia during emergent cardiac arrest. As this patient does not have a disruption in the conductance of electrical impulses between the atria and ventricles, this procedure would not effectively treat her sinus bradycardia. Likewise, pharmacologic cardioversion (D) would not be warranted in a patient with sinus bradycardia and is used in patients with dysrhythmias such as atrial fibrillation.

Which of the following is the best way to diagnose acute rheumatic fever? A) Echocardiogram B) Joint aspiration and examination of the synovial fluid C) Jones criteria plus evidence of recent group A Streptococcal infection D) Wells criteria plus evidence of recent Staphylococcus aureus infection

Explanation: Acute rheumatic fever is a complication of group A Streptococcus (GAS) infections that occurs in about 1% of untreated cases of acute streptococcal pharyngitis. The pathogenesis involves a multisystemic autoimmune response, in which antibodies against the streptococcus bacteria cross-react with host tissue. Although acute rheumatic fever can occur in any age group, it is most common in children five to 15 years old, which is the same demographic that is most commonly infected with streptococcal pharyngitis. The clinical symptoms and signs of acute rheumatic fever typically occur two to four weeks after streptococcal pharyngitis and may include arthritis, pancarditis, Sydenham chorea, erythema marginatum, subcutaneous nodules, and fever. An acute febrile illness with polyarthritis affecting multiple joints in quick succession is the most common presentation. Carditis can cause damage to the endocardial layer, which may impact the heart valves resulting in murmurs. The most common valvular abnormality is mitral stenosis. Sydenham chorea is a neurologic symptom consisting of abrupt, nonrhythmic, and involuntary movements. Erythema marginatum is a pink or red nonpruritic rash with annular lesions usually involving the trunk. The subcutaneous nodules are firm, painless, and usually occur over bony surfaces. The diagnosis is made by major and minor manifestations listed in the Jones criteria. Major manifestations include migratory polyarthritis, carditis, Sydenham's chorea, erythema marginatum, and subcutaneous nodules. Minor manifestations include polyarthralgias, fever, elevated acute phase reactants (ESR, CRP), and prolonged PR interval on ECG. Two major manifestations or one major plus two minor are required for the diagnosis of acute rheumatic fever in a patient with evidence of a preceding GAS infection. Evidence of a preceding GAS infection can include throat culture, rapid antigen detection test, or elevated antistreptococcal antibody titers (e.g., antistreptolysin O). Elevated antistreptococcal antibody titers are the most sensitive since the streptococcal infection may no longer be able to be detected by throat culture or rapid antigen detection test at the time of acute rheumatic fever presentation. Treatment consists of nonsteroidal anti-inflammatory medications (e.g., aspirin or naproxen) for arthritis and antibiotics (penicillin or erythromycin) for the eradication of streptococcal infection. Intramuscular penicillin G benzathine is a commonly used antibiotic. Heart failure management should be done as necessary. Acute rheumatic fever can be prevented by quickly diagnosing and treating streptococcal pharyngitis. Echocardiogram (A) is performed in patients with acute rheumatic fever to evaluate the extent of damage to the heart including the valves. However, a broader clinical picture is needed to diagnose acute rheumatic fever. Joint aspiration and examination of the synovial fluid (B) is the diagnostic test of choice in other types of acute arthritis including gout, pseudogout, and septic arthritis. It is not used in the diagnosis of acute rheumatic fever. Wells criteria plus evidence of recent Staphylococcal aureus infection (D) is incorrect because the Wells criteria are used to assess the probability of a pulmonary embolism and rheumatic fever is associated with group A strep, not staph.

Which of the following is the most significant risk factor for developing an acute aortic dissection? A) Aortic aneurysm B) Connective tissue disorder C) Hypertension D) Male sex

Explanation: Aortic dissection is a pathologic separation between layers of the aortic wall due to an intimal tear. The aorta consists of three distinct layers that comprise the durable vessel wall: the intima, media, and adventitia, from innermost to outermost layers, respectively. This disruption of intima integrity allows passage of blood at high pressures to separate the intima from the other layers of the aorta wall, thereby creating a true lumen and a false lumen. Two classification systems (the DeBakey system and the Stanford system) are used to describe aortic dissections. The DeBakey system is based on the origin of the tear, with type 1 originating in the ascending aorta and involving at least the aortic arch, type 2 originating in and confined to the ascending aorta, and type 3 originating in the descending aorta with involvement distally or proximally. In contrast, the Stanford system stratifies aortic dissections regardless of origination, with all those involving the ascending aorta labeled type A and all others labeled type B. Dissections can be either spontaneous, genetically mediated, iatrogenic, or traumatic. A spontaneous aortic dissection may originate in the ascending or descending aorta, with ascending aortic dissections being twice as common and having a higher mortality rate. The separation can then propagate either proximally or distally. While aortic dissection can include the abdominal aorta, it is a much less common entity. Iatrogenic causes include surgeries on or near the aorta. The most common cause of traumatic aortic dissection are motor vehicle collisions, which account for 81% of traumatic blunt thoracic injuries. Other traumatic etiologies include motorcycle collisions, aircraft crashes, pedestrian versus automobile collisions, falls, and crush injuries. The most significant risk factor for developing a traumatic aortic dissection is rapid deceleration. The isthmus, a section of the aorta just distal to the left subclavian artery, is the most common area for a traumatic aortic dissection to occur. Other risk factors for aortic dissection include hypertension, atherosclerosis, prior cardiac surgery, known aortic aneurysm or connective tissue disorder, bicuspid aortic valve, and prior aortic surgery. Men are more likely to sustain an aortic dissection. Systemic hypertension is the most important factor predisposing patients to aortic dissection. Long-term hypertension leads to increased stress on the aortic wall, thus affecting the wall's integrity and weakening it over time. If the dissection progresses proximally from its origination, it can involve the aortic valve and cause aortic regurgitation or the pericardial sac and cause cardiac tamponade. The most common symptom of an aortic dissection is acute onset of severe chest or back pain. Occurring in 80-90% of patients, the pain is typically described as a "tearing" sensation and is located in the chest for ascending aortic dissections and in the back for dissections of the descending aorta. With progression of the dissection, patients exhibit signs and symptoms of hypoperfusion, such as shock, syncope, acute congestive heart failure, myocardial ischemia, stroke, paraplegia, extremity ischemia, and mesenteric ischemia. Physical examination of a trauma patient with suspected aortic dissection may reveal signs of chest wall trauma, such as a seatbelt sign from abrupt deceleration after an automobile collision. A new-onset cardiac or interscapular murmur (consistent with aortic regurgitation) may be heard along with the appearance of a left subclavicular hematoma. Patients may also present with a pulse deficit and considerable variation (> 20 mm Hg) between systolic blood pressure readings in the arms. Focal neurological deficits, such as stroke, Horner syndrome, hoarseness, or acute paraplegia secondary to spinal cord ischemia, arise from the progression of the dissection to involvement of branch arteries or the mass effect of the expanding aorta. Evaluation with chest radiography reveals a widened mediastinum and, occasionally, an associated pleural effusion. Due to the limited sensitivity of chest radiograph findings, further imaging studies are required to definitively diagnose aortic dissections. The first-line diagnostic test in hemodynamically stable patients with suspected aortic dissection is CT angiography spanning the entirety of the aorta. Hemodynamically unstable patients (e.g., patients who are hypotensive, have an altered mental status, or have cool, clammy skin) can be first assessed using bedside transthoracic or transesophageal echocardiography. Once the patient is assessed for hemodynamic stability using the primary survey (airway, breathing, circulation), treatment is aimed at decreasing shear stress to minimize lesion progression. This treatment is accomplished by using intravenous beta-blockers (e.g., esmolol, labetalol) or nondihydropyridine calcium channel blockers (e.g., diltiazem, verapamil). Arterial line monitoring of blood pressure is essential to facilitate rapid management of blood pressure, with the goal being the lowest systolic blood pressure tolerated without compromising mentation or adequate urine output (100-120 mm Hg). Nitroprusside, a potent vasodilator, can be added if systolic blood pressure is not adequately controlled after beta-blockade. Beta-blockers are generally contraindicated in patients with acute cocaine toxicity due to unopposed alpha-stimulation, however, selective beta-blockers, such as esmolol and labetalol, remain reasonable treatment options as these agents have mixed activity. Intravenous opioids are used to treat the pain associated with dissection. Surgical intervention depends on the location of involvement and the presence of complications. Type A lesions are treated with early surgery as medical management alone increases mortality to 50% over the course of 30 days. In comparison, patients surgically treated have a 96% one-year survival rate. Type B lesions are treated with medical management unless complications, such as resistant hypertension or progression of dissection, necessitate surgical intervention.

A 75-year-old woman with history of hypertension presents to the emergency department with dizziness and palpitations. On exam, vitals include a heart rate of 120 beats per minute, and auscultation reveals an irregularly irregular rhythm. Which of the following is the most likely diagnosis? A) Atrial fibrillation B) Atrial flutter C) Sinus tachycardia D) Ventricular fibrillation

Explanation: Atrial fibrillation (AFib) is a common arrhythmia that is characterized by irregular atrial contractions resulting in a "quivering" movement of the atria rather than true contractions. The most common disorders associated with atrial fibrillation are hypertension and coronary artery disease, while others include diabetes mellitus, obstructive sleep apnea, chronic obstructive pulmonary disease, and hyperthyroidism. Episodes of atrial fibrillation can be provoked by anxiety, alcohol consumption, and exercise. Patients may experience palpitations, dizziness, syncope, headache, fatigue, dyspnea, and chest pain. Complications include heart failure and peripheral embolism or stroke secondary to thrombus formation within the atrium. Physical exam findings include an irregularly irregular heart rate, and tachycardia is common. Diagnosis is suspected based on clinical presentation and is confirmed with electrocardiogram showing an irregularly irregular rhythm with no p-waves with a varying ventricular rate. If a patient's rate is greater than 100 beats per minute, they are said to have atrial fibrillation with rapid ventricular response. Treatment is aimed at either heart rate control or rhythm control and may utilize medications or procedures such as cardioversion, cardiac ablation, and pacemaker placement. Acute AFib in a hemodynamically unstable patient requires immediate electrical cardioversion. Acute AFib in a hemodynamically stable patient requires ventricular rate control with beta blockers or calcium channel blockers, and cardioversion to sinus rhythm after rate control is achieved. If AFib present for >48 hours or unknown period of time, patient requires three weeks of anticoagulation prior and four weeks after cardioversion. Transesophageal echocardiogram (TEE) can be used to evaluate left atrium for thrombus prior to cardioversion. Chronic AFib requires rate control with beta blockers or calcium channel blockers. Anticoagulation should also be considered in patients with chronic atrial fibrillation where the medication's benefit outweighs the risk of bleeding. The CHA2DS2 VASc score is a tool used to calculate the risk of a patient with atrial fibrillation developing a stroke or thromboembolism over one year and is used to guide the decision to start long term anticoagulation. Atrial flutter (B) is an atrial tachycardia characterized by regular atrial contractions originating outside of the sinoatrial node. Atrial flutter is most common in individuals with underlying pulmonary disease or heart conditions such as valvular disorders, recent or remote heart surgery, and pericardial disease. Symptoms are similar to atrial fibrillation including dizziness, headache, fatigue, chest pain, and shortness of breath. Physical exam may show tachycardia with or without an irregular heart rate noted. Diagnosis is made with electrocardiogram revealing classic flutter waves described as a "sawtooth" pattern. The most common ventricular rate in a patient presenting with atrial flutter is 150 beats per minute. Similar to atrial fibrillation, treatment should address heart rate versus rhythm control as well as anticoagulation. Sinus tachycardia (C) is a normal physiologic response to increased oxygen demands in the body such as with exercise, fever, or infection. Patients are typically asymptomatic but may complain of palpitations. Physical exam reveals a regular, rapid heart rate. Diagnosis is confirmed with an electrocardiogram showing a narrow-complex tachycardia with normal p-waves prior to each QRS. Treatment is generally aimed at addressing the underlying condition. Ventricular fibrillation (D) is a ventricular arrhythmia that results in rapid, unorganized ventricular contractions with no substantial cardiac output. Left untreated, this condition is fatal within minutes. Ventricular fibrillation most commonly occurs in patients with underlying heart disease. Patients have sudden collapse into an unresponsive state with no pulse or respirations. Telemetry or electrocardiogram reveals the presence of irregular fibrillatory waves of varying amplitude and morphology with no p-waves or QRS complexes. Treatment requires emergent cardiopulmonary resuscitation and defibrillation.

Persistent chest pain at rest for what length of time would be most consistent with the diagnosis of unstable angina? A) 10 minutes B) 2 minutes C) 20 minutes D) 5 minutes

Explanation: Chest pain that is not relieved by nitroglycerin and persistent for 20 minutes or greater is highly suggestive of an acute coronary syndrome (ACS), such as unstable angina (UA), and should be investigated further. ACS includes unstable angina, non-ST elevation myocardial infarction (NSTEMI), or ST elevation myocardial infarction (STEMI). Unstable angina (UA) may also present as an acute worsening in anginal symptoms that limits physical activity, angina at rest, or new-onset angina that is severe and worsening. It is characterized by myocardial ischemia without evidence of actual myocardial infarction. Causes include atherosclerotic plaque rupture, thrombosis, or vasospasm. UA is more common in men and patients often present in their early 60s with other cardiovascular comorbidities such as coronary artery disease, prior myocardial infarction, hyperlipidemia, diabetes mellitus, and smoking history. Symptoms are often triggered by an increased demand for myocardial perfusion (such as in exercise, tachyarrhythmias, fever, or cocaine or amphetamine use) without the ability to provide the adequate supply. Ischemic chest pain associated with UA occurs gradually and is provoked by activity and unrelieved with rest or nitrates. The pain is poorly localized to the chest with possible radiation to the upper extremities, lower jaw, throat, upper abdomen, and upper back. Patients typically describe the pain as "crushing," "squeezing," or "discomfort" and may clench their fist over the sternum (the "Levine sign"). Atypical presentation is more common in elderly and female patients. Diagnostics include chest X-ray, electrocardiogram (ECG), and laboratory work, including cardiac enzymes (troponins, CK-MB), electrolytes, a lipid panel, and complete blood count. ECG may show ST-segment depressions or T wave changes indicative of ischemia. Unstable angina can be differentiated from a NSTEMI by the absence of elevated cardiac enzymes. NSTEMI and STEMI will both have elevated cardiac enzymes but can be differentiated by the waveforms seen on ECG. Management of ACS in the acute setting includes serial ECGs, morphine, oxygen administration to maintain saturations > 90%, nitrates as indicated, aspirin, beta-blockers, P2Y12 inhibitors such as clopidogrel, direct thrombin inhibitors, or heparin. Patients with hemodynamic instability from cardiogenic shock, left ventricular dysfunction, or minimal response to medical therapy may benefit from cardiac catheterization. Long-term management includes lifestyle changes, such as weight reduction, dietary changes, smoking cessation, statins, aspirin, beta-blockers, ACE-inhibitors, and nitrates. Typical anginal pain lasts from 2-5 minutes (B, D), is triggered by activity, and is relieved with rest and the use of nitrates. Nitrates allow for coronary vasodilation as well as arterial and venous dilation, which decreases afterload and preload, respectively. Patients who present to the emergency department with chest pain with possible ACS should be initially assessed, and an ECG should be done within 10 minutes (A) of their arrival.

Which of the following findings is most common in patients with constrictive pericarditis? A) Atrial fibrillation B) Chest pain CElevated jugular venous pressure D) Pulsus paradoxus

Explanation: Constrictive pericarditis is the result of scarring and consequent loss of the normal elasticity of the pericardial sac. These changes impair the ability of the ventricles to distend, which reduces venous return and preload. Reduction in preload causes a reduction in cardiac output and leads to symptoms consistent with congestive heart failure. Pericardial constriction is usually chronic but may be subacute or transient. Constrictive pericarditis usually occurs due to inflammation around the heart, and possible etiologies include prior thoracic radiation exposure, prior cardiothoracic surgery, tuberculosis, or chest trauma. Constrictive pericarditis typically causes clinical findings related to fluid overload, diminished cardiac output, or both. Findings related to fluid overload include peripheral edema, anasarca, and elevated jugular venous pressure. Findings related to decreased cardiac output are worse with exertion and include dyspnea and fatigue. Physical examination findings may include elevated jugular venous pressure, pulsus paradoxus (an exaggerated drop in systolic blood pressure during inspiration), Kussmaul sign (lack of an inspiratory decline in jugular venous pressure), a pericardial knock (accentuated heart sound occurring slightly earlier than S3), edema, ascites, and cachexia. Elevated jugular venous pressure has been found in as many as 93% of patients with surgically confirmed constrictive pericarditis. The initial evaluation of patients with suspected constrictive pericarditis consists of electrocardiography (ECG), chest X-ray, and echocardiography. Electrocardiography findings are nonspecific and may include tachycardia, nonspecific ST and T wave changes, and atrial fibrillation. Pericardial calcification seen on a chest X-ray is supportive but is often absent. Doppler echocardiography is critical for the diagnosis of constrictive pericarditis. Classic findings of constrictive pericarditis on echocardiography include increased pericardial thickening and abnormal filling of the ventricles during early diastole. Additional testing is often needed if the diagnosis of constrictive pericarditis remains uncertain or if surgical intervention is planned. Patients commonly undergo cardiac catheterization and concurrent coronary angiography prior to surgical intervention to help define the coronary anatomy. Cardiac computed tomography or magnetic resonance imaging is sometimes needed to provide additional anatomic detail about the degree of pericardial thickening, calcification, and scarring. Common imaging findings include pericardial thickening, with or without calcification, and dilation of the inferior vena cava. The management of constrictive pericarditis is pericardiectomy. In some patients, however, conservative management with anti-inflammatory agents can be attempted for two to three months prior to proceeding to pericardiectomy. Other conditions that present with symptoms and exam findings similar to constrictive pericarditis include cardiac tamponade, restrictive cardiomyopathy, and cirrhosis. Atrial fibrillation (A) is caused by multiple foci in the atria firing chaotically, which causes an irregularly irregular heart rhythm. In advanced cases of constrictive pericarditis, atrial fibrillation can occur due to increased atrial pressure. However, atrial fibrillation is a less common finding than increased jugular venous pressure. Chest pain (B) is a classic finding in patients with acute pericarditis. The chest pain in acute pericarditis is described as pleuritic, which is a chest pain that is worse with deep inspiration. Some patients with constrictive pericarditis have a similar pericardial chest pain, but it is not as common as increased jugular venous pressure. Pulsus paradoxus (D) is an exaggerated (more than 10 mm Hg) drop in systolic blood pressure during inspiration. Pulsus paradoxus occurs in less than 20% of patients with constrictive pericarditis and is a more common finding in cardiac tamponade than constrictive pericarditis.

Which of the following is the most appropriate diagnostic study for definitively diagnosing giant cell arteritis? A) Erythrocyte sedimentation rate B) Magnetic resonance imaging of the head C) Rheumatoid factor D) Temporal artery biopsy

Explanation: Giant cell arteritis, also termed temporal arteritis, is a chronic, inflammatory disease that affects the medium- and large-sized arteries. Vessels ranging from the aorta to the cranial branches of the aortic arch may be involved. It is present in individuals older than 50 years of age and is the most common systemic vasculitis. The pathogenesis of this condition is not completely understood, but both adaptive and innate immune responses participate in the inflammatory pathways that infiltrate the artery wall. Vascular remodeling occurs secondary to inflammatory injury with intimal lining hyperplasia and subsequent vessel occlusion. This occlusion can then cause deleterious complications, such as vision loss. The greatest risk factor for developing giant cell arteritis is aging, with a mean age of occurrence of 76 years of age. Persons of Scandinavian ethnicity, including Americans of Scandinavian descent, have the highest incidence of giant cell arteritis. Contrarily, African American populations rarely present with this disease. Women are three times more likely than men to have giant cell arteritis. Giant cell arteritis may present with constitutional symptoms, such as fever, fatigue, and weight loss. Other complaints include headache with scalp tenderness, jaw claudication, diplopia, visual hallucinations, pitting edema, and transient vision loss (amaurosis fugax). Permanent blindness is a potential complication of giant cell arteritis and is typically painless and sudden, partial or complete, and may be unilateral or bilateral. Closely linked to giant cell arteritis, polymyalgia rheumatica is characterized by aching and morning stiffness of the shoulders, pelvic girdles, neck, and torso. Large vessel involvement predisposes patients to aortic aneurysm, aortic dissection, and aortitis. Physical examination may reveal diminished pulses or blood pressure variation between the arms. Temporal artery abnormalities may be evident and present as a prominent or enlarged artery, an absent temporal pulse, and temporal artery tenderness. Auscultation demonstrates bruits over the carotid or supraclavicular areas; the axillary, brachial, or femoral arteries; or over the abdominal aorta. Ascending aortic aneurysms secondary to giant cell arteritis are suspected with new-onset murmurs consistent with aortic regurgitation. All patients with suspected giant cell arteritis require fundoscopic exam and assessment of visual acuity. The fundus may appear normal or demonstrate cotton wool spots, which indicates local retinal ischemia. A normochromic anemia with a normal leukocyte count is typical on hematology studies. Markers of inflammation, erythrocyte sedimentation rate, and C-reactive protein levels are nearly always elevated. However, these markers are not specific to giant cell arteritis and should be used in conjunction with clinical presentation and other tests to assess the likelihood of the disease. Temporal artery biopsy is the gold standard study to definitively diagnose giant cell arteritis with characteristic histopathological findings including panarteritis and giant cells. Treatment should not be delayed to obtain temporal artery biopsy due to the catastrophic consequence of permanent vision loss. In experienced clinics, color Doppler ultrasound has been proposed as an alternative to biopsy. The cornerstone of treatment is high-dose systemic glucocorticoids. Patients without vision loss at presentation are treated with prednisone 1 mg/kg up to 60 mg daily. Those with threatened or established vision loss are treated with methylprednisolone 500-1,000 mg intravenously daily for three days. Gradual taper of steroid treatment is appropriate after two weeks, but disease relapses are most common after the daily dosage of prednisone falls below 20 mg. Patients should be monitored using erythrocyte sedimentation rate and C-reactive protein levels. Clinicians should be aware of and monitor glucocorticoid side effects, such as osteoporosis, hyperglycemia, early cataract development, and Cushingoid appearance, among others. Patients who cannot tolerate glucocorticoids may be treated with tocilizumab or methotrexate. Disease course is variable and may persist for multiple years, however, most patients eventually successfully discontinue glucocorticoid treatment. Erythrocyte sedimentation rate (A) is a commonly ordered laboratory study to aid in the diagnosis of giant cell arteritis. This study is not specific to giant cell arteritis but can be used to monitor response of the condition to glucocorticoid therapy. Magnetic resonance imaging of the head (B) is not warranted in diagnosing giant cell arteritis. This imaging is useful in the diagnosis of cerebral abnormalities, such as tumors, lesions, or cerebral atrophy or ischemia. Rheumatoid factor (C) is not associated with giant cell arteritis. Disorders with positive rheumatoid factor include rheumatoid arthritis, Sjögren syndrome, mixed connective tissue disease, mixed cryoglobulinemia, systemic lupus erythematosus, and polymyositis or dermatomyositis.

A 68-year-old man with a history of poorly controlled diabetes mellitus and hypertension presents to the clinic complaining of gradually worsening dyspnea on exertion, swelling of his ankles, and palpitations. Physical exam reveals hypertension, tachycardia, an irregularly irregular cardiac rhythm, and a laterally displaced apical impulse. There is 2+ pitting pedal edema bilaterally. Rales are auscultated in bilateral lung bases. Which of the following physical exam findings is most closely associated with heart failure? A) Bilateral pitting pedal edema B) Irregularly irregular rhythm C) Laterally displaced apical impulse D) Pulmonary rales

Explanation: Heart failure refers to disordered contractility of the cardiac muscle leading to decreased tissue perfusion. Heart failure can be divided into two broad categories: heart failure with preserved ejection fraction (diastolic heart failure) and heart failure with decreased ejection fraction (systolic heart failure). Heart failure with preserved ejection fraction occurs in the setting of hypertrophic or restrictive cardiomyopathy, where diastolic filling is decreased. Heart failure with decreased ejection fraction results from heart muscle that is dilated or weak. The effect of both types of heart failure is decreased cardiac output. Symptoms of heart failure include dyspnea, fatigue, orthopnea, peripheral edema, distended neck veins, a sustained and laterally displaced apical impulse, audible extra heart sounds, and bibasilar rales or dullness to percussion at the lung bases. Of these, a sustained and laterally displaced apical impulse has the highest correlation with heart failure. Patients at increased risk of heart failure are those with a history of myopathy, familial heart disease, rheumatic heart disease, hyperthyroidism, pheochromocytoma, dyslipidemia, diabetes mellitus, hypertension, sleep apnea, peripheral arterial disease, substance abuse, or chemotherapy or radiation to the chest. Diagnosis of heart failure is made with ECG, chest X-ray, and echocardiography. Of these, echocardiography alone can evaluate the ejection fraction. Treatment involves addressing the underlying pathologies and causative factors. In addition, heart failure patients may need diuretics, angiotensin-converting enzyme inhibitors, positive inotropic agents, and beta-blockers. Patients with congestive heart failure should be advised to decrease sodium intake, monitor daily weights, and engage in light physical activity as tolerated. Bilateral pitting pedal edema (A) is frequently found in heart failure but is also found in many other disorders, such as nephropathy, malnutrition, and venous insufficiency. The presence of pedal edema must be taken in historical context. Patients with pre-existing heart failure can be monitored for changes in their fluid status by carefully monitoring pedal edema and taking serial weights. An irregularly irregular rhythm (B) is indicative of atrial fibrillation. Rhythm abnormalities predispose patients to filling defects and thus are often found concurrently with heart failure but are not specific to the disease. Pulmonary rales (D) are common to many different disease states, including heart failure. They can also be auscultated in pneumonia, chronic obstructive pulmonary disease, and restrictive lung diseases.

Which of the following patients meet criteria for statin lipid-lowering therapy for the primary prevention of cardiovascular disease? A) 16-year-old patient with type 1 diabetes and an LDL of 140 mg/dL B) 22-year-old patient with an LDL of 180 mg/dL C) 34-year-old patient with an LDL of 146 mg/dL and a 4% risk of having a heart attack or stroke in the next 10 years D) 42-year-old patient with type 1 diabetes and an LDL of 105 mg/dL

Explanation: Hyperlipidemia, also known as dyslipidemia, is a condition defined by abnormal lipid values, including low HDL, high LDL, or high total cholesterol. There are genetic and lifestyle components that impact lipid levels. Most patients are asymptomatic, but some patients have xanthomas (hard, yellowish masses found in tendons) or xanthelasma (yellow plaques on eyelids) in severe cases. More importantly, hyperlipidemia is a risk factor for cardiovascular disease, including myocardial infarction, stroke, and peripheral arterial disease. The screening of hyperlipidemia includes total cholesterol and HDL levels. If total cholesterol is greater than 250 mg/dL or HDL cholesterol is lower than 40 mg/dL, then a full fasting lipid profile should be ordered. A full fasting lipid profile includes HDL cholesterol, LDL cholesterol, triglycerides, and total cholesterol. Initial screening for hyperlipidemia in patients at higher cardiovascular disease risk should begin between the ages of 25 and 30 in men and between 30 and 35 in women. Higher cardiovascular risk is defined as having one or more of the following risk factors: hypertension, diabetes mellitus, cigarette smoking, and a family history of premature coronary heart disease (CHD). For patients without another of these risk factors, lipid screening can begin at age 35 in men and age 45 in women. Lifestyle and pharmacologic therapy can be used to improve lipid levels. Lifestyle modifications include weight loss in overweight patients, aerobic exercise, and eating diets lower in saturated fats. Statins are the first-line pharmacologic therapy for hyperlipidemia. Primary prevention means you are trying to prevent a disease before it exists. For example, if a patient has a myocardial infarction and then takes a statin to help not have another myocardial infarction, that would be secondary prevention. Deciding which patients should take statins for primary prevention is based on lipid levels, comorbid conditions, such as diabetes mellitus, and the patient's overall risk of cardiovascular disease. The recommended groups for statin therapy for primary prevention of cardiovascular disease include any patient with LDL ≥ 190 mg/dL, patients with diabetes mellitus aged between 40-75 years with an LDL above 70 mg/dL, and nondiabetic patients aged between 40-75 years with an LDL between 70-189 mg/dL and a 10-year risk of cardiovascular disease ≥ 7.5%. It is also reasonable to offer treatment with moderate intensity statin therapy to patients with an estimated 10-year cardiovascular disease risk between 5.0 and 7.5%. The percent of risk of cardiovascular disease is calculated using a cardiovascular disease risk calculator from the American Heart Association and American College of Cardiology. The efficacy of taking statins seen in most trials is a 20-30% reduction in coronary heart disease.

An 8-year-old boy presents to the emergency department with a five-day history of low-grade fever, runny nose, and hacking cough. Yesterday, he started complaining of some chest pain that is worse with lying flat on his back. The patient appears non-toxic, and you note that while seated on the exam table, he is leaning forward in a tripod position. Physical exam reveals a pericardial friction rub on auscultation, and a chest radiograph shows a slightly enlarged heart. Which of the following is the most likely diagnosis? A) Bacterial endocarditis B) Constrictive pericarditis C) Systemic lupus erythematosus D) Viral pericarditis

Explanation: Infective pericarditis is an infection of the pericardium that can be caused by bacteria, fungi, parasites, or viruses. Viral pericarditis is the most common type in both children and adults, with coxsackievirus B being the most common cause. Infection of the pericardium can lead to pericardial effusion which can cause decreased cardiac output, hemodynamic instability, tamponade, and death. Patients present with fever and chest pain often radiating to the left shoulder that is relieved with sitting forward and worse with lying down and deep breathing. Patients may often present with a hacking cough, runny nose, diarrhea, and rash. Patients are less toxic in appearance than those with bacterial pericarditis but can become toxic if cardiac output is compromised. Physical exam may reveal a pericardial friction rub on auscultation, and palpation of the sternum can induce pain. Workup includes laboratory studies (e.g., complete blood count), pericardial fluid testing, imaging studies, such as chest radiography, computed tomography scanning, or magnetic resonance imaging, electrocardiography, pericardioscopy, and pericardial biopsy. A chest radiograph may reveal cardiomegaly depending on the size of the effusion. Echocardiography is the imaging tool of choice to rapidly diagnose the effusion and to determine its distribution and size. Classic electrocardiogram findings of acute pericarditis include diffuse ST segment elevation in multiple leads, PR depression, and PR elevation in aVR. Management of viral pericarditis is conservative including supportive care with anti-inflammatory agents (e.g., nonsteroidal anti-inflammatory drugs). Corticosteroids can be used if anti-inflammatories are unsuccessful in reducing symptoms. Pericardiocentesis, pericardial drainage, or pericardiectomy can be implemented in severe cases. Patients with bacterial endocarditis (A) will present with fever and flu-like symptoms and may have classic symptoms of petechiae, splinter hemorrhages, Osler nodes, Janeway lesions, and Roth spots. Constrictive pericarditis (B) is a chronic condition due to thickening of the pericardium that can occur after an episode of acute pericarditis. This patient's presentation is more acute, and he also has evidence of a recent viral upper respiratory infection. Patients with systemic lupus erythematosus (C) are at high risk for pericarditis, but this patient does not present with other symptoms associated with lupus such as facial rash, joint pain, and fatigue.

Which of the following medications used in the management of stable angina pectoris acts by reducing cardiac contractility and causing coronary vasodilation? A) Metoprolol tartrate B) Nitroglycerin C) Ranolazine D) Verapamil

Explanation: Ischemic chest pain associated with stable angina pectoris occurs gradually, is provoked by activity and relieved with rest or nitroglycerin, and is poorly localized to the chest with possible radiation to the upper extremities, lower jaw, throat, upper abdomen, and upper back. Patients typically describe the pain as "crushing," "squeezing," or "discomfort" and may clench their fist over the sternum (the "Levine sign"). Anginal chest pain is a consequence of coronary atherosclerotic disease leading to plaque buildup that partially obstructs the lumen and decreases blood supply to the myocardium leading to transient ischemia. Diagnosis of stable angina is often based on history and the presence of one or more cardiac risk factors, such as obesity, hyperlipidemia, hypertension, smoking history, or diabetes mellitus. Physical examination in most patients is normal. In some cases, ECG and cardiac stress testing may confirm the diagnosis. ECG may be normal in asymptomatic patients at rest, or it may reveal ST depressions associated with myocardial ischemia if the patient is symptomatic. Positive cardiac stress testing may demonstrate ECG changes associated with exertion (ST segment depression) or patients may develop chest pain, hypotension, or arrhythmia. Holter monitoring can be useful in detecting silent ischemia. Coronary angiography is the definitive test for determining coronary artery disease. Treatment includes diet and lifestyle changes, such as weight loss and smoking cessation. Medication therapy often involves a combination of beta-blockers, calcium channel blockers, aspirin, and nitrates. Calcium channel blockers work in both cardiac and smooth muscle by preventing the calcium influx into the myocytes that is needed to initiate contraction. Verapamil is a nondihydropyridine calcium channel blocker that causes coronary vasodilation and has negative inotropic and chronotropic properties, meaning it reduces both the contractility and rate of contraction. Primary care physicians should follow patients with coronary artery disease and anginal symptoms regularly to ensure adequate management and to monitor for progression of the disease. Metoprolol tartrate (A) is a beta-blocker that has negative inotropic effects but does not cause coronary or peripheral vasodilation. Metoprolol is a cardioselective beta-blocker, acting only on the beta-1 receptors. Beta-blockers are the only medication class used in stable angina that have been shown to decrease the risk of myocardial infarction and overall morbidity. Metoprolol tartrate is the immediate-release version and is dosed twice per day. Metoprolol succinate is extended-release and is dosed once per day. Nitroglycerin (B) is a potent vasodilator of coronary, arterial, and venous vasculature that improves coronary perfusion and reduces both preload and afterload. Nitrates are considered first-line treatment for acute anginal chest pain. Ranolazine (C) is a newer therapy and works by blocking late sodium channels in the heart. It is indicated for use in patients with chronic stable angina as adjunctive therapy or in patients who cannot tolerate other therapies.

What congenital heart defect is characterized by persistent communication between the pulmonary artery and the aorta? A) Atrial septal defect B) Patent ductus arteriosus C) Tetralogy of Fallot D) Ventricular septal defect

Explanation: Patent ductus arteriosus is a vascular connection between the descending thoracic aorta and the pulmonary artery that fails to close at birth. In a fetus, the ductus arteriosus vessel provides a pathway from the right ventricle to the pulmonary artery, through the ductus arteriosus, and into the aorta. This allows for the majority of blood to bypass the lungs and return to systemic circulation. A combination of prostaglandins excreted by the placenta and the low oxygen tension of the blood keep the ductus arteriosus patent. At birth, the placenta is removed, prostaglandin production drops, and oxygen tension increases. This leads to closing of the ductus arteriosus which typically occurs within 10-15 hours of birth and is complete by two to three weeks of age. The remnant of tissue remaining between the aorta and pulmonary artery is known as the ligamentum arteriosum. When this vessel fails to close at birth, it produces a left-to-right shunt and blood goes from the aorta back into the pulmonary artery. This results in a larger volume of blood within the pulmonary circulation and increased resistance within the pulmonary vasculature. If left untreated, patients go on to develop volume overload, right-sided heart failure and pulmonary hypertension. Signs and symptoms vary depending on the degree of shunting and include signs of heart failure, cyanosis, respiratory distress, and failure to thrive. Cardiac exam in patients with patent ductus arteriosus is characterized by a continuous machine-like murmur which is heard best over the left upper sternal border. Diagnosis is made with echocardiogram. Treatment varies depending on age and other risk factors at the time of diagnosis. In premature infants, a prostaglandin inhibitor such as indomethacin is often effective in closing the defect. This is not effective in term infants and children whose treatment options range from close observation to surgical ligation of the patent ductus arteriosus. Atrial septal defect (A) is congenital heart defect characterized by a hole in the septum between the right and left atrium. Patients are typically asymptomatic, however, older children and adults may experience atrial arrhythmias, fatigue, and shortness of breath. Classic murmur is a soft, midsystolic ejection murmur with an associated split second heart sound, however, this may be difficult to appreciate on exam. Diagnosis is made with an echocardiogram. If treatment is indicated, the defect can be closed by surgical means or percutaneous intervention. Tetralogy of Fallot (C) is a congenital disorder characterized by four heart defects: pulmonary artery stenosis, ventricular septal defect, concentric right ventricular hypertrophy, and a rightward shift of the aorta so that it lays over the ventricular septal defect rather than the left ventricle. Signs and symptoms include cyanosis, "tet" spells, and a harsh, crescendo-decrescendo systolic murmur. Diagnosis is made with echocardiogram. Ventricular septal defect (D) is a congenital heart defect characterized by a hole in the septum that separates the right and left ventricle. Patients may be asymptomatic in cases of small defects or present with heart failure. Cardiac exam classically reveals a harsh, holosystolic murmur that is best heard along the left sternal border. Ventricular septal defects are typically identified during infancy and diagnosis is made with echocardiogram.

A 56-year-old man is found to have atrial fibrillation with a ventricular rate of 130 beats per minute. His blood pressure is 138/86 mmHg, oxygen saturation is 99% on room air, and respiratory rate is 14 breaths per minute. Which of the following is the most appropriate next step in management? A) Anticoagulation B) Cardioversion C) Defibrillation D) Rate control

Explanation: Rate control is the priority in any hemodynamically stable patient presenting with new-onset atrial fibrillation (AF) with a ventricular rate exceeding 100 beats per minute. This can be achieved with beta-blockers or calcium channel blockers. In patients with underlying heart disease, digoxin or amiodarone can be used as well. Atrial fibrillation is the most common sustained cardiac dysrhythmia and can be divided into new onset, paroxysmal, or persistent atrial fibrillation. Atrial fibrillation is associated with conditions that lead to left atrial enlargement (such as hypertension) and subsequent changes in the atrial myocardium. A rapidly-firing focus within the left atrium usually near the pulmonary veins triggers the fibrillation. Risk factors include older age, male gender, obesity, family history, heavy alcohol use, hypertension or other underlying heart disease, or ischemia. Presentation can include chest pain, palpitations, or dyspnea and an irregularly irregular pulse on examination. Electrocardiogram reveals lack of P waves, irregular R-R intervals, and possible rapid ventricular rate. In addition to rate control, treatment of new onset atrial fibrillation involves conversion to sinus rhythm with either pharmacologic antiarrhythmics or electrical cardioversion. Patients with persistent atrial fibrillation should be managed with daily oral beta-blockers or calcium channel blockers and be considered for anticoagulation. The CHA2DS2-VASc score can aid in risk stratification when considering the benefits of chronic anticoagulation versus bleeding risk in certain patients. Anticoagulation options include aspirin, direct oral anticoagulants such as apixaban, dabigatran, rivaroxaban, or coumadin. Anticoagulation (A) is indicated in certain patients with chronic atrial fibrillation where the risk of embolization and stroke outweighs the risk of bleeding. The clinical scoring tool CHA2DS2-VASc allows for risk stratification of patients based on clinical characteristics. Components of the score include age, female gender, history of hypertension, heart failure, diabetes, prior history of stroke or thromboembolism, and a history of vascular disease. Patients with a score of two or greater benefit from oral anticoagulation with coumadin or a direct oral anticoagulant. Cardioversion (B) is used to convert atrial fibrillation to normal sinus rhythm. It is indicated in any patient in atrial fibrillation that is hemodynamically unstable with hypotension, signs of myocardial ischemia, or signs of acute heart failure with pulmonary edema. Cardioversion is also indicated in stable patients with atrial fibrillation present for less than 48 hours. When atrial fibrillation is present for more than 48 hours, patients should undergo evaluation with transesophageal echocardiogram (TEE) to rule out thrombus formation prior to cardioversion. In the presence of a thrombus in the left atrium, the patient should undergo three weeks of anticoagulation prior to cardioversion and 4 weeks after cardioversion to avoid the risk of embolization. Defibrillation (C) is the delivery of a shock that is not in synchrony with the QRS complex and is indicated only in the case of ventricular fibrillation or pulseless ventricular tachycardia. Defibrillation is not indicated in atrial fibrillation.

On echocardiogram you note the following findings: left ventricular enlargement and hypokinesis with wall thinning and diminished ejection fraction. Which of the following is the most likely diagnosis? A) Dilated cardiomyopathy B) Hypertrophic cardiomyopathy C) Restrictive cardiomyopathy D) Stress-induced cardiomyopathy

Explanation: The diagnosis of dilated cardiomyopathy (DCM) is typically made when there is evidence of left ventricular spherical dilation and impaired contraction, normal or reduced wall thickness, poor systolic wall thickening, and reduced inward endocardial systolic motion on echocardiogram. Dilated cardiomyopathy is the most common cardiomyopathy and up to 50% of cases are idiopathic. Presentation is variable and can include sudden death. Presentation can also be similar to that of heart failure due to the impaired left ventricular function: dyspnea on exertion, paroxysmal nocturnal dyspnea, exercise intolerance, and peripheral edema. Physical exam may also reveal signs of cardiomegaly and heart failure, including laterally displaced PMI, S3 gallop on cardiac auscultation, ascites, peripheral edema, and pallor or cyanosis. Many patients may have a coexisting arrhythmia (atrial fibrillation is most common). Chest X-ray will show an enlarged cardiac silhouette with possible pulmonary vascular congestion. Medical management is similar to that for heart failure and includes digoxin, beta-blockers, angiotensin converting enzyme (ACE) inhibitors, angiotensin II receptor blockers (ARBs), diuretics, and aldosterone antagonists. Definitive management is cardiac transplantation. Additionally, first-degree family members should be screened for DCM as studies have shown that nearly one-third of patients with idiopathic DCM have familial DCM. Hypertrophic cardiomyopathy (HCM) (B) is characterized by thickening of the left ventricle or interventricular septum leading to filling defects and subsequent diastolic dysfunction. Interventricular septal hypertrophy can also lead to blockage of the left ventricular outflow tract, which leads to an inability for the heart to accommodate increased workload and can lead to fatigue, dyspnea, and syncope with exertion. Restrictive cardiomyopathy (RCM) (C) is less common than DCM and HCM and also leads to impaired ventricular filling except without evidence of ventricular wall hypertrophy. Causes include infiltrative processes such as amyloidosis or sarcoidosis or storage diseases such as hemochromatosis. Stress-induced cardiomyopathy (D) is also known as apical ballooning or Takotsubo cardiomyopathy and is characterized by ST elevations and troponin elevations but without any active coronary disease or calcifications seen on cardiac catheterization.

A 58-year-old woman presents for follow-up to her primary care physician with concern for her varicose veins. In her first visit for this concern one week ago, she reported symptoms of bilateral leg pain, itching, and a feeling of heaviness. Compression stockings were recommended at that time, and she has started using them. She returns with her duplex ultrasound results, which demonstrated the presence of superficial venous reflux. Which of the following is the most appropriate treatment plan for this patient? A) Conservative treatment followed by ablation therapy B) Continue conservative treatment C) Immediate ablation therapy D) Medical management

Explanation: Venous insufficiency is caused by elevated venous pressure which leads to vein dilation, interstitial fluid accumulation, skin changes, or skin ulceration. Physiologically, the underlying issues leading to venous hypertension include inadequate muscle pump function, incompetent venous valves (reflux), and venous thrombosis or obstruction. Risk factors for these problems include arteriovenous shunt, family history, high estrogen states, increasing age, ligament laxity, lower extremity trauma, obesity, pregnancy, sedentary lifestyle, smoking, prior venous thrombosis, and some genetic conditions. Clinical presentation ranges from cosmetic concerns of superficial varicose veins to leg aches and pain, heaviness, swelling, muscle cramps, and dry, tight, itching, irritated skin. Symptoms usually worsen at the end of the day and are exacerbated by periods of sitting and inactive standing. Leg elevation may provide some relief. On examination, shiny, thin and atrophic skin, edema, dilated veins, lipodermatosclerosis (brownish-red pigmentation and induration), or ulceration (usually above the medial malleolus) may be noted. Clinical presentation is sufficient for an initial diagnosis in symptomatic patients, but duplex ultrasound provides definitive diagnosis and quantitative data for developing an appropriate management plan based on the duration of reversed flow and the depth of the veins involved. The most appropriate first-line management plan includes conservative treatment which focuses on compression therapy, exercise, and leg elevation, which compress dilated veins, decrease edema, improve oxygen transportation, and reduce inflammation. These measures serve to control mild symptoms and delay the need for intervention. If ulcers develop, management may also include wet-to-dry saline dressings and unna venous boot. In cases of confirmed and symptomatic venous insufficiency, however, conservative measures will not be ultimately therapeutic, and ablation therapy will be necessary. Due to insurance requirements, symptomatic patients are usually treated with conservative measures first, before moving to ablation. Continue conservative treatment (B) alone is an insufficient treatment plan for this symptomatic patient with a demonstrated venous reflux on duplex ultrasound. Immediate ablation therapy (C) is also not the best treatment plan, primarily due to the fact that most insurance carriers will require a documented trial period of conservative treatment prior to proceeding with ablation therapy. Medical management (D) is primarily recommended for those who have not responded to conservative treatment or have not been able to tolerate compression therapy.

Deep vein thrombosis (A) is a thrombus that affects a deep vein, such as the femoral, popliteal, tibial, fibular, and gastrocnemius veins. Thrombus in these areas can lead to pulmonary embolism. Superficial vein thrombosis, particularly in areas proximal to deep veins, can lead to deep vein thrombosis. Phlebitis (B) is the inflammation of the vein without a thrombus present. It will present as pain and erythema in the area, but duplex ultrasound will show only venous wall thickening with no thrombus or occlusion. It does not require anticoagulation treatment, but patients should return for follow-up within seven to 10 days to evaluate for resolution. Superficial thrombophlebitis (C) is limited to the accessory veins or tributary veins. Inflammation, pain, and thrombus are present. If there is low risk of venous thromboembolism, nonsteroidal anti-inflammatory medications may be sufficient to treat the condition. Which of the following conditions is associated with a harsh holosystolic murmur heard best along the left sternal border? A) Aortic stenosis B) Mitral insufficiency C) Patent ductus arteriosus D) Ventricular septal defect

Explanation: Ventricular septal defect is one of the most common congenital heart defects that is characterized by a defect or "hole" in the interventricular septum. Depending on the size of the defect, this communication between the left and right ventricles can lead to a left-to-right shunt where blood coming into the left side of the heart enters the right side of the heart through the septum before entering the pulmonary circulation. This leads to a larger volume of blood going into the pulmonary circulation and ultimately the left ventricle as well, which can lead to pulmonary hypertension and heart failure. The classic murmur associated with ventricular septal defect is a harsh, blowing holosystolic murmur with thrill heard best along the left sternal border. Clinical presentation ranges based on the size of defect from asymptomatic to signs and symptoms of congestive heart failure including tachypnea, tachycardia, poor feeding or poor weight gain, rales, and respiratory distress. Diagnosis may be suspected based on these findings and is confirmed with echocardiogram. The majority of small ventricular septal defects close spontaneously within the first two years of life. Moderate to large defects have a higher risk of heart failure secondary to left-to-right shunting. Treatment depends on the size of defect and the patient's symptoms. Medical therapy may be used to delay surgical management in hopes of spontaneous closure, however, surgical closure of the defect may also be warranted. Aortic stenosis (A) is a narrowing of the aortic valve that restricts the blood flow moving from the left ventricle to the aorta. Classic murmur associated with aortic stenosis is a high-pitched, crescendo-decrescendo murmur heard best over the right sternal border that radiates to the neck. Patients with aortic stenosis may be asymptomatic or present with shortness of breath, chest pain, dizziness, or syncope. Diagnosis is made with echocardiogram. Patients with aortic stenosis may be medically managed, however, if the stenosis is severe or associated with clinical symptoms, surgical valve replacement may be indicated. Mitral insufficiency (B) occurs when blood goes back into the atrium from the ventricle during systole due to a defect of the mitral valve leaflets. This retrograde blood flow leads to volume overload of the left atrium which can cause enlargement of the atrium leading to dysrhythmias and increased risk of clot formation. Patients may be asymptomatic or present with complaints of dyspnea, fatigue, and palpitations. Physical exam reveals an early systolic, decrescendo murmur that is heard best at the apex and radiates to the left axilla. Signs and symptoms of heart failure may also be present, including tachypnea, tachycardia, rales, edema, and respiratory distress. Diagnosis is made with echocardiogram, and the decision to surgically correct is based on the severity of presentation. Patent ductus arteriosus (C) is characterized by a failure or delayed closure of the ductus arteriosus which leads to persistent communication between the pulmonary artery and aorta. In utero, the ductus arteriosus allows for passage of blood from the right side of the heart to the systemic circulation, bypassing the majority of the pulmonary circulation which is nonfunctional in utero. This vessel normally closes shortly after birth, however, when it remains patent, blood that leaves the left ventricle into the aorta can leak back into the pulmonary artery, leading to volume overload in the pulmonary circulation. Classic murmur associated with patent ductus arteriosus is a continuous machine-like murmur. Diagnosis is with echocardiogram. Treatment may include medication therapy such as IV indomethacin in premature infants or surgical ligation in term infants or children.

A 48-year-old woman presents to the clinic complaining of shortness of breath on exertion for the past few years with progressive worsening. She takes no medications, does not smoke, and states that she was diagnosed with "some sort of heart murmur" when she was young but never needed any medication. Her vitals are within normal limits, including an oxygen saturation of 99%. On physical exam, a systolic ejection murmur that peaks late in systole is auscultated at the left sternal border, and a left parasternal lift is palpable. There is also a prominent "A" wave in the jugular venous pulse. Which of the following valvulopathies is most likely in this patient? A) Aortic regurgitation B) Mitral regurgitation C) Pulmonic stenosis D) Tricuspid stenosis

Explanation: Pulmonic stenosis is thickening of the pulmonic valve, which leads to decreased right ventricular output during systole. Most commonly, pulmonic stenosis is a congenital defect, occurring alone or in conjunction with other cardiac defects or syndromes. Congenital pulmonic stenosis is generally mild, causing little to no symptoms in childhood. As a patient with pulmonic stenosis ages, the pulmonic valve may become thicker or calcified, leading to right ventricular hypertrophy, pulmonary artery dilatation, or cardiomyopathy. Signs and symptoms of progressive disease range from dyspnea on exertion to frank heart failure with peripheral edema, orthopnea, and dyspnea at rest. Auscultation of a patient with mild to moderate pulmonic stenosis may reveal a systolic ejection murmur loudest at the left upper sternal border and an ejection click in early-to-mid systole. The murmur has a crescendo-decrescendo quality and may increase in intensity with inspiration. A split S2 may be present and can be fixed in severe disease. Auscultation of severe pulmonic stenosis may yield no ejection click, as the sound happens in early systole too close to S1 to be heard. A left parasternal lift indicates right ventricular hypertrophy in the setting of severe pulmonic stenosis. A prominent "A" wave can usually be noted in the jugular venous pulse of patients with pulmonic stenosis. Definitive diagnosis of pulmonic stenosis is made with echocardiography. Treatment of pulmonic stenosis is conservative. In childhood, if the valve is not severely diseased and the patient is not symptomatic, then observation alone is sufficient. The treatment of severe pulmonic stenosis involves surgical balloon valvotomy. Aortic regurgitation (A) produces an early diastolic murmur, a wide pulse pressure, and a "water-hammer" pulse. Patients with aortic regurgitation may complain of exertional dyspnea or exertional angina. Common causes of aortic regurgitation include rheumatic heart disease (in the developing world) and aortic root dilation. Mitral regurgitation (B) leads to a high-pitched, blowing systolic murmur heard best at the apex and radiating to the left axilla, as well as a wide splitting of S2 due to early closure of the aortic valve. Symptoms include fatigue, dyspnea, or palpitations. Common causes of mitral regurgitation include rheumatic heart disease, ruptured chordae tendineae, focal myocardial dysfunction, and congenital valvular disease. Tricuspid stenosis (D) produces a low-frequency diastolic murmur best auscultated at the lower left sternal border. Symptoms and signs of tricuspid stenosis vary with severity of disease and range from fatigue to venous congestion, hepatic congestion, abdominal pain, or ascites. Most cases of tricuspid stenosis are the result of rheumatic heart disease, although congenital factors, various forms of endocarditis, or tumors of the right heart can also cause tricuspid stenosis.

A 52-year-old man reports to the emergency department complaining of substernal chest discomfort that has been worsening over the past two hours and has not resolved with rest. His past medical history includes hypertension, diabetes mellitus type 2, and hyperlipidemia. He is given sublingual nitroglycerin and aspirin 325 mg chewed and swallowed with partial relief of his symptoms. An ECG shows no changes from baseline, and serum cardiac biomarkers are normal after three series of collections spanning 12 hours. Which of the following diagnoses is most likely? A) Acute myocardial infarction B) Gastroesophageal reflux disorder C) Stable angina D) Unstable angina

Explanation: Acute coronary syndrome is defined as a group of cardiac disorders in which the myocardium is not being adequately perfused, resulting in ischemia and eventual infarct. This group of disorders is comprised of unstable angina and acute myocardial infarction, which can be further subcategorized as ST elevation myocardial infarction and non-ST elevation myocardial infarction. Acute coronary syndrome is the presence of cardiac ischemia defined as rest angina lasting longer than 20 minutes, new-onset angina that markedly limits physical activity, or angina with increasing frequency or duration or decreased activity provocation. Unstable angina is further defined by having no cardiac biomarker elevation. Cardiac ischemia most commonly arises secondary to coronary artery narrowing and decreased perfusion on the myocardium, which is most commonly caused by plaque buildup within the coronary arteries. Vessel wall injury, including thromboembolism or plaque dislodgement, can lead to partial or total occlusion of the coronary artery, thereby causing myocardial ischemia and infarct. The most important risk factor for cardiovascular disease, atherosclerosis, is associated with dyslipidemia, hypertension, and diabetes. Other risk factors include smoking, sedentary lifestyle, obesity, increasing age, male sex, family history, and chronic kidney disease. Cardiovascular disease is the leading cause of death in most developed countries. Coronary heart disease comprises between one-third to one-half of all cardiovascular disease worldwide. The most common symptom of cardiac ischemia is chest pain, which is distinguished from nonischemic chest pain by gradual onset, provocation with activity, quality of pain (often described as discomfort rather than pain), difficult localization of pain, and chest pain at rest that generally lasts longer than 30 minutes. Ischemic pain displays characteristic radiation to the upper abdomen, shoulders, arms, wrists, fingers, neck and throat, lower jaw and teeth, and the back. Pain radiating to the upper extremities is highly suggestive of ischemic pain. Associated symptoms of ischemic pain include shortness of breath, belching, nausea, indigestion, vomiting, diaphoresis, dizziness, lightheadedness, clamminess, and fatigue. Women, diabetics, and elderly patients are more likely to present with absence of classic chest pain and related symptoms so it is important to take a detailed history and physical identifying their risk factors. The immediate evaluation of patients with suspected acute coronary syndrome should begin with assessment of responsiveness, airway, breathing, and circulation. Further evaluation may reveal cardiogenic shock, which is indicated by hypotension, tachycardia, impaired cognition, and cool, clammy, pale, ashen skin. Heart failure secondary to decreased myocardial perfusion may be found through physical exam with signs including jugular venous distention, new or worsening pulmonary crackles, hypotension, tachycardia, a new S3 gallop, and a new or worsening mitral regurgitation murmur. The diagnosis of acute myocardial damage, as seen in both ST elevation and non-ST elevation myocardial infarction, is confirmed by obtaining elevated serum cardiac enzymes troponin T and I. Patients should receive an ECG upon arrival and thereafter at 15 to 30 minute intervals. Unstable angina presents without ST segment elevation or Q waves and may reveal ST segment depression, deep T wave inversions, or no changes. The thrombolysis in myocardial infarction risk score (TIMI) and Global Registry of Acute Coronary Events risk model are used to stratify those patients with risk factors for cardiac ischemia and provide guidance on appropriate treatment. Initial intervention for patients with acute coronary syndrome includes sublingual nitroglycerin, aspirin 325 mg chewed and swallowed, a beta-blocker (e.g., metoprolol tartrate) and atorvastatin. Morphine sulfate can be used if the chest pain is not relieved by nitroglycerin or other means, Those with heart failure should not be given a beta-blocker and should instead be given a loop diuretic and noninvasive positive pressure ventilation. All unstable angina patients should be started on antiplatelet therapy (e.g., clopidogrel) in addition to aspirin and anticoagulant therapy (e.g., unfractionated heparin). Modifiable risk factors, such as obesity, diet, and lack of exercise, are key to the prevention of coronary heart disease. Acute myocardial infarction (A) is a component of acute coronary syndrome, however, this patient does not demonstrate elevated serum cardiac biomarkers. Non-ST elevation myocardial infarction shares many similarities with the presentation of unstable angina and is best distinguished based on cardiac enzyme elevation. Gastroesophageal reflux disease (B) may share characteristics of presentation with unstable angina and other cardiac ischemia. In this patient, his risk factors for coronary heart disease and his pain characteristics make unstable angina more likely. Stable angina (C) is not as likely as unstable angina given that the patient's pain has progressively worsened despite rest. Stable angina is predictable chest pain that occurs after exertion and is relieved with rest or medications.

Which of the following medication classes is used in the treatment of acute decompensated heart failure? A) Beta-blockers B) Calcium channel blockers C) Loop diuretics D) Thiazide diuretics

Explanation: Acute decompensated heart failure is marked by an acute worsening of heart failure symptoms. It can be the initial presentation of new-onset heart failure but is more commonly decompensation of chronic heart failure. There are usually triggers or precipitants contributing to acute decompensation in patients with chronic heart failure. These include nonadherence with diet or medications, systemic infection, uncontrolled hypertension, rhythm disturbances, worsening renal function, anemia, and others. Patients typically present with worsening dyspnea and the retention of fluid, which may manifest as pulmonary edema, lower extremity edema, jugular venous distention, or hepatic congestion. Other exam findings may include a third heart sound, crackles auscultated on pulmonary exam, and in severe cases, hemodynamic instability. The diagnosis of decompensated heart failure is generally straightforward in patients with findings suggestive of fluid retention and worsening dyspnea with a history of heart failure. It is a clinical diagnosis supported by ECG, chest X-ray, echocardiogram, and B-type natriuretic peptide findings. There is not one single clinical or diagnostic finding that confirms the diagnosis. An echocardiogram is recommended if this is the initial presentation of heart failure or if there is an abrupt deterioration in the patient's condition. The initial treatment of acute decompensated heart failure involves dietary sodium restriction, supplemental oxygen, assisted ventilation, if necessary, and loop diuretics, such as furosemide, for volume overload. Vasodilators (IV nitroglycerin) are also used in some patients, such as patients with severe hypertension, to reduce the afterload. Some patients who have pulmonary edema despite use of oxygen, diuretics, and nitrates may benefit from inotropic agents such as dobutamine. Beta-blockers (A) are often used in the treatment of heart failure but these are not added during acute decompensation. Patients already taking beta-blockers are usually left on them. Calcium channel blockers (B) have no direct role in the treatment of heart failure. Thiazide diuretics (D) are not the diuretics initially used for diuresis in heart failure.

Which of the following is a risk factor for acute limb ischemia? A) History of peripheral arterial disease B) Oral contraceptive use C) Prior venous thromboembolism D) Recent surgical procedure

Explanation: Acute limb ischemia is defined as a sudden decrease in limb perfusion that usually produces symptoms and often threatens limb viability. Acute limb ischemia is almost exclusively associated with arterial occlusion (common femoral artery is the most common site of occlusion). This most commonly occurs in the setting of a previously patent but atherosclerotic artery, however, it may be due to acute thrombosis of a stent or graft, dissection of an artery, or the result of an embolus from a proximal source (most commonly heart due to atrial fibrillation) lodging in a more distal vessel. Prior history of peripheral arterial disease (PAD) is a common risk factor. The clinical presentation of acute arterial occlusion depends on the length of time since the occlusion, the location of the affected vessel, and the presence of underlying vascular disease. It may present with new or worsening claudication or with sudden paralysis of the affected limb. Patients with acute or chronic limb ischemia typically have a less drastic presentation due to the development of collateral circulation that develops over time in patients with chronic ischemia. In contrast, patients with an acute extremity embolus can often pinpoint the exact time that symptoms began. A diagnosis of acute extremity ischemia can often be made clinically with history and physical examination findings. Patients with viable or marginally threatened limbs require vascular imaging (computed tomographic angiography (CTA) or catheter-based arteriography) to evaluate arterial anatomy and define site of occlusion. For those who present with acute limb ischemia, anticoagulation with a heparin drip and intravenous fluid therapy should be immediately initiated prior to making plans for intervention. Options for managing lower extremity embolism include open embolectomy, thrombolysis, and transcatheter thrombectomy. Once the acute limb issues have been attended to, the subsequent diagnostic evaluation is focused on identifying the suspected embolic source, typically using echocardiography or additional vascular imaging. Oral contraceptive use (B) is a risk factor for venous thromboembolism, including deep vein thrombosis and pulmonary embolism. It is not a risk factor for arterial embolisms. Prior venous thromboembolism (C) is the most important risk factor for future deep vein thrombosis or pulmonary embolism. It does not predict arterial embolisms. Recent surgical procedure (D) is another risk factor for venous embolisms but not arterial occlusion.

A previously healthy 68-year-old man with a 10 pack-year smoking history presents to the office for his annual exam. He had a normal colonoscopy last year and wants to make sure that he is up to date on his preventive health screenings. Which of the following health conditions should he be screened for? A) Abdominal aortic aneurysm B) Colorectal cancer C) Lung cancer D) Osteoporosis screening

Explanation: An abdominal aortic aneurysm (AAA) occurs when the aorta dilates to 50% greater than its normal size. This type of aneurysm is common and can be life-threatening. While an aneurysm can occur in other segments of the aorta, the abdominal segment between the renal arteries and iliac bifurcation is the most common location. Risk factors for the development of AAA include smoking (most common preventable risk factor), male sex, advanced age, Caucasian descent, a family history of AAA, and atherosclerosis. AAAs are generally asymptomatic until they enlarge or rupture. When symptomatic, patients with AAA present with pain in the abdomen, back, flank, or groin. The triad of abdominal pain, hypotension, and a palpable pulsatile abdominal mass indicates a ruptured AAA. Syncope is also a common occurrence when there is a ruptured AAA, and the majority of patients with a ruptured AAA die of cardiovascular complications prior to arriving at the hospital. Ultrasound is the standard imaging technique used to diagnose and evaluate AAAs. Treatment for AAA is surgical. Unruptured aneurysms are monitored until they reach 5.5 cm, at which point elective surgery is considered. Surgical repair should also be considered in rapidly expanding (> 0.5 cm in 6 months or > 1 cm per year) AAA. Ruptured aneurysms require emergent laparotomy. As a part of regular health maintenance, men with any smoking history are recommended to have a one-time ultrasound done between the ages of 65 and 75 years. Follow-up on the ultrasound is based on results. Colorectal cancer (B) screening is recommended for all adults without risk factors starting at age 50 years. Colonoscopy is one of the screening modalities used, and if normal, the recommendation is to repeat the colonoscopy in 10 years. Fecal occult blood tests and flexible sigmoidoscopy are also acceptable screening options. Smoking is the most significant risk factor for the development of lung cancer (C). Annual screening is indicated for adults aged 55 to 74 years who have a 30 pack-year smoking history. Low-dose helical computed tomography is the preferred diagnostic test. All women age 65 years and older and those under age 65 with risk factors, such as previous fracture, long-term glucocorticoid therapy, excessive alcohol use, smoking, or rheumatoid arthritis, should be screened for osteoporosis (D). Men without risk factors for osteoporosis are generally not screened.

Which of the following antibiotics should be used for empiric therapy of infective endocarditis in patients who are hemodynamically unstable? A) Cefazolin B) Dicloxacillin C) Piperacillin and tazobactam D) Vancomycin

Explanation: Infective endocarditis refers to infection of the endocardial surface of the heart. It typically involves infection of one or more heart valves or infection of an intracardiac device. Risk factors for infective endocarditis include cardiac risk factors, such as a history of prior infective endocarditis, the presence of a prosthetic valve or cardiac device, and a history of valvular disease, and noncardiac risk factors, such as intravenous drug use, the presence of an indwelling intravenous catheter, immunosuppression, or a recent dental or surgical procedure. The clinical manifestations of infective endocarditis are highly variable. Infective endocarditis may present as an acute, rapidly progressive infection or as a subacute or chronic infection with low-grade fever and nonspecific symptoms. Fever, chills, anorexia, weight loss, malaise, headache, myalgias, night sweats, abdominal pain, cough, and pleuritic chest pain are common clinical findings. Cardiac murmurs are observed in 85% of patients with infective endocarditis. The mitral valve is typically the most commonly involved heart valve. However, in intravenous drug abusers the tricuspid valve is the most common valve affected. Additional examination findings may include petechiae and splinter hemorrhages. Splinter hemorrhages are non blanching, linear, reddish-brown lesions under the nail bed. Janeway lesions, Osler nodes, and Roth spots are relatively uncommon clinical manifestations that are highly suggestive of infective endocarditis. Janeway lesions are non tender erythematous macules on the palms and soles. Osler nodes are tender subcutaneous violaceous nodules mostly on the pads of the fingers and toes. Roth spots are edematous, hemorrhagic lesions of the retina with pale centers. Janeway lesions are more likely to occur in acute presentations, whereas Osler nodes and Roth spots are more common in delayed presentations. Osler nodes and Roth spots are thought to represent the sequelae of vascular occlusion by microthrombi leading to localized immune-mediated reactions. Janeway lesions represent microabscesses with neutrophil infiltration of capillaries. The diagnosis should be suspected in patients with fever and cardiac or noncardiac risk factors. The diagnosis is established based on clinical manifestations, blood cultures, and echocardiography in accordance with the modified Duke criteria. The modified Duke criteria consider major criteria and minor criteria to estimate the odds of a patient having infective endocarditis. The major criteria include positive blood cultures for organisms likely to cause infective endocarditis and evidence of endocardial involvement on echocardiogram. The minor criteria include the presence of risk factors (e.g., intravenous drug use or prosthetic heart valve), fever, vascular phenomena (e.g., Janeway lesions), immunologic phenomena (e.g., Osler nodes, Roth spots), and microbiologic evidence that does not meet the major criteria. For the diagnosis of infective endocarditis to be made, one of the following must be present: two major criteria, one major and three minor criteria, or five minor criteria. Echocardiography should be performed in all patients with suspected infective endocarditis as soon as possible after the diagnosis if suspected. Echocardiography is considered positive for infective endocarditis if there is vegetation, abscess, or new dehiscence of a prosthetic valve. Transthoracic echocardiography (TTE) is the initial diagnostic test. However, the sensitivity is only 75%. Therefore, transesophageal echocardiography (TEE) is obtained in cases with negative transthoracic echocardiography but high clinical suspicion. The treatment of infective endocarditis requires bactericidal antibiotic therapy. The antibiotic regimen should be targeted to the organism isolated from blood cultures. However, empiric antibiotic therapy may be necessary prior to blood culture results in patients who are clinically unstable, but empiric antibiotics are often not used in clinically stable patients. Patients are considered unstable if there is rapid clinical deterioration or the patient is tachycardic or hypotensive. When empiric therapy is going to be started prior to blood culture results, at least three blood cultures should be obtained from separate venipuncture sites prior to initiating antibiotics. The empiric regimen should cover staphylococci (methicillin-sensitive and methicillin-resistant), streptococci, and Enterococci. Vancomycin is an appropriate choice when empiric therapy is needed. Once culture results are obtained, the antibiotic treatment of infective endocarditis should be targeted to the cultured organism. Methicillin-sensitive Staphylococcus aureus can be treated with nafcillin or oxacillin. Viridans streptococci are usually penicillin-sensitive and are treated with penicillin G or ceftriaxone. The duration of therapy in native valve endocarditis typically ranges from four to six weeks depending on the virulence of the pathogen, the site of valvular infection (left or right-sided heart valves), and the presence of complications, such as heart failure. More virulent pathogens require closer to six weeks of antibiotic therapy. Patients with complications of infective endocarditis, such as heart failure, persistent bacteremia, or multidrug-resistant organisms, require early surgical consultation. Cefazolin (A) is a first-generation cephalosporin that provides coverage against methicillin-sensitive Staphylococcus aureus and most streptococcus species. Cefazolin does not provide coverage against methicillin-resistant Staphylococcus aureus, therefore, vancomycin is a better choice for empiric antibiotic coverage. Dicloxacillin (B) is a penicillin antibiotic that is active against beta-lactamase-producing organisms. Dicloxacillin can provide coverage against methicillin-sensitive Staphylococcus aureus but does not cover methicillin-resistant Staphylococcus aureus. Vancomycin is a better choice for empiric therapy because it covers both methicillin-sensitive and methicillin-resistant Staphylococcus aureus. Piperacillin and tazobactam (C) is a penicillin and beta-lactamase inhibitor antibiotic. The combination of piperacillin and tazobactam is often used to provide coverage against Pseudomonas aeruginosa. Pseudomonas aeruginosa is not a common cause of infective endocarditis, therefore, the combination of piperacillin and tazobactam is not the best antibiotic choice for empiric coverage.

A 62-year-old man with a history of hyperlipidemia and hypertension presents with three days of cough, fever, and difficulty breathing. A chest X-ray confirms right-lower-lobe pneumonia, but incidentally, widening of the mediastinal silhouette and enlargement of the aortic knob are also noted, concerning for thoracic aortic aneurysm. Which of the following processes is the most likely cause of this aneurysm? A) Autoimmune B) Degeneration C) Infection D) Trauma

Explanation: An aortic aneurysm is defined by the presence of at least a 50% increase in the diameter of a full-thickness segment of a blood vessel. The most common cause of thoracic aortic aneurysm (TAA) is degenerative, and patients with or at-risk for atherosclerosis (including patients with dyslipidemia, hypertension, male sex, and smoking) are more likely to develop TAA. Mechanical factors and protein degradation are believed to result in medial degeneration and breakdown of extracellular matrix proteins with subsequent loss of integrity and strength to the vascular wall. The most common location for an aortic aneurysm is abdominal and one-third are thoracic. Most patients with TAA are asymptomatic. A TAA may be found incidentally when a patient requires a chest X-ray and findings consistent with TAA are evident. Classically, these radiologic findings include displacement of the trachea from midline, enlarged aortic knob, and a widened mediastinum. Patients with symptomatic TAA may present with acute onset of chest pain, upper back pain, or nerve dysfunction related to compression of structures surrounding the aneurysm. These symptomatic TAAs are usually very large, at high-risk for rupture, and are associated with high mortality rates. Classification of TAAs is based on location, extent of aortic involvement, and morphology. Classification and etiology influence management. Management includes lifestyle changes, serial imaging, medical management, and surgical repair.

A 62-year-old man presents to the clinic complaining of intermittent left-sided chest pain that does not radiate. He describes the pain as deep pressure. The pain is elicited by physical labor or stress, lasts two to five minutes, and is alleviated by rest. His vital signs are within normal limits except for a body mass index of 30 kg/m². He does not smoke, takes no daily medications, and has a sedentary lifestyle. Physical exam is negative, and laboratory results are pending. Which of the following diagnostic studies represents the next best step in the evaluation of this patient's chest pain? A) 12-lead electrocardiogram B) Chest X-ray C) Exercise stress test D) Holter monitor

Explanation: Angina pectoris is transient chest pain or discomfort due to myocardial ischemia from atherosclerotic disease of the coronary vessels or vasospasm. The pain or discomfort of stable angina pectoris usually lasts between two and five minutes, has a predictable course, and can be reproduced by repeating the inciting exertional activity. Unstable angina is a worsening of angina symptomatology or symptoms occurring at rest. The chest pain of angina can be described as crushing, aching, or squeezing pressure. Atypical angina presentations can involve jaw or shoulder pain, dyspepsia, or confusion. Women are more likely to present with atypical angina than men. Risk factors for stable angina pectoris are those for myocardial ischemia: sedentary lifestyle, diabetes mellitus, hypertension, dyslipidemia, male sex, obesity, smoking, and family history of heart disease. An asymptomatic patient presenting with a recent history of symptoms compatible with stable angina pectoris should be evaluated for the presence of these risk factors, including a fasting glucose level and lipid panel. In addition, an ECG should be obtained. ECG is often normal in patients with stable angina pectoris. The ECG is used to rule out the presence of previous or active infarct. After ECG, laboratory studies, and addressing risk factors and comorbidities, the patient should be referred for cardiac stress testing. Positive stress testing calls for cardiac catheterization to determine the level and degree of coronary artery blockage. Treatment of stable angina pectoris depends on the degree of atherosclerotic disease and usually begins with medical therapies, such as aspirin and nitroglycerine, treatment of comorbid disorders, and lifestyle modifications. Extensive coronary artery blockage on cardiac catheterization necessitates invasive procedures, such as coronary angioplasty or coronary artery bypass surgery. A chest X-ray (B) would not be necessary for an asymptomatic patient with a normal physical exam. If history or physical exam raised suspicion of heart failure, a chest X-ray would be an appropriate next step. This patient has a negative physical exam and no dyspnea on exertion or orthopnea. Exercise stress test (C) is going to be necessary for this patient but should be performed after an ECG is obtained. If an old or active myocardial infarct is noted on ECG, or an abnormal rhythm is detected, exercise stress testing may not be appropriate, and a nuclear stress test, cardiac catheterization, or echocardiogram would be the next step. Holter monitor (D) is recommended in patients who complain of intermittent symptoms of dysrhythmia or presyncope and whose symptoms are not easily reproducible.

Which of the following antihypertensive medications is associated with the adverse effect peripheral edema? A) Amlodipine B) Chlorthalidone C) Lisinopril D) Valsartan

Explanation: Angiotensin-converting enzyme inhibitors (ACEI), angiotensin II receptor blockers (ARBs), thiazide diuretics, long-acting calcium channel blockers, and beta-blockers are each commonly used to treat primary hypertension. Although beta-blockers are no longer considered a first-line agent for primary hypertension, they are still commonly used as an antihypertensive because patients with hypertension frequently have other indications for a beta-blocker, such as stable angina or atrial fibrillation with a rapid ventricular rate. These classes of antihypertensives each have important adverse effects and contraindications. The adverse effects of angiotensin-converting enzyme inhibitors (ACEIs) include hypotension, acute kidney injury, hyperkalemia, cough, angioedema, and anaphylactoid reactions. Angiotensin II receptor blockers (ARBs) are associated with lower rates of cough and angioedema compared to angiotensin-converting enzyme inhibitors but have a higher risk of hypotension. Combined therapy with an angiotensin-converting enzyme inhibitor and angiotensin II receptor blockers should not be used because it increases the risk of adverse effects. Angiotensin-converting enzyme inhibitor and angiotensin II receptor blockers are contraindicated in pregnancy. Calcium channel blockers can be divided into dihydropyridine calcium channel blockers, such as amlodipine, and nondihydropyridine calcium channel blockers, such as verapamil or diltiazem. The mechanism of action of dihydropyridine calcium channel blockers is vasodilation, and the common adverse effects are headache, lightheadedness, flushing, and peripheral edema. The major potential adverse effects of nondihydropyridine calcium channel blockers are constipation and bradycardia. Nondihydropyridine calcium channel blockers are contraindicated in patients with sick sinus syndrome, second- or third-degree heart block, and systolic heart failure (heart failure with reduced ejection fraction). Thiazide diuretics are associated with hypokalemia, hyponatremia, hypomagnesemia, hyperuricemia, hyperlipidemia, hyperglycemia, decline in sexual function in men, and sleep disturbances. These potential adverse effects should be monitored by checking these parameters one to two weeks after initiating therapy or changing a dose and then monitoring again after six to 12 months. Beta-blockers are contraindicated in patients with decompensated heart failure, symptomatic bradycardia, and are relatively contraindicated in patients with reactive airway disease, such as asthma or chronic obstructive pulmonary disease. Additional adverse effects of beta-blockers can include depression, fatigue, and sexual dysfunction. Beta-blockers should be tapered at discontinuation since acute withdrawal can cause a hyperadrenergic state. Chlorthalidone (B) is a thiazide diuretic. The most common adverse effects of thiazide diuretics are hypokalemia, hyponatremia, hypomagnesemia, hyperglycemia, hyperuricemia, hyperlipidemia, decline in sexual function, and sleep disturbances. Lisinopril (C) is an angiotensin-converting enzyme inhibitor. The most common adverse effects of angiotensin-converting enzyme inhibitors are hypotension, acute kidney injury, hyperkalemia, cough, angioedema, and anaphylactoid reactions. Valsartan (D) is an angiotensin II receptor blocker. Angiotensin II receptor blockers have similar adverse effects to angiotensin-converting enzyme inhibitors.

16-year-old patient with type 1 diabetes and an LDL of 140 mg/dL (A) does not meet criteria for statin therapy. Patients with diabetes mellitus should be treated with statin therapy between the ages of 40-75 years if LDL-C is above 70 mg/dL. This patient should be encouraged to make the lifestyle modifications listed above. 22-year-old patient with an LDL of 180 mg/dL (B) also does not meet criteria for statin therapy. While the LDL-C is high, it does not meet the 190 mg/dL cutoff, and due to the patient's young age, the overall 10-year risk of cardiovascular disease is relatively low. This patient should also be educated on lifestyle modifications. 34-year-old patient with an LDL of 146 mg/dL and a 4% risk of having a heart attack or stroke in the next 10 years (C) also does not meet the above criteria for statin therapy. What is the imaging modality of choice to rule out aortic dissection in hemodynamically-stable patients presenting to the emergency department? A) Chest X-ray B) CT angiography C) Magnetic resonance imaging D) Transthoracic echocardiography

Explanation: Aortic dissection is a relatively rare but dangerous disorder that occurs when the aortic intima is torn, and a false lumen is created. Patients often present with severe, ripping, stabbing or tearing chest pain or back pain and often have hemodynamic compromise. Other clinical signs include pulse or BP asymmetry between limbs, diaphoresis, new heart murmur (aortic regurgitation), focal neurologic deficit, hypotension, and syncope. Dissections of the ascending aorta are classified as type A (proximal), while all other dissections of the descending aorta are classified as type B (distal). The most important risk factor for aortic dissection is long-standing systemic hypertension, however, bicuspid aortic valve, genetic disorders, connective tissue diseases (Marfan, Ehlers-Danlos syndrome), inflammatory vasculitis, pregnancy, trauma and cocaine use also increase patients' risk. Initial workup generally includes an ECG, chest X-ray (widened mediastinum), and labs. For definitive diagnosis, however, a CT angiogram of the chest through pelvis is the imaging modality of choice in stable patients. This is partly due to the widespread availability, speed, and excellent sensitivity and specificity. Once the diagnosis is made, prompt surgical consult is necessary. Efforts must also be made to correct any hypertension promptly to avoid further propagation. Commonly used medications include IV esmolol, IV labetalol, and IV sodium nitroprusside. Type A ascending dissections are surgical emergencies, while Type B descending dissections without end-organ compromise may be managed medically with IV labetalol, IV esmolol, or IV propranolol and pain control with morphine or dilaudid. Chest X-ray (A) is a reasonable screening test to look for mediastinal widening, however, a modest percentage of aortic dissections will have a normal chest radiograph. A chest X-ray should be included in the workup to rule out other causes of chest pain but is not adequate as a single imaging study to diagnose aortic dissection. Magnetic resonance imaging (C) has excellent sensitivity and specificity in diagnosing aortic dissection, however, due to the issue of time, availability, and higher cost, it is not the imaging modality of choice. It may be considered for serial monitoring to avoid excessive radiation or in the non-emergent setting. Transthoracic echocardiography (TTE) (D) is effective in identifying ascending aortic dissection, however, it does not favorably visualize the aortic arch or descending aorta. Transesophageal echocardiography (TEE) is favored over transthoracic, however, due to availability issues and the need for endotracheal intubation, it is not favored over CT angiogram. However, TEE is the test of choice in unstable patient because it can be performed at the bedside.

What is the imaging modality of choice to rule out aortic dissection in hemodynamically-stable patients presenting to the emergency department? A) Chest X-ray B) CT angiography C) Magnetic resonance imaging D) Transthoracic echocardiography

Explanation: Aortic dissection is a relatively rare but dangerous disorder that occurs when the aortic intima is torn, and a false lumen is created. Patients often present with severe, ripping, stabbing or tearing chest pain or back pain and often have hemodynamic compromise. Other clinical signs include pulse or BP asymmetry between limbs, diaphoresis, new heart murmur (aortic regurgitation), focal neurologic deficit, hypotension, and syncope. Dissections of the ascending aorta are classified as type A (proximal), while all other dissections of the descending aorta are classified as type B (distal). The most important risk factor for aortic dissection is long-standing systemic hypertension, however, bicuspid aortic valve, genetic disorders, connective tissue diseases (Marfan, Ehlers-Danlos syndrome), inflammatory vasculitis, pregnancy, trauma and cocaine use also increase patients' risk. Initial workup generally includes an ECG, chest X-ray (widened mediastinum), and labs. For definitive diagnosis, however, a CT angiogram of the chest through pelvis is the imaging modality of choice in stable patients. This is partly due to the widespread availability, speed, and excellent sensitivity and specificity. Once the diagnosis is made, prompt surgical consult is necessary. Efforts must also be made to correct any hypertension promptly to avoid further propagation. Commonly used medications include IV esmolol, IV labetalol, and IV sodium nitroprusside. Type A ascending dissections are surgical emergencies, while Type B descending dissections without end-organ compromise may be managed medically with IV labetalol, IV esmolol, or IV propranolol and pain control with morphine or dilaudid. Chest X-ray (A) is a reasonable screening test to look for mediastinal widening, however, a modest percentage of aortic dissections will have a normal chest radiograph. A chest X-ray should be included in the workup to rule out other causes of chest pain but is not adequate as a single imaging study to diagnose aortic dissection. Magnetic resonance imaging (C) has excellent sensitivity and specificity in diagnosing aortic dissection, however, due to the issue of time, availability, and higher cost, it is not the imaging modality of choice. It may be considered for serial monitoring to avoid excessive radiation or in the non-emergent setting. Transthoracic echocardiography (TTE) (D) is effective in identifying ascending aortic dissection, however, it does not favorably visualize the aortic arch or descending aorta. Transesophageal echocardiography (TEE) is favored over transthoracic, however, due to availability issues and the need for endotracheal intubation, it is not favored over CT angiogram. However, TEE is the test of choice in unstable patient because it can be performed at the bedside.

A 51-year-old man with history of hypertension presents to the office with worsening fatigue and shortness of breath with exercise over the past three to four months. He occasionally has accompanied chest pain that is improved with rest. Physical exam reveals a forceful apical pulse as well as a high-pitched blowing diastolic murmur best heard at the left sternal border. Electrocardiogram shows some evidence of left ventricular hypertrophy but no ST segment elevation or T wave inversions. Which of the following is the most likely diagnosis? A) Acute coronary syndrome B) Aortic regurgitation C) Aortic stenosis D) Mitral regurgitation

Explanation: Aortic regurgitation can be rheumatic and nonrheumatic in nature, with the latter being more common since the introduction of modern antibiotics. Most patients with the condition usually have history of infective endocarditis, rheumatic disease, bicuspid aortic valve, ankylosing spondylitis, Marfan syndrome, Ehlers-Danlos syndrome, syphilis, or long-standing hypertension. Incompetent closure of the aortic valve causes forward and backward flow of the blood across the valve, eventually causing left ventricular enlargement and hypertrophy as a compensatory mechanism to preserve the ejection fraction. Patients will be asymptomatic until about middle age when the left ventricle reaches its maximum diameter and left-sided heart failure develops as a result. The most common symptoms of left-sided heart failure are exertional dyspnea and fatigue, as well as paroxysmal nocturnal dyspnea and pulmonary edema. Chest pain may also occasionally be present. Those with chronic aortic regurgitation may present with classic physical findings such as a water-hammer pulse or Corrigan pulse (a bounding and forceful pulse caused by a high stroke volume being ejected into a low pressure peripheral vascular system), Quincke pulses (nail bed capillary pulsations), and Musset sign (head bob with each pulse). Other physical findings include a prominent apical pulse and a high-pitched blowing decrescendo diastolic murmur best heard at the left sternal border, widened pulse pressure (markedly increased systolic blood pressure, with decreased diastolic blood pressure). Echocardiography is the tool of choice to diagnose and determine the severity of the condition. An electrocardiogram will show moderate to severe left ventricular hypertrophy. Treatment with positive inotropes (e.g. dopamine) and vasodilators (e.g., nitroprusside, CCBs) may be required to improve systolic function and decrease afterload. Surgical intervention is warranted when there is evidence of left ventricular dysfunction, in symptomatic patients, and in patients with acute aortic regurgitation (e.g., post-MI). Acute coronary syndrome (A) is not the best answer since the patient has an obvious murmur and does not have any electrocardiogram findings such as ST segment elevation or T wave inversions. Aortic stenosis (C) and mitral regurgitation (D) are characterized by a crescendo-decrescendo murmur and a holosystolic murmur upon auscultation, respectively. They are not characterized by diastolic murmurs.

A 70-year-old woman presents to the clinic following an episode of syncope. She denies chest pain, palpitations, edema, or shortness of breath. Cardiac exam reveals a crescendo-decrescendo, systolic murmur which is heard best over the right upper sternal border and radiates to the carotid artery. Which of the following is the most likely diagnosis? A) Aortic stenosis B) Mitral regurgitation C) Mitral stenosis D) Tricuspid regurgitation

Explanation: Aortic stenosis is a valvular disorder characterized by narrowing of the aortic valve opening. This increased resistance causes the left ventricle to produce more pressure to pump blood out of the heart. This results in complications such as left ventricular hypertrophy, heart failure, pulmonary hypertension, arrhythmias, endocarditis, and sudden cardiac death. There are three primary causes of aortic stenosis: calcification of a congenitally abnormal unicuspid or bicuspid valve, calcification of trileaflet aortic valve in elderly, and rheumatic fever. Patients are often asymptomatic for years, however, as the narrowing worsens, symptoms include fatigue, decreased exercise tolerance, syncope, chest pain, and heart failure. Physical exam findings include a harsh crescendo-decrescendo, systolic murmur that is heard best over the right upper sternal border and radiates to the carotid artery. Other findings include a delayed carotid pulse with decreased amplitude. Aortic stenosis is often suspected based on exam and is confirmed with echocardiogram. Treatment may consist of medical management in mild-moderate cases, however, surgical replacement of the valve is often required in severe cases. Mitral regurgitation (B) occurs when the mitral valve fails to close properly, resulting in reverse blood flow from the left ventricle to the left atrium during systole. Complications include heart failure and atrial fibrillation. The severity of regurgitation and presence of other complications from the disease will determine if the patient is symptomatic. Symptoms, if present, most commonly include shortness of breath and fatigue. Physical exam reveals a holosystolic murmur heard best at the apex. Diagnosis is made with an echocardiogram. Mitral stenosis (C) a narrowing of the mitral valve, is most commonly caused by rheumatic heart disease. Mitral stenosis leads to increased resistance that the left atrium must overcome to deliver blood to the left ventricle and can lead to left atrium dilation and increased pressure within the pulmonary circulation. Complications include heart failure, pulmonary hypertension, and atrial arrhythmias. The most common symptom is dyspnea on exertion which may progress to dyspnea at rest. Cardiac auscultation reveals an opening snap and low-pitched diastolic murmur. Diagnosis may be suspected based on exam and is confirmed on echocardiogram. Tricuspid regurgitation (D) is an insufficiency of the tricuspid valve that leads to reverse blood flow from the right ventricle to the right atrium. Tricuspid regurgitation typically occurs secondary to right atrial enlargement. Often, this condition is asymptomatic and found incidentally. If severe, symptoms include a pulsatile sensation in the neck, peripheral edema, and ascites. Physical exam may show distended jugular veins, hepatomegaly, and edema. Cardiac auscultation reveals a holosystolic murmur along the right or left mid-sternal border. Diagnosis is made with an echocardiogram.

A 22-year-old man who is otherwise healthy presents to the office with complaints of easy fatigability and shortness of breath with exercise that has worsened over the past six months. He is active and has been jogging for exercise for years but recently is unable to run for more than one or two minutes without stopping. He has a family history of heart disease and hyperlipidemia, with his maternal grandfather passing away from a myocardial infarction at age 55. The patient has a blood pressure of 122/76, a pulse of 76, and pulse oximetry of 98%. Upon physical examination, you note a harsh crescendo-decrescendo systolic murmur with normal S1 and S2 heart sounds. Which of the following is the most likely diagnosis? A) Acute coronary syndrome B) Aortic stenosis C) Mitral regurgitation D) Mitral stenosis

Explanation: Aortic stenosis is the obstruction of blood flow across the aortic valve, which results in left ventricular hypertrophy. It has a high mortality rate in patients who are symptomatic and can result in heart failure and sudden death if left untreated. It has multiple etiologies, including congenital (bicuspid aortic valve) and acquired causes, with calcification of tricuspid aortic valve (due to degenerative changes) being more common than rheumatic. Patients with congenitally abnormal bicuspid aortic valve are at risk for aortic root dilation involving the ascending aorta and aortic dissection. Risk factors for calcification of tricuspid aortic valve include age, hypertension, hypercholesterolemia, diabetes mellitus, and smoking. Patients are asymptomatic for about 10-20 years and commonly complain of exertional dyspnea or fatigue when initially symptomatic. The classic triad of symptoms is chest pain (worse with exertion, better with rest), heart failure symptoms (including dyspnea, orthopnea, dyspnea on exertion, and shortness of breath), and exertional syncope. Other symptoms include dizziness, lightheadedness, easy fatigability, and progressive exercise intolerance. Patients will also have a classic harsh crescendo-decrescendo systolic murmur that is heard best in the second right intercostal space radiating to carotid arteries. The S1 heart sound is usually normal. S2 heart sound is soft and may be single since the aortic component may be delayed and merge into P2. The S2 may also become paradoxically split when the stenosis is severe and associated with left ventricular dysfunction. There may also be a prominent S4 sound due to forceful atrial contraction into the hypertrophied left ventricle. The carotid arterial pulse is typically delayed and has a plateaued peak, decreased amplitude, and a gradual downslope (pulsus parvus et tardus). Echocardiography is the imaging tool for choice to diagnose and determine the severity of this condition. Findings may show an echo-dense aortic valve with no cusp motion, a decrease in the maximal aortic cusp separation, and the presence of unexplained left ventricular hypertrophy. A normal aortic valve area is around 3.0 to 4.0 cm2 and mild to moderate stenosis is diagnosed when the area is between 1-1.5cm2 with severe aortic stenosis diagnosed when the area is <1.0 cm2. If clinical symptoms are not consistent with echocardiogram results (e.g., the results show only mild to moderate stenosis in a patient with severe clinical findings), cardiac catheterization is recommended for further evaluation since it is an accurate tool in measuring stenosis. Electrocardiography, chest radiography, serum electrolyte levels, cardiac biomarkers, and a complete blood count should be obtained to rule out other causes of angina. The definitive treatment for aortic stenosis is aortic valve replacement, which is indicated for symptomatic patients with severe aortic stenosis. If patient is asymptomatic, no treatment is indicated. Acute coronary syndrome (A) is unlikely due to the patient's age and lack of risk factors for heart disease (such as hyperlipidemia, diabetes, hypertension). Exercise intolerance is common with mitral regurgitation (C), but physical exam findings would reveal a holosystolic murmur at the apex which radiates the back or clavicular area with a diminished S1 heart sound and a wide splitting of the S2 heart sound. Mitral stenosis (D) is characterized by an accentuated first heart sound with an opening snap, and diastolic rumble murmur.

Which of the following tick-borne diseases is most likely to be associated with an atrioventricular heart block? A) Babesiosis B) Ehrlichiosis C) Lyme disease D) Rocky Mountain spotted fever

Explanation: Lyme carditis should be considered in the differential diagnoses for any patient presenting with a new-onset atrioventricular (AV) block. Lyme disease is a tick-borne illness caused by the spirochete Borrelia burgdorferi (transmitted by deer tick Ixodidae scapularis) and is most prevalent in the northeast and upper midwestern areas of the United States. The peak incidence occurs during summer months and is associated with outdoor activities such as hiking, camping in wooded areas. There are three stages of infection: early localized, disseminated, and late, persistent infection. Classic signs and symptoms of early localized Lyme disease include erythema migrans (large, painless, well-demarcated target-shaped lesions), fevers, myalgias, arthralgias, malaise, and lymphadenopathy. Disseminated Lyme disease occurs within weeks to months of initial infection and is characterized by cardiac manifestations, such as myocarditis and AV conduction delays, and neurologic manifestations, including Bell's palsy, meningitis, and encephalitis. It is estimated in the US that approximately 1% of patients with lyme disease develop Lyme carditis. Persistent infection includes arthritis and central nervous system manifestations and occurs months to years after initial infection in untreated patients. Diagnosis is made with enzyme-linked immunosorbent assay (ELISA) IgG and IgM tests, followed by a confirmatory Western Blot analysis for equivocal results. Since Lyme carditis is a late manifestation of Lyme disease, most patients will have positive results on serology testing. First-line treatment for early localized Lyme disease is oral doxycycline 100 mg twice daily for 10-21 days. Amoxicillin or cefuroxime should be given to pregnant patients and children less than eight years of age. Disseminated and persistent Lyme disease are treated with intravenous ceftriaxone, which should be continued until the resolution of any AV nodal block that is present. Additionally, patients with AV nodal blocks should be observed in the hospital on telemetry and considered for temporary pacemaker placement depending on the severity of the nodal block. Babesiosis (A) is caused by Babesia species, most frequently B. microti, and infects red blood cells. It is most frequently reported in the northeastern and upper midwestern US. While there are no classic cardiac manifestations of Babesiosis, patients can present similarly to early Lyme disease with symptoms such as fevers, malaise, fatigue, myalgias, and arthralgias, as well as gastrointestinal symptoms, mild hepatomegaly, or splenomegaly. Patients often have a hemolytic anemia, thrombocytopenia, elevated serum creatinine, and elevated blood urea nitrogen (BUN) levels. Diagnosis is made by identification of Babesia on peripheral blood smear or on polymerase chain reaction (PCR). Treatment in adults is atovaquone and azithromycin orally for seven to ten days. Ehrlichiosis (B) is caused by Ehrlichia species and is typically reported in the southeastern and south-central US. It can be fatal if left untreated and presents with flu-like symptoms, gastrointestinal symptoms, and altered mental status. Laboratory values show thrombocytopenia, leukopenia, and anemia. Diagnosis is made via PCR analysis. Rocky Mountain spotted fever (D) is caused by the spirochete Rickettsia rickettsii and is endemic in the southeastern, midwestern, and western US. It is classically associated with a maculopapular rash appearing two to five days after exposure, which begins peripherally and spreads centrally. Diagnosis is made by PCR analysis of skin biopsy or serum IgG titer. Treatment for both Ehrlichiosis and Rocky Mountain spotted fever is doxycycline 100 mg twice daily for both children in adults for at least one week.

Which of the following would be seen in the early stages of atherosclerosis? A) Calcifications B) Connective tissue C) Lipid-laden macrophages D) Low-density lipoproteins

Explanation: Atherosclerosis is a pathologic process that causes the lumen of the arteries to narrow and possibly become occluded either acutely or due to chronic disease process. The earliest stage of atherosclerosis is the formation of fatty streaks on the walls of the intima. These fatty streaks are composed of lipid-laden macrophages known as foam cells. Very low-density lipoproteins and low-density lipoproteins accumulate in these areas leading to the expansion of the fatty streak. This process can begin early in life. As the fatty streak lesions expand, they incorporate connective tissues forming plaques within the vessels. In advanced stages of atherosclerosis, a necrotic, lipid-rich core can be found within the plaques and calcifications. Many factors have been found to play a role in the progression of atherosclerosis. Dyslipidemia, inflammation, endothelial dysfunction, plaque rupture, and smoking are major risk factors. Genetics have been shown to play a lesser role. In its early stages, atherosclerosis is asymptomatic. Symptoms present when there is a significant reduction in the diameter of the vessel. This reduction can be chronic or acute. In chronic atherosclerosis, the plaque formation leads to a gradual narrowing of the vessel. An acute reduction in the flow through the vessel can occur if there is a rupture or erosion of the plaques that lead to thrombosis. The reduction in flow caused by the narrowing of vessels or thrombosis can produce ischemia, which may present as peripheral artery disease, angina, myocardial infarction, stroke, or sudden death. Treatment of the condition involves addressing risk factors to prevent the progression of the sclerosis of the vessels. Patients should be counselled on the importance of controlling their cholesterol levels, hypertension, and diabetes. Smoking cessation and diet modifications are lifestyle modifications that can help to slow the disease progression. Calcifications (A) are one of the later stage findings seen in atherosclerosis. Advanced lesions with a lipid-rich core may eventually evolve to lesions with calcifications. Fibrous plaques evolve from the fatty streak. These plaques form with the accumulation of connective tissue (B) in the area. Low-density lipoproteins (D) and very low-density lipoproteins accumulate early in the formation of the fatty streak. They are bound and trapped in the area by a proteoglycan known as biglycan.

Which of the following tachyarrhythmias has similar risk factors and presents similarly to atrial fibrillation? A) Atrial flutter B) Atrioventricular nodal reentry tachycardia C) Multifocal atrial tachycardia D) Ventricular tachycardia

Explanation: Atrial flutter shares many of the same risk factors and occurs in similar clinical scenarios as atrial fibrillation. Atrial flutter is a supraventricular tachycardia characterized by rapid atrial contractions at times exceeding > 300 beats per minute. It is much less common than atrial fibrillation and can be divided into typical and atypical forms. Typical atrial flutter involves a reentry circuit through the cavotricuspid isthmus in the right atrium. Atypical atrial flutter does not involve this pathway and is more likely contributable to the presence of scar tissue or intrinsic heart disease. Risk factors include obesity, obstructive sleep apnea, hyperthyroidism, and previous cardiac surgery. Atrial flutter can sometimes occur as a result of antiarrhythmic medications used in atrial fibrillation. Symptoms include palpitations, shortness of breath, fatigue, and dizziness. Physical exam may reveal a regular or irregular pulse with possible signs of congestive heart failure if cardiac output is diminished. Electrocardiogram reveals an atrial rate of around 300 beats per minute, a ventricular rate of approximately 150 beats per minute, and may show the classic "sawtooth" pattern in the inferior leads. Management includes rate control through the use of beta-blockers or calcium channel blockers and radiofrequency catheter ablation to restore normal sinus rhythm. Anticoagulation can be considered in patients with sustained atrial flutter to prevent embolization. Atrioventricular nodal reentry tachycardia (AVNRT) (B) is the most common paroxysmal supraventricular tachycardia. It typically has a regular ventricular response and usually occurs more often in women and at younger ages, with an average age of onset of 32 years. The presence of two electrical pathways near the AV node causes AVNRT. Patients can take abortive antiarrhythmic medications ("pill-in-the-pocket") as needed, and catheter ablation is considered for refractory cases. Multifocal atrial tachycardia (MAT) (C) is a narrow QRS complex tachycardia characterized by at least three different P wave morphologies. It is most often associated with significant lung disease, such as chronic obstructive pulmonary disease (COPD), though it can be seen with underlying cardiac disease, electrolyte abnormalities, and medications as well. Like atrial fibrillation, patients typically have an irregular pulse. Management includes calcium channel blockers or ablation. Ventricular tachycardia (D) is a wide QRS complex tachycardia that, if sustained, usually leads to hemodynamic instability and collapse if untreated. It typically occurs in patients with structural heart disease, and management involves initiation of antiarrhythmics, cardioversion, or defibrillation in unresponsive patients and follows the advanced cardiac life support (ACLS) protocols.

Which of the following congenital heart disorders is most commonly associated with a defect of the ostium secundum? A) Atrial septal defect B) Coarctation of the aorta C) Patent ductus arteriosus D) Ventricular septal defect

Explanation: Atrial septal defect is a common congenital heart defect characterized by malformation of the septum between the right and left atrium. During fetal development, the atrial septum develops as the septum primum grows downward from the superior atrium and fuses to close the ostium primum orifice. Meanwhile, a second hole is formed in the septum primum called the ostium secundum. The septum secundum arises on the right atrial side and grows downward to cover the ostium secundum, leaving a small opening called the foramen ovale on the right atrium side. This passage allows oxygenated blood (that comes from the placenta) to bypass the lungs and flow directly from the right atrium through the foramen ovale and ostium secundum, into the left atrium, and out to the rest of the body. At birth, the septum secundum and septum primum fuse, closing off the foramen ovale and allowing the lungs to provide oxygenation. The majority of atrial septal defects are secondary to the ostium secundum remaining open. A smaller percentage of defects are due to the ostium primum not closing. Patients with small atrial septal defects are typically asymptomatic. Larger defects may lead to heart failure, respiratory distress, or failure to thrive. Other complications include pulmonary hypertension, atrial arrhythmias (atrial fibrillation), stroke from paradoxical emboli or atrial fibrillation (AFib). Murmur can be difficult to appreciate on exam, but it is classically described as a soft, systolic ejection murmur at pulmonary area with an associated fixed split second heart sound. Diagnosis is made with an echocardiogram. Treatment may include observation for spontaneous closure of the defect or surgical means to patch or plug the opening. Coarctation of the aorta (B) is a congenital heart defect that commonly occurs in association with other heart defects such as bicuspid aortic valve or ventricular septal defect. The condition is characterized by a narrowing of the descending portion of the aorta that most commonly occurs around the region of the ductus arteriosus. In cases of moderate-severe narrowing, infants may display tachypnea, irritability, and difficulty feeding. Mild cases may not be identified until later in life as they may be asymptomatic. The most common presentation in adults is hypertension, followed by headaches, cold feet, leg cramps, or chest pain. Physical exam reveals a higher systolic blood pressure in the upper extremities than the lower extremities and reduced or delayed lower extremity pulses. If untreated, complications include heart failure and intracranial aneurysms. Diagnosis is suspected based on blood pressure discrepancy and confirmed with echocardiogram. Treatment options include surgery and balloon angioplasty. Patent ductus arteriosus (C) is a congenital heart disorder marked by the persistent patency of the ductus arteriosus vessel that normally closes shortly after birth. This communication between the aorta and pulmonary vein allows blood that has been pumped from the heart into the aorta to go back into the pulmonary vein through the ductus arteriosus. This results in a higher volume of blood flow going back through the pulmonary circulation which can lead to complications such as congestive heart failure and pulmonary hypertension. Diagnosis is made with echocardiogram, and treatment is aimed at closing the vessel with medical therapy such as indomethacin or by surgical means. Ventricular septal defect (D) is a congenital heart defect that occurs when there is a hole in the interventricular septum which can result in a left-to-right shunt of oxygenated blood from the left to right ventricle. Complications include pulmonary hypertension and heart failure. Patients may be asymptomatic in cases of small defects or present with fatigue, poor feeding, tachypnea, and failure to thrive. Physical exam classically reveals a harsh, holosystolic murmur. Diagnosis is made with echocardiogram, and treatment is with surgical closure of the defect.

A 43-year-old man presents to the clinic for his annual physical exam. You order an ECG with findings shown above. Physical examination reveals no abnormalities and the patient's vital signs are within normal limits for his age. What is the most likely diagnosis? A) Brugada syndrome B) Left bundle branch block C) Right bundle branch block D) Wolff-Parkinson-White syndrome

Explanation: Brugada syndrome is an autosomal dominant genetic disorder defined by characteristic ECG findings. The diagnosis is associated with an increased risk of ventricular tachydysrhythmias and sudden cardiac death. Etiologies for the development of Brugada syndrome include mutations in the cardiac sodium channel genes, right ventricular abnormalities, autonomic tone, fever, and the use of cocaine and psychotropic drugs (e.g., amitriptyline, haloperidol, olanzapine). It is usually diagnosed in adulthood, with an average age of occurrence of 41 years, and is more common in men. Brugada pattern ECGs are significantly more common in schizophrenia patients. The most significant risk factor for Brugada syndrome is the presence of a first-degree relative with either sudden cardiac death or Brugada ECG patterns. Sudden cardiac death may be the first manifestation of Brugada syndrome in up to one-third of patients. Ventricular fibrillation and polymorphic ventricular tachycardia are the most common presenting dysrhythmias and are most likely to occur between the ages 22 and 65 with a predominance during sleep or at night. Syncope, unrelated to exercise, is an initial presentation in another one-third of patients. Atrial dysrhythmias, particularly atrial fibrillation, are more likely to occur in patients with Brugada syndrome. The presence of atrial fibrillation has been associated with an increased risk of ventricular fibrillation. Sudden, unexpected nocturnal death and nocturnal agonal respiration have been correlated with Brugada syndrome and can be associated with low serum potassium levels and aborted cardiac dysrhythmias, respectively. Clinical criteria have been created for the accurate diagnosis of Brugada syndrome. This criteria divides Brugada syndrome into type 1 and type 2 variants. Type 1 Brugada syndrome demonstrates a characteristic coved-type ST segment elevation in more than one right precordial lead (V1-V2). This type is demonstrated on ECG as a steep-sloped elevation at the end of the QRS complex followed by downsloping ST. A saddle-back type ST segment elevation is suggestive of type 2 Brugada syndrome when found in more than one right precordial lead. The ST segment is followed by positive T wave in V2 with variable morphology in V1. This ECG pattern converts to type 1 ECG findings after administration of a sodium channel blocker. Additionally, a patient diagnosed with either subtype must also have either a family history of sudden cardiac death at younger than 45 years of age, a relative with type 1 Brugada pattern ECG findings, a documented history of either ventricular fibrillation or polymorphic ventricular tachycardia, inducible ventricular tachycardia during electrophysiology study, unexplained syncope, or nocturnal agonal respiration. Sodium channel blockers (e.g., flecainide, procainamide) can be administered to unmask an underlying type 1 Brugada ECG pattern. This drug challenge is useful in stratifying risk of sudden cardiac death in asymptomatic patients and those symptomatic patients without previously demonstrated type 2 Brugada pattern. Echocardiography can be utilized to rule out structural heart disease. Patients who require further risk stratification may undergo invasive electrophysiology testing. Any patient with documented Brugada syndrome should undergo genetic testing with subsequent testing of first-degree relatives if a mutation is identified. Sudden cardiac death prevention is the primary treatment goal for patients diagnosed with Brugada syndrome. Patients with a history of sudden cardiac arrest or syncope suspected to be secondary to ventricular tachydysrhythmia should undergo placement of an implantable cardioverter-defibrillator to terminate future ventricular dysrhythmias. In those deemed an unsuitable candidate for cardioverter implantation, therapy should be initiated with either quinidine or amiodarone. Patients with an implantable cardioverter-defibrillator who have recurrent dysrhythmia and subsequent shocks may undergo catheter ablation of right ventricular outflow tract causing the anomalies. Most deaths related to Brugada syndrome occur secondary to ventricular fibrillation or polymorphic ventricular tachycardia. Clinical history and repeat ECG screening should be completed every one to two years in first-degree relatives of a patient with Brugada syndrome who initially screen negative. Left bundle branch block (B) presents with widened QRS complexes in leads I, avL, and V6, as opposed to the changes found in V1-V2 of Brugada syndrome. Left bundle branch blocks can interfere with the correct diagnosis of ventricular hypertrophy, myocardial ischemia, and acute myocardial infarction. The ECG findings of right bundle branch block (C) present similarly to Brugada syndrome ECG findings, however, the QRS complex of Brugada syndrome is much wider than that of a right bundle branch block. Additionally, right bundle branch block ECG findings have a qRS pattern in lead V6 that is absent in Brugada syndrome ECGs. Wolff-Parkinson-White syndrome (D) is commonly found in asymptomatic patients via routine ECG, similarly to Brugada syndrome. The ECG findings, however, demonstrate a shortened PR interval with an early ventricular activation, termed a delta wave.

A 48-year-old man presents to the clinic for evaluation of occasional, fleeting chest pain on exertion. An ECG reveals normal sinus rhythm with a left bundle branch block. The patient is currently asymptomatic. Which of the following represents the next best step in evaluating this patient's cardiac health? A) Cardiac electrophysiologic study B) Chest X-ray C) Echocardiography D) Serum creatinine kinase and troponin I levels

Explanation: Bundle branch block is a conduction abnormality affecting the electrical pathways of the right or left ventricle or both. Left bundle branch block refers to impaired or delayed conduction through the left ventricle and is often asymptomatic. Causes of left bundle branch block include hypertension, cardiomyopathy, and coronary artery disease. Left bundle branch block most often occurs gradually in the presence of chronic heart disease. However, occasionally an acute myocardial infarction or myocarditis can cause abrupt onset of left bundle branch block. ECG is diagnostic of left bundle branch block and will show a widened QRS complex, widened R waves and absent Q waves in the lateral leads, and ST segments and T waves opposite in direction from the QRS complexes. ECG does not provide all the necessary information to diagnose the underlying, causative heart condition. For this reason, if left bundle branch block is found on ECG, echocardiography and cardiac stress testing should be performed. Treatment of left bundle branch block involves addressing the underlying cardiac pathology. Patients with left bunch branch block and syncope may be candidates for permanent pacemaker implantation. Patients with left bunch branch block and decreased ejection fraction may be candidates for cardiac resynchronization. Patients with bundle branch block that progresses to Mobitz type II second-degree heart block or third-degree heart block may require permanent pacemaker insertion. Cardiac electrophysiologic study (A) would not provide any additional useful information to guide treatment for this patient. Indications for invasive cardiac electrophysiologic studies include syncope in patients with pre-existing cardiac disease, survivors of sudden cardiac death of unknown etiology, and patients with atrioventricular block that is not otherwise explained by clinical and ECG findings. Chest X-ray (B) may show cardiomegaly or signs of congestive heart failure, but echocardiography will demonstrate more specific findings and more effectively guide treatment. Serum creatinine kinase and troponin levels (D) would be indicated in the emergency department for a patient presenting with left bundle branch block and chest pain or other clinical signs of an acute cardiac ischemic event.

A 65-year-old man presents with chest pain, dyspnea, jugular venous distension, and hypotension. He sustained blunt chest trauma from a motor vehicle accident one hour prior to arrival. ECG shows electrical alternans. Which of the following is the most likely diagnosis? A) Acute myocardial infarction B) Cardiac tamponade C) Pulmonary contusion D) Tension pneumothorax

Explanation: Cardiac tamponade is the accumulation of pericardial fluid under pressure, usually secondary to a rapidly-developing pericardial effusion. The increased pressure leads to compression of the cardiac chambers and decreased systemic venous return and cardiac output. Cardiac tamponade can be caused by infectious etiologies such as pericarditis, acute bleeding, post-MI with free wall rupture and trauma. Signs and symptoms include chest pain, hypotension, sinus tachycardia, elevated jugular venous pressure, muffled heart sounds, narrowed pulse and pulsus paradoxus (an exaggerated drop in systemic blood pressure with inspiration). ECG may show electrical alternans, a beat-to-beat alteration in the QRS complex that is caused by the heart swinging in the pericardial fluid. Though electrical alternans is specific for cardiac tamponade, it is not sensitive. Chest X-ray may show a widened cardiac silhouette from the fluid accumulation. Definitive diagnosis can be made with echocardiography, computed tomography, and cardiac magnetic resonance imaging. Treatment consists of emergent surgery or pericardiocentesis.

In which of the following locations does the mechanism of action of statins take place? A) Distal convoluted tubule B) Liver hepatocytes C) Loop of Henle D) Proximal convoluted tubule

Explanation: Cardiovascular disease is a common cause of morbidity and mortality worldwide. Treatment of hypercholesterolemia by reducing low density lipoprotein-cholesterol and increasing high density lipoprotein-cholesterol is important in reducing the incidence of myocardial infarctions, peripheral vascular disease, and cerebrovascular accidents. Hypercholesterolemia is also frequently a comorbid condition with hypertension, diabetes mellitus, obesity, and metabolic syndrome, leading to an even greater risk of complications from cardiovascular disease. Risk evaluation for cardiovascular disease should begin at 20 years of age and there are a number of risk calculator tools that may be used to help guide treatment. Patients with hypercholesterolemia should be counseled on lifestyle interventions including weight loss, diets that are low in saturated fats, and aerobic exercise. Pharmacotherapy should be initiated in patients with a 10-year cardiovascular disease risk of 7.5-10% or higher. When medication is indicated, statin therapy is first-line. The mechanism of action of statins takes place in the liver hepatocytes, where they inhibit HMG-CoA reductase, an enzyme that is involved in the early phases of low density lipoprotein-cholesterol synthesis. Statin therapy may also result in a decrease in triglycerides and an increase in high density lipoprotein-cholesterol. Most patients have few side effects with statins. When side effects occur, they most commonly include nausea, constipation, diarrhea, mild headache, and muscle aches. Serious side effects are rare, with hepatotoxicity and severe muscle pain due to statin-induced damage to the muscles being the most concerning. The distal convoluted tubule (A), loop of Henle (C), and proximal convoluted tubule (D) are all parts of the kidney. Statins are metabolized in the liver, not the kidney, so these areas are not involved in the metabolism of statins.

An 82-year-old woman presents with two months of intermittent, atraumatic right leg pain. She states her pain comes when she tries to walk her dog down the street. The pain generally resolves after 10-15 minutes of rest but will start again after she starts walking. Which of the following is the most likely diagnosis? A) Chronic arterial insufficiency B) Chronic venous insufficiency C) Deep vein thrombosis D) Lumbosacral radiculopathy

Explanation: Chronic arterial insufficiency is a chronic obstruction of peripheral arteries secondary to atherosclerosis. Risk factors include age > 70 years, smoking (the most important risk factor), diabetes, hypertension, dyslipidemia, and CAD. The most common presenting symptoms of chronic arterial insufficiency include cramping leg pain and intermittent claudication, however, many patients are largely asymptomatic. Intermittent claudication is characterized by pain in the affected extremity during exercise/walking that resolves with rest. This pain is caused by ischemia to the extremity. Rest pain suggests severe ischemia. Patients usually presents with prominent leg pain at night; hanging the foot over side of bed or standing improves pain. Rarely, patients presenting with severe disease can have a threatened limb from critical arterial stenosis. Signs and symptoms of a threatened limb include decreased or absent distal pulses, severe pain and pallor. Diagnosis is made by physical exam and measurement of ankle-brachial index (normal is between 0.9 and 1.3), exercise testing, and vascular imaging. The gold standard diagnostic test is arteriography. Treatment consists of risk factor modification, such as smoking cessation, treating underlying comorbidities, antithrombotic therapy, such as aspirin or clopidogrel, and in severe cases, surgical intervention. Surgical treatment is indicated in patients with rest pain, ischemic ulcerations (tissue necrosis), sever symptoms refractory to conservative management. Surgical options include angioplasty (balloon dilation with or without stenting) and surgical bypass grafting. Chronic venous insufficiency (B) is characterized by dilated veins, skin changes, and ulcerations. Patients may also have feelings of heavy legs and leg pain, however, intermittent claudication is not a common symptom of chronic venous insufficiency. Deep vein thrombosis (C) often presents with unilateral edema, erythema, and pain. Peripheral arterial disease does not generally have erythema or edema. Lumbosacral radiculopathy (D) presents with low back pain that radiates down the lateral and posterior leg. A straight leg raise maneuver will reproduce the symptoms in most patients with lumbosacral radiculopathy.

In individuals with coarctation of the aorta, where is the narrowing of the aorta most commonly located? A) Atrial septum B) Distal to the foramen ovale C) Distal to the left subclavian artery D) Ventricular septum

Explanation: Coarctation of the aorta is a cardiac condition that causes a pressure overload on the left ventricle due to narrowing of the descending aorta. It is most commonly seen in children, with males affected more often than females. This condition makes up 4-6% of all congenital heart defects. In vitro, coarctation of the aorta is caused by a developmental abnormality of the embryologic fourth and sixth aortic arches on the left. The anatomic location of the narrowing is most commonly distal to the left subclavian artery at the insertion of the ductus arteriosus. Risk factors for coarctation of the aorta include family history of the condition and Turner syndrome. Symptoms of the disorder in early life include poor perfusion to the lower body, congestive heart failure, and severe acidosis. Individuals may be asymptomatic if the coarctation is mild. Older children with coarctation of the aorta are generally asymptomatic, and as they grow into adulthood, the most common sign is hypertension. It is difficult to diagnose coarctation of the aorta with prenatal ultrasound because only a small percentage of the fetal cardiac blood flow passes across the defect. After birth, the diagnosis is based on physical findings, including systolic hypertension in the arms, delayed or decreased femoral pulses, and low or undetectable arterial blood pressure in the legs. Echocardiogram provides the definitive diagnosis and usually reveals left ventricular hypertrophy. CXR may reveal notching of the ribs and "figure 3" appearance due to indentation of the aorta at site of coarctation. Treatment is surgical, with the preferred procedure being to resect the coarctation and create an end-to-end anastomosis to repair it. Prognosis is poor for those not treated with most not surviving past the fourth decade of life.

Which of the following best describes the predominant pathogenesis of lower extremity edema in congestive heart failure? A) Decreased albumin production B) Decreased arterial pressure C) Increased capillary permeability D) Increased venous pressure

Explanation: Congestive heart failure refers to decreased cardiac output as a result of cardiac disease, which leads to increased venous pressure and decreased tissue perfusion. Causes of congestive heart failure include coronary artery disease, valvular disease, cardiomyopathy, hypertension, or cor pulmonale. As the cardiac output decreases, blood that is not adequately pumped into the arterial system backs up, leading to increased venous pressure. Vital organ perfusion, including renal perfusion, decreases, which leads to compensatory renal sodium retention. Both the increase in venous pressure and the renal sodium retention lead to edema in the heart failure patient. Left heart failure typically produces pulmonary edema, whereas right heart failure produces lower extremity edema and ascites. Patients with cardiomyopathy often demonstrate pulmonary edema as well as peripheral edema. Signs and symptoms of congestive heart failure include shortness of breath, dyspnea, decreased exercise tolerance, increased abdominal girth, lower extremity edema, hypotension, and orthopnea. Signs of congestive heart failure on chest X-ray include cardiomegaly, pleural effusion, and increased vascular markings. Definitive diagnosis of congestive heart failure is made on echocardiogram where the ejection fraction is used to measure disease severity. A left ventricular ejection fraction under 53% for women and 51% for men, along with signs and symptoms of congestion, is diagnostic of congestive heart failure. A left ventricular ejection fraction under 30% in a symptomatic patient is considered severe heart failure. Treatment of congestive heart failure involves addressing any underlying pathology, managing hypertension, restricting sodium and fluid intake, daily monitoring of weight, smoking cessation, and losing weight. Pharmacologic agents that may prolong patient survival include angiotensin-converting enzyme inhibitors, angiotensin receptor-neprilysin inhibitors, beta-blockers, hydralazine plus nitrate, and aldosterone antagonists. Diuretics are often prescribed to treat symptoms of congestion and edema, although benefits to long-term survival are unclear. A select group of patients with severe congestive heart failure may benefit from internal defibrillators and cardiac resynchronization therapy. Decreased albumin production (A) is a cause of edema in patients with malnutrition, liver disease, or renal disease. It is not a common cause of edema in patients with heart failure. Decreased arterial pressure (B) is a late finding in patients with severe congestive heart failure but is not a cause of edema. As the heart pump becomes weak, the left ventricular ejection fraction decreases, and cardiac output decreases as well. This weakening can lead to hypotension and decreased arterial pressure. However, capillary beds become leaky when venous pressure increases, as a result of inadequate cardiac emptying, resulting in edema. Increased capillary permeability (C) is the result of vascular injury and occurs in disease states, such as burns, diabetes mellitus, idiopathic capillary leak syndromes, kwashiorkor, and adult respiratory distress syndrome.

An 85-year-old man is pulseless and apneic. Chest compressions are initiated and an electrocardiogram (ECG) shows the above rhythm. Which of the following is the next best step? A) Adenosine B) Atropine C) Defibrillation D) Magnesium sulfate

Explanation: Defibrillation is indicated in all pulseless patients with ventricular fibrillation or ventricular tachycardia. Early defibrillation is associated with improved outcomes and higher likelihood of successful return of circulation and survival. Ventricular fibrillation usually occurs as a result of electrophysiologic alterations in an already-diseased heart such as those with coronary artery disease or structural heart disease. This leads to a lack of coordinated depolarization and resultant fibrillation within the ventricular myocardium leading to a pulseless state. Non-cardiac causes of ventricular fibrillation include certain medications such as antiarrhythmics and electrolyte abnormalities such as hypokalemia or hypomagnesemia. Ventricular fibrillation is a pulseless rhythm, so patients present as unresponsive, apneic, and without a pulse. ECG tracings show disorganized rapid waveforms without any discernible QRS complexes which decrease in amplitude over time and eventually lead to asystole if left untreated. Once a diagnosis of cardiac arrest has been established, cardiopulmonary resuscitation (CPR) should be performed with as few interruptions as possible until a defibrillator is present. If after two minutes of CPR and one attempt at defibrillation there is no return of spontaneous circulation (ROSC), epinephrine 1 mg is administered and continued every three to five minutes until ROSC. If above fails, other antiarrhythmic agents such as IV amiodarone (preferred), lidocaine, magnesium, and procainamide may be used. In the absence of ROSC, CPR can be terminated if there is no sustained perfusing rhythm after at least 30 minutes of resuscitation, obvious signs of death, absent brainstem reflexes, asystole, or presence of severe comorbidities. Adenosine (A) is used in the management of regular, monomorphic tachycardias. It is administered in an initial dose of 6 mg via rapid intravenous push followed immediately by a saline flush, followed by an additional 12 mg dose if ineffective. For stable wide QRS tachycardias, procainamide, amiodarone, or sotalol can be utilized. Atropine (B) is utilized in the management of bradycardia with hypotension and hemodynamic instability. It is given in 0.5 mg increments every three to five minutes until 3 mg has been administered. If ineffective, dopamine, epinephrine, or transcutaneous pacing can be utilized. Magnesium sulfate (D) is the treatment of choice for Torsades de pointes, a polymorphic ventricular tachycardia triggered by a prolonged QT interval.

Which of the following is the most likely diagnosis based on the ECG rhythm strip shown above? A) Atrial fibrillation B) First-degree atrioventricular block C) Normal sinus rhythm D) Sinus bradycardia

Explanation: ECG findings can aid clinicians in the evaluation and diagnosis of dysrhythmias, conduction disorders, and myocardial ischemia. Twelve-lead ECGs are superior to three-lead ECGs when determining the location of pathologic involvement as they provide a 3-dimensional analysis of electrical conduction. The components evaluated on an ECG tracing include the P wave, PR interval, QRS complex, ST segment, and T wave. The P wave corresponds to atrial depolarization and is included in the PR interval, which is the distance from the beginning of the P wave to the start of the QRS complex. This interval provides information on the time it takes for electrical impulses to travel from the sinoatrial node through the atrioventricular node, bundle of His, and Purkinje fibers. Ventricular depolarization is indicated by the QRS complex with initial negative deflections representing the Q wave, a positive deflection representing the R wave, and the final negative deflection representing the S wave. The T wave identifies atrial repolarization, is separated from the QRS complex by the ST segment, and is used primarily to interpret myocardial ischemia. Systematic interpretation of the ECG involves evaluation of the rate and rhythm followed by the axis, intervals, P wave, QRS complex, and ST segment. A normal sinus rhythm has a rate between 60-100 beats per minute and regular rhythm with a narrow QRS complex, uniform P wave present prior to all QRS complexes, and nonelevated ST segment. Atrial fibrillation (A) is characterized by its irregularly irregular rhythm with no discernable P waves prior to QRS complexes. First-degree atrioventricular block (B) presents with a prolonged PR interval of greater than 200 milliseconds. Sinus bradycardia (D) is defined as a sinus rhythm with a heart rate that is slower than 60 beats per minute.

Which of the following describes a classic electrocardiogram finding in a patient with an anterior ST-segment elevation myocardial infarction? A) ST-segment elevation in leads I, aVL, V5, and V6 B) ST-segment elevation in leads II, III, and aVF C) ST-segment elevation in leads V1, V2, V3, and V4 D) Tall R waves in leads V1 and V2

Explanation: Early electrocardiograms (ECGs) are essential in the evaluation and diagnosis of patients with suspected acute coronary syndrome (ACS). However, an initially normal electrocardiogram does not exclude ischemia or infarction, and electrocardiograms should be repeated at five- or 10-minute intervals in cases with high clinical suspicion for acute coronary syndrome. Patients with a suspected myocardial infarction (MI) can be initially classified based on the first electrocardiogram findings as having a ST-segment elevation myocardial infarction (STEMI), a non-ST-segment elevation acute coronary syndrome (including non-ST elevation myocardial infarction and unstable angina), or undifferentiated chest pain syndrome. Undifferentiated chest pain syndrome indicates that the initial ECG did not show ischemic changes. However, based on clinical suspicion, these patients should have repeat ECGs and cardiac enzymes, such as troponin, obtained. There is a typical sequence of changes on the electrocardiogram for patients with a ST-segment elevation myocardial infarction. The earliest change is the development of hyperacute (peaked) T waves. Next, the ST-segment elevates in the leads recording electrical activity of the involved region of the myocardium. The electrocardiogram criteria required for the diagnosis of a ST-segment elevation myocardial infarction are new ST-segment elevation at the J point in two contiguous leads of at least 0.1 millivolts (mV) in all leads except leads V2 and V3. For leads V2 and V3, the cutoff point is ≥ 0.20 mV in men over 40 years old, ≥ 0.25 mV in men younger than 40 years, and ≥ 0.15 mV in women. Over time, there is further change of the electrocardiogram findings, including gradual return of the ST-segment to baseline, marked reduction in the R wave amplitude, Q wave deepening, and T wave inversion. These changes generally occur within the first two weeks but may progress more rapidly. Q waves are a clinically important electrocardiogram finding but are not required for the electrocardiogram diagnosis of a myocardial infarction. Q waves are generally considered pathologic (suggestive of current or prior myocardial infarction) if they are ≥ 0.03 seconds in duration and ≥ 0.1 mV deep. Whether a Q wave occurs on the electrocardiogram depends on the size of the myocardial infarction. In a non-ST-segment elevation myocardial infarction, the typical electrocardiogram findings are different. The classic findings are ST-segment depression and T wave inversion in two or more contiguous leads. T wave flattening or inversion usually precedes ST-segment depression, and Q waves are typically absent. However, since reciprocal ST-segment depression can occur during a ST-segment elevation myocardial infarction, these conditions can be confusing. Therefore, whenever there is ST-segment depression, the electrocardiogram should be carefully assessed for ST-segment elevation. Identifying which electrocardiogram leads are involved can predict the location of the ischemia or infarction. Ischemia or infarction causing ST-segment shifts or Q waves in leads V1-V4 is considered anterior, and changes in the ST-segment of leads V5-V6, I, and aVL are considered to suggest lateral ischemia or infarction. ST-segment shifts or Q waves in leads II, III, and aVF suggest inferior wall ischemia or infarction. The electrocardiogram leads also correlate with the involved coronary artery, such as the left anterior descending (LAD) artery correlating with leads V1-V4; the circumflex artery correlating with leads I, aVL, V5, and V6; and the right coronary artery correlating with leads II, III, and aVF. In addition to ECG findings, cardiac enzymes are important in the diagnosis of acute myocardial infarction. Cardiac troponin (cTn) I and T are specific and sensitive markers of cardiac injury and are the preferred serologic tests for the evaluation of patients with suspected acute myocardial infarction. Patients with ST elevation myocardial infarction and non-ST elevation myocardial infarction have elevated cardiac enzymes, whereas unstable angina causes ischemic ECG changes without elevations in cardiac enzymes. ST-segment elevation in leads I, aVL, V5, and V6 (A) suggests a lateral myocardial wall infarction. ST-segment elevation in leads II, III, and aVF (B) suggests an inferior myocardial wall infarction. The right coronary artery supplies the inferior myocardial wall. Tall R waves in leads V1 and V2 (D) are seen in posterior myocardial wall myocardial infarction, right ventricular hypertrophy, dextrocardia, Wolff-Parkinson-White pattern, hypertrophic cardiomyopathy, lead malposition, and dextrocardia. ST-segment depression in leads V1-V2 is also seen in posterior myocardial wall infarctions.

Which of the following reduces a patient's risk of developing heart failure? A) Controlling hypertension B) Excessive alcohol intake C) Increasing fluid intake D) Smoking cigarettes

Explanation: Heart failure is the inability of the heart to pump sufficient amount of blood to meet the metabolic demands of the body. There are many causes and forms of heart failure. The most common causes are coronary artery disease and uncontrolled hypertension. Controlling hypertension either with lifestyle changes or pharmacologic therapy reduces the risk of heart failure. Other causes of heart failure include valvular heart disease, congenital heart disease, cardiomyopathies, and high-output causes of heart failure, such as severe anemia, wet beriberi, arteriovenous shunting, and thyrotoxicosis. There is left-sided heart failure and right-sided heart failure. There is also systolic and diastolic heart failure. In systolic heart failure, the problem is related to decreased ejection fraction due to impaired contractility. In diastolic heart failure, the heart may have a normal ejection fraction, but the ventricles are not being filled sufficiently due to impaired relaxation, increased stiffness, or both. Heart failure presents most commonly with dyspnea, which is initially exertional. Patients commonly have pulmonary edema, peripheral lower extremity edema, and other exam findings, including jugular venous distention and hepatic congestion. Echocardiogram is the most helpful diagnostic tool. It is useful in determining whether systolic (EF < 40%) or diastolic (EF > 40%) dysfunction predominates. In addition, chest X-ray may show cardiomegaly and pulmonary edema. B-type natriuretic peptide (BNP > 150 pg/mL) is a lab finding that can be helpful in indicating heart failure as the cause of a patient's dyspnea. Treatment involves lifestyle modification, which may include sodium restriction, fluid restriction, exercise, and smoking cessation. Pharmacologic treatment involves loop diuretics, such as furosemide, angiotensin converting enzyme inhibitors, beta-blockers, and digoxin. Mineralocorticoid receptor antagonists, such as spironolactone, and hydralazine may also be used. Ultimately, patients may need a ventricular assist device or heart transplant.

Which of the following ocular findings is a possible complication of uncontrolled primary hypertension? A) Increased intraocular pressure B) Inflammation of the eyelid C) Opacification of the lens D) Optic disc swelling

Explanation: Primary (essential) hypertension most commonly presents as an asymptomatic elevated blood pressure reading. However, sudden severe hypertension, such as hypertensive urgency or emergency, can cause dangerous acute clinical manifestations. Hypertensive urgency is diagnosed in patients who are relatively asymptomatic with blood pressure that is ≥ 180/120 mm Hg. Hypertensive emergency is diagnosed when patients with significantly elevated blood pressure have acute target-organ damage. Malignant hypertension is now considered an outdated term. Target-organ damage may include damage to the kidneys, brain, heart, eyes, and other vasculature. Patients presenting with severely elevated blood pressure (≥ 180/120 mm Hg) should be evaluated for focal neurologic symptoms, retinal changes, nausea and vomiting, chest discomfort, severe chest or back pain, dyspnea, pregnancy, and use of drugs that can produce a hyperadrenergic state. Focal neurologic symptoms could suggest an acute ischemic or hemorrhagic stroke. Retinal changes, including hemorrhages, exudates (cotton-wool spots), or papilledema, can be seen with hypertension. In severe cases, optic disc swelling, which represents papilledema, can be seen. Nausea and vomiting can suggest an increase in intracranial pressure. Chest discomfort may be indicative of myocardial ischemia. Sudden onset of severe pain with a ripping or tearing quality in the chest or back may suggest aortic dissection. Dyspnea may be associated with acute pulmonary edema secondary to severe hypertension. Pregnancy can cause preeclampsia or eclampsia, which present with hypertension. Drugs, such as cocaine, amphetamines, monoamine oxidase inhibitors, or recent discontinuation of clonidine, can produce a hyperadrenergic state. Electrocardiography (ECG) should be done in all patients with severe hypertension, and chest radiography, serum electrolytes, serum creatinine, urinalysis, and cardiac biomarkers may also be needed depending on the history and physical examination findings. In addition to the immediate risks of significant acute hypertension, chronic hypertension is associated with a significant increase in the risk of adverse cardiovascular and renal outcomes. Left ventricular hypertrophy (LVH), systolic and diastolic heart failure, ischemic stroke, intracerebral hemorrhage, ischemic heart disease, chronic kidney disease, and retinal changes are each closely associated with the presence of long-term uncontrolled hypertension. Hypertension, along with diabetes, dyslipidemia, and smoking cigarettes, is a major modifiable risk factor for premature cardiovascular disease. Increased intraocular pressure (A) is associated with the development of glaucoma. Uncontrolled hypertension is associated with causing increased intracranial pressure but not increased intraocular pressure. Inflammation of the eyelid (B) is called blepharitis. Blepharitis is associated with dermatologic conditions, such as rosacea or seborrheic dermatitis, but is not associated with uncontrolled hypertension. Opacification of the lens (C) is the classic description of a cataract. Opacification of the lens is most commonly associated with aging and is not classically associated with uncontrolled hypertension.

A 67-year-old man presents with visual changes and headache. His blood pressure is 220/120 mm Hg, and you diagnose him with hypertensive emergency. By what percentage should his blood pressure be lowered in the first hour? A) 50 % B) B< 10 % C) C10-20 % D) D25-40 %

Explanation: Hypertensive emergencies are a relatively uncommon condition that involve a severely elevated blood pressure, generally systolic pressure ≥180 or diastolic pressure ≥120 mmHg, and signs of end-organ damage. Severely elevated blood pressure in the absence of signs of end-organ damage is called severe asymptomatic hypertension and is not generally managed the same as a hypertensive emergency. Signs of end-organ damage include agitation, stupor, visual changes, chest pain, shortness-of-breath, flame hemorrhages, and papilledema. For patients who have both hypertension and signs of end-organ damage, prompt blood pressure lowering must be initiated. In most cases, it is recommended to lower the blood pressure by 10-20% in the first hour. This generally achieves the goal blood pressure of < 180/< 120 mm Hg. The blood pressure should be lowered further by 5-15% over the next 23 hours. Lowering the blood pressure too rapidly may cause hypoperfusion to the brain if the vessels have autoregulated to the higher blood pressure. Lowering the blood pressure too slowly may result in permanent end-organ damage. Intravenous antihypertensive agents (hydralazine, esmolol, nitroprusside, labetalol, or nitroglycerin) are generally used initially to lower the blood pressure to target range. The specific medication choice depends on the type of end-organ damage that is occuring. Lowering the blood pressure by 25-40% (B), and >50% (D) in the first hour is not recommended due to the risk of hypoperfusion brain injury. Lowering the blood pressure <10% (C) in the first hour would be inadequate in the treatment of hypertensive emergency.

A 48-year-old man presents to the clinic stating his triglycerides were elevated at a community cholesterol screening event. He states he feels well, takes no medications, and denies any chronic health problems. In addition to a fasting lipid panel, which of the following diagnostic studies is the next best step in evaluating this patient? A) Complete metabolic panel (fasting) B) Electrocardiogram C) Exercise stress test D) Hemoglobin A1C

Explanation: Hypertriglyceridemia describes a blood triglyceride level over 150 mg/dL and can be further subdivided into three categories: mild (151-499 mg/dL), moderate (500-886 mg/dL), and severe (887 mg/dL or above). The causes of hypertriglyceridemia include diabetes mellitus, obesity, sedentary lifestyle, hypothyroidism, nephrotic syndromes, pancreatitis, familial hypertriglyceridemia, and excessive alcohol intake. Medications, such as corticosteroids, immunosuppressants, protease inhibitors, retinoids, some beta-blockers, tamoxifen, and estrogen, can all cause increases in serum triglyceride levels. A patient with hypertriglyceridemia should have the following laboratory studies: fasting lipid panel, fasting blood glucose, thyroid-stimulating hormone, urinalysis, and liver function studies. Hypertriglyceridemia is usually asymptomatic, but symptoms of other disease states that contribute to high triglyceride levels may be evident, and severe hypertriglyceridemia can lead to fatty deposits in the skin known as xanthomas. At triglyceride levels of 1,000-2,000 mg/dL, patients may develop pancreatitis. Treatment of hypertriglyceridemia involves controlling the underlying disease process. If hypertriglyceridemia is accompanied by increased low-density lipoproteins levels, treatment with a high potency statin drug, such as atorvastatin, is the treatment of choice. In a patient with triglyceride levels over 500 mg/dL, a prescription-strength omega-3 fatty acid ethyl ester is indicated. Electrocardiogram (A) is not the most appropriate next step in evaluating this patient's hypertriglyceridemia. Elevated triglycerides are a risk factor for cardiovascular disease, but the patient is asymptomatic. The root cause of the hypertriglyceridemia should be investigated first through careful history and bloodwork. Exercise stress test (B) may be indicated in the future for this patient, but he is currently asymptomatic and costly tests should be avoided while lifestyle modification and pharmacologic treatment for his hypertriglyceridemia have yet to be instituted. Hemoglobin A1C (D) would be appropriate for this patient if his fasting blood glucose is elevated. The fasting blood glucose, as well as liver enzymes, would be evaluated in a complete metabolic panel, making it a better choice for initial workup in a patient with elevated triglycerides.

A 40-year-old man with a history of type 2 diabetes mellitus and alcohol use disorder presents to your office to review recent laboratory results. Lipid panel reveals a triglyceride count of 900 mg/dL. A diagnosis of hypertriglyceridemia is made. Which of the following is the most appropriate initial therapy? A) Fenofibrate B) Fenofibrate plus fish oil C) Losartan D) Nicotinic acid

Explanation: Hypertriglyceridemia is diagnosed when triglyceride levels are elevated and is categorized as mild, moderate, or severe. It is often asymptomatic and detected on a screening lipid panel. Hypertriglyceridemia may be caused or worsened by obesity, poorly controlled diabetes mellitus, and sedentary lifestyle. Individuals with hypertriglyceridemia are at a higher risk of developing coronary artery disease, and extremely high levels greater than 1,000 mg/dL can lead to acute pancreatitis. Most patients are asymptomatic, especially with mild to moderate cases of elevated triglycerides. When symptoms present, they include nausea, vomiting, abdominal pain, shortness of breath, xanthomas, and ophthalmologic findings, such as corneal arcus and xanthelasmas. In patients with an elevated triglyceride level, evaluation should be done to determine the cause, which may be genetic or due to metabolic disease or drugs. Workup is done to confirm that the triglyceride results were done fasting and also to rule out secondary causes, such as hypothyroidism, diabetes mellitus, renal failure, hormone replacement therapy, or alcohol abuse. Patients should be counseled on lifestyle modifications before starting any pharmacotherapy. These modifications include smoking cessation, weight loss for those who are overweight or obese, regular aerobic exercise, avoidance of alcohol, and a healthy diet that is low in saturated fat and cholesterol. Consideration for initiating pharmacotherapy should begin at triglyceride levels of 500 mg/dL or higher and should definitely be started in patients when they reach the severe triglyceridemia group, which is greater than 886 mg/dL. First-line treatment is with a fibrate, such as fenofibrate or gemfibrozil, and a response may be seen in as soon as two weeks.

A 15-year-old boy presents to his pediatrician for a routine pre-athletics physical screening. He has a history of occasionally feeling like he is going to faint after strenuous workouts. A screening ECG is ordered and notable for down-sloping ST segments and T wave inversions, concerning for left ventricular hypertrophy. Which of the following is the best diagnostic study to confirm this diagnosis? A) Cardiac catheterization B) Cardiovascular magnetic resonance C) Echocardiography D) Exercise testing

Explanation: Hypertrophic cardiomyopathy is a genetically inherited condition that leads to development of left ventricular hypertrophy. This pathology can lead to left ventricular outflow obstruction, diastolic dysfunction, mitral regurgitation, and myocardial ischemia. Symptoms of hypertrophic cardiomyopathy are typically recognized as those related to arrhythmia, chest pain, and heart failure. For example, symptoms may include dyspnea on exertion, fatigue, palpitations, atypical or anginal chest pain, arrhythmias (atrial fibrillation, ventricular arrhythmias), presyncope, or syncope. If present, abnormal heart sounds associated with hypertrophic cardiomyopathy include a fourth heart sound, left ventricular lift, and harsh crescendo-decrescendo systolic ejection murmur best heard at the apex and lower left sternal border with radiation to axilla and base. Many individuals may be asymptomatic, and these, in particular, are at risk for sudden cardiac death associated with supraventricular and ventricular arrhythmias. The best screening test for hypertrophic cardiomyopathy is electrocardiogram to identify left ventricular hypertrophy (LVH). It is a sensitive test; individuals with hypertrophic cardiomyopathy are unlikely to have a normal ECG. However, the abnormalities seen are not specific to hypertrophic cardiomyopathy, so a subsequent echocardiogram is needed to both establish the diagnosis and evaluate the degree of mitral regurgitation, extent of systolic and diastolic dysfunction, and severity of any left ventricular outflow tract gradient, if it exists. After establishing a diagnosis, further evaluation with exercise testing and ambulatory ECG is recommended to look for arrhythmias, ischemia, or obstruction. Genetic testing may also be considered for confirmation of the sarcomere mutation associated with hypertrophic cardiomyopathy. Hypertrophic cardiomyopathy is typically not progressive. Depending on the severity of symptoms and pathology, management can range from supportive care to medical management, plantation of cardioverter-defibrillators, or surgical myectomy. Beta blockers, calcium channel blockers (CCBs), and diuretics can be used for symptomatic patients. All patients should avoid strenuous exercise. Cardiac catheterization (A) provides an excellent analysis of cardiac hemodynamics, however, it is an unnecessarily invasive test given that echocardiography typically provides sufficient data for diagnosis and management. It may be considered in cases where there are discrepancies between clinical presentation and other diagnostic findings. Cardiovascular magnetic resonance (B) is superior to echocardiography for diagnosis given the amount of details it can provide about the structure and anatomy of the heart. However, it is expensive, less-readily available, and requires the use of IV contrast, which has additional risks. It is recommended in cases when echocardiography is not able to provide a definitive diagnosis. Exercise testing (D) is recommended as part of the routine diagnostic evaluation of hypertrophic cardiomyopathy to stratify risk and assess the left ventricular outflow tract gradient. However, this test is usually conducted after the diagnosis has been established through echocardiography.

A 35-year-old woman presents to the clinic with fever and cough and is diagnosed with pneumonia. Physical exam reveals tachycardia with a heart rate of 145 beats per minute. Which of the following is the most appropriate interpretation of the electrocardiogram shown above? A) Atrial fibrillation B) Atrial flutter C) Sinus tachycardia D) Ventricular tachycardia

Explanation: In normal sinus rhythm, electrical impulses originate from the sinoatrial node which results in an electrocardiogram with normal, upright p-waves before each narrow QRS complex at a rate of 60-100 beats per minute. Sinus tachycardia also originates from the sinoatrial node, however, it is distinguished from normal sinus rhythm in that the ventricular rate is greater than 100. Electrocardiogram reveals a regular, narrow complex tachycardia with normal p-waves and a rate greater than 100. Sinus tachycardia is most commonly a physiologic response to increased demands in the body as in the case of fever, dehydration, infection, anemia, and pulmonary embolism. Medication side effects or withdrawal and recreational drug use are also common causes. Sinus tachycardia is rarely a presenting chief complaint as it is usually asymptomatic, however, patients may report palpitations. Diagnosis is suspected based on a regular, rapid pulse on exam and can be confirmed on an electrocardiogram. Treatment of sinus tachycardia is generally aimed at treating the underlying condition. Atrial fibrillation (A) is a common arrhythmia consisting of rapid and irregular atrial contractions that originate outside of the sinoatrial node. Patients may be asymptomatic, however, they often report palpitations, chest pain, fatigue, dizziness, or shortness of breath. Electrocardiogram reveals an irregularly irregular rhythm with narrow QRS complexes and no distinct p-waves. The ventricular rate may vary. However, if the rate is greater than 100, it is termed atrial fibrillation with rapid ventricular response. Atrial flutter (B) is an atrial tachycardia that is distinguished as having regular atrial contractions that originate outside of the sinoatrial node. This is demonstrated by the absence of normal p-waves and the presence of atrial flutter waves, or a "sawtooth" pattern, on electrocardiogram. Ventricular rate can vary, however, it is commonly around 150 beats per minute. Atrial flutter has a similar presentation to atrial fibrillation, including palpitations, dizziness, chest pain, and dyspnea. Ventricular tachycardia (D) is an arrhythmia that originates from abnormal electrical impulses within the ventricles. Electrocardiogram reveals a wide QRS complex with no p-waves and a rate typically greater than 120 beats per minute. The QRS complexes may be similar (monomorphic) to each other or vary (polymorphic). Ventricular tachycardia is a life-threatening rhythm resulting in poor cardiac output which may progress to death. Emergent treatment is required which may consist of medical therapy or electrical cardioversion, depending on the hemodynamic stability of the patient.

A 35-year-old man presents to the emergency department with low-grade fever, flu-like symptoms, petechiae, and a new-onset murmur. He does not have a history of intravenous drug use or previous heart valve replacement. He is diagnosed with bacterial endocarditis. Which of the following organisms is the most likely the cause of his condition? A) Candida albicans B) Methicillin-resistant Staphylococcus aureus C) Pseudomonas aeruginosa D) Streptococcus viridans

Explanation: Infective endocarditis is an infection of one or more heart valves that is almost always fatal if left untreated. Most native valve infections are caused by Streptococcus species (S. viridans). Patients who have prosthetic valve replacements or are intravenous drug users have different bacteriology, with Staphylococcus epidermidis being the most common organism for the former group and Methicillin-resistant Staphylococcus aureus being more common in the latter. Symptoms of subacute endocarditis can be subtle and nonspecific, including low-grade fever, anorexia, weight loss, influenza-like syndromes, and pleuritic pain. Patients may also present with the following classic findings: petechiae, splinter hemorrhages on proximal nail bed, Osler nodes (painful, red, raised lesions found on the hands and feet), Janeway lesions (non-tender, erythematous macular or nodular lesions found on the palms and soles of feet), and Roth spots (retinal hemorrhages with pale centers). Signs of neurologic involvement include embolic stroke with focal neurologic deficits caused by vegetations breaking off from the valves (most common neurologic presentation), intracerebral hemorrhage, and multiple microabscesses. The Duke diagnostic criteria are used to examine both blood cultures and echocardiographic findings to make a definitive diagnosis of infective endocarditis. In order for the diagnosis of endocarditis to be made, there must be at least two major Duke criteria present, or one major criterion with three minor criteria, or five minor criteria present. Major criteria include: (1) two separate positive blood cultures with microorganisms typical for infective endocarditis and (2) evidence of valvular lesions on echocardiogram or clearly established new valvular regurgitation. Minor criteria include: (1) predisposing heart condition or history of intravenous drug use, (2) fever (3) vascular phenomena (e.g., arterial emboli, intracranial hemorrhage, Janeway lesions, etc.), (4) immunological phenomena (e.g. Osler nodes, glomerulonephritis, etc.), and (5) positive blood culture but not meeting major criterion or serologic evidence of active infection with organism consistent with infective endocarditis. Transesophageal echocardiogram is the diagnostic test of choice for identifying valvular vegetations in patients who are at risk since it is the most sensitive test, although diagnosis cannot be excluded based on negative echocardiogram findings. Antibiotics are the mainstay of treatment and should empirically provide coverage against Streptococci and methicillin-resistant Staphylococcus aureus. Prophylactic antibiotics for dental procedures, biopsy/excision of respiratory mucosa, procedure involving infected musculoskeletal tissue/skin are recommended for patients who have a prosthetic heart valve, history of endocarditis, a heart transplant with abnormal heart valve function, and certain congenital heart defects (e.g. cyanotic congenital heart disease).

Which of the following is a risk factor for infective endocarditis? A) Age less than 60 years old B) Female sex C) Having a prosthetic heart valve D) Oral illicit drug use

Explanation: Infective endocarditis refers to infection of the endocardial surface of the heart. It typically involves infection of one or more heart valves or infection of an intracardiac device. Cardiac risk factors include prior infective endocarditis, having a prosthetic heart valve or intracardiac device, and a history of valvular or congenital heart disease. Noncardiac risk factors include intravenous drug use, indwelling intravenous catheters, immunosuppression, and recent dental or surgical procedures. When infective endocarditis occurs in intravenous drug users, it most commonly affects the valves on the right side of the heart. In contrast, infective endocarditis most commonly affects the valves on the left side of the heart in patients who do not use intravenous drugs. Since the clinical manifestations of infective endocarditis are nonspecific, assessing these risk factors is important in deciding when to consider infective endocarditis. Fever is the most common symptom of infective endocarditis. Other common symptoms are also nonspecific and include chills, malaise, weight loss, night sweats, myalgias, arthralgias, and headache. Most patients with infective endocarditis have cardiac murmurs on auscultation. Infective endocarditis most commonly presents acutely in developed countries, but some patients have subacute or chronic presentations, such as a low-grade fever lasting for days. Splinter hemorrhages, Roth spots, Osler nodes, and Janeway lesions are less common but more specific findings. Splinter hemorrhages are linear, reddish-brown lesions seen on the nail bed. Roth spots are hemorrhagic lesions on the retina with pale centers. Osler nodes are tender subcutaneous nodules that most commonly occur on the pads of the fingers or toes. Janeway lesions are non tender erythematous macules that most commonly occur on the palms or soles. The diagnosis of infective endocarditis is made by assessing risk factors, clinical findings, blood cultures, and echocardiography. A chest X-ray and an electrocardiogram (ECG) should also be obtained. The management of infective endocarditis includes intravenous antibiotics and surgical consultation in some patients. Blood cultures must be obtained prior to starting antibiotics and should ideally be obtained from at least two venipuncture sites. If patients with infective endocarditis are stable, then it is appropriate to wait for blood culture results to begin antibiotics. In patients with unstable presentations, vancomycin is typically used for empiric antibiotic therapy. In either stable or unstable patients, the antibiotic regimen should be targeted to the culture results once the culture results are received. The typical duration of antibiotic therapy is four to six weeks. Patients with complications, such as multidrug-resistant organisms or heart failure symptoms, require early surgical consultation for possible valve repair or replacement. Antimicrobial prophylaxis is recommended for patients at high risk for infective endocarditis who are undergoing procedures likely to result in bacteremia with a microorganism that has the potential to cause bacterial endocarditis. Examples of conditions that increase the risk of infective endocarditis include prosthetic heart valves, prior history of infective endocarditis, and prosthetic material used for cardiac valve repair. The highest risk procedures include dental procedures that involve manipulation of gingival tissue or the periapical region of teeth or perforation of the oral mucosa. This includes routine dental cleaning. Respiratory tract procedures and procedures involving skin and musculoskeletal tissue are also considered high risk. The recommended antimicrobial prophylactic regimen for individuals with cardiac risk factors undergoing high-risk procedures is 2 grams of amoxicillin administered 30-60 minutes prior to the procedure.

Which of the following is the best antimicrobial choice for a hemodynamically stable patient with native valve infective endocarditis caused by viridans group streptococcus? A) Aqueous penicillin G and gentamicin B) Nafcillin C) Vancomycin D) Vancomycin and gentamicin

Explanation: Infective endocarditis refers to infection of the endocardial surface of the heart. It typically involves infection of one or more heart valves or infection of an intracardiac device. Cardiac risk factors include prior infective endocarditis, having a prosthetic valve or intracardiac device, and a history of valvular damage. Noncardiac risk factors include intravenous drug use, having an indwelling intravenous catheter, immunosuppression, and recent dental or surgical procedures. Infective endocarditis in intravenous drug abusers is more likely to affect the valves on the right side of the heart, whereas the left side of the heart is more likely to be affected in patients who do not use intravenous drugs. Infective endocarditis has variable clinical manifestations and the onset can be acute, subacute, or chronic. Fever is the most common symptom, but other nonspecific systemic symptoms include chills, weight loss, malaise, myalgias, arthralgias, and headache. Cardiac murmurs are present in most patients. Less common but more specific findings include splinter hemorrhages, Roth spots (hemorrhagic lesions on the retina with pale centers), Osler nodes (tender subcutaneous nodules on the pads of the fingers and toes), and Janeway lesions (nontender erythematous macules on the palms and soles). The diagnosis is confirmed using the modified Duke criteria, which considers clinical manifestations, blood cultures, and echocardiography. The modified Duke criteria consider major criteria and minor criteria to estimate the odds of a patient having infective endocarditis. The major criteria include positive blood cultures for organisms likely to cause infective endocarditis (e.g., Staphylococcus aureus, Streptococcus viridans, and HACEK organisms) and evidence of endocardial involvement on echocardiogram. The minor criteria include the presence of risk factors (e.g., intravenous drug use or prosthetic heart valve), fever, vascular phenomena (e.g., Janeway lesions), immunologic phenomena (e.g., Osler nodes, Roth spots), and microbiologic evidence that does not meet the major criteria. For the diagnosis of infective endocarditis to be made, one of the following must be present: two major criteria, one major and three minor criteria, or five minor criteria. Blood cultures from multiple venipuncture sites should be obtained prior to initiating antibiotic therapy. The management of infective endocarditis consists of intravenous antibiotics and, in certain cases, surgical referral for valve repair or replacement. Empiric antibiotics are used in hemodynamically unstable patients who present with acute infective endocarditis. However, once a patients blood culture results are back, the antibiotic therapy should be targeted to the causative organism. The most common causative organisms of infective endocarditis are Staphylococcus aureus, viridans group streptococci, Streptococcus bovis, and Enterococci. Staphylococcus aureus is a particularly common causative organism in intravenous drug users. Methicillin-sensitive Staphylococcus aureus infective endocarditis can be treated with nafcillin or oxacillin. Methicillin-resistant Staphylococcus aureus infective endocarditis should be treated with vancomycin. Viridans group streptococci infective endocarditis should be treated with aqueous penicillin G and gentamicin. In addition to intravenous antibiotics, some patients with infective endocarditis require early surgical referral. Indications for early surgical referral include heart failure symptoms, heart block, paravalvular extension of the infection (e.g., annular abscess), infection caused by difficult to treat pathogens, and persistent bacteremia that lasts more than seven days. Patients with early surgical referral may or may not have surgery because of patient-specific factors causing high operative risk. Nafcillin (B) is a beta-lactamase-resistant penicillin that can be used against methicillin-susceptible Staphylococcus aureus. Nafcillin is not used to treat infective endocarditis caused by Streptococcus species. Vancomycin (C) is the antibiotic of choice for infective endocarditis caused by methicillin-resistant Staphylococcus aureus and is the antibiotic used for empiric therapy in patients who are hemodynamically unstable. Penicillin G and gentamicin are used for viridans group streptococcus species. Vancomycin and gentamicin (D) is incorrect because penicillin are first-line treatment for native valve infective endocarditis due to viridans group Streptococcus. Vancomycin is considered in patients who have penicillin allergies.

A 40-year-old woman presents to the clinic with palpitations. On exam, auscultation reveals a holosystolic murmur heard best at the apex. Which of the following is the most likely diagnosis? A) Aortic stenosis B) Mitral insufficiency C) Patent ductus arteriosus D) Tricuspid regurgitation

Explanation: Mitral insufficiency occurs when the mitral valve does not close properly, leading to reverse blood flow from the left ventricle into the left atrium during systole. This results in increased blood volume within the left atrium and ultimately in the left ventricle. Complications include left ventricular enlargement, heart failure, and arrhythmias (atrial fibrillation). If patients are symptomatic, presentation most commonly includes dyspnea on exertion, and decreased exercise tolerance. As the disease progresses, patients may have pulmonary edema and dyspnea at rest. If atrial fibrillation is present, palpitations may also be present. Physical exam includes a bounding peripheral pulse and laterally displaced point of maximal impulse. Crackles may also be present. Cardiac auscultation reveals a holosystolic murmur that is heard best at the apex, which radiates to the back or clavicular area. Diagnosis is made with an echocardiogram study. Definitive treatment options include surgical repair or replacement of the mitral valve. The decision to proceed with surgical intervention is based on the severity of regurgitation, symptoms, and the left ventricular ejection fraction. Aortic stenosis (A) is a narrowing of the aortic valve that results in classic symptoms of chest pain, syncope, and heart failure. The heart murmur associated with aortic stenosis is characterized as a crescendo-decrescendo, systolic murmur which is heard best over the right upper sternal border and radiates to the carotid artery. Patent ductus arteriosus (C) is a congenital heart defect characterized by delayed or failed closure of the ductus arteriosus vessel that is present in-utero. The result is a left-to-right shunt that may result in poor feeding, failure to thrive, fatigue, and respiratory distress. Cardiac auscultation reveals a continuous, machine-like murmur heard best over the left upper sternal border. Tricuspid regurgitation (D) results in backward blood flow from the right ventricle to the right atrium during systole. Patients are often asymptomatic, however, they may complain of pulsatile sensation in neck, peripheral edema, and ascites. If atrial fibrillation is also present, the patient may complain of palpitations as well. Similar to mitral insufficiency, the murmur associated with tricuspid regurgitation is holosystolic in nature, however, it is located over the right or left mid-sternal border, not the apex.

A 70-year-old man with a recent history of myocardial infarction presents to the emergency department with the complaint of extreme fatigue and shortness of breath. On physical exam, the patient appears pale and diaphoretic. There is jugular venous distention. His blood pressure is 90/45 mm Hg and his O2 saturation is 93% on room air. On auscultation, a holosystolic murmur is detected at the apex. Which of the following is the most likely diagnosis? A) Aortic stenosis B) Mitral regurgitation C) Tricuspid regurgitation D) Ventricular septal defect

Explanation: Mitral regurgitation is one of three holosystolic murmurs. The murmur is heard best at the cardiac apex, which radiates to the back or clavicular area. The murmur itself is caused by insufficient closure of the mitral valve allowing for regurgitant flow. Blood flows from the left ventricle in a retrograde direction through the mitral valve and into pulmonary circulation with resultant pulmonary edema. The condition can be either acute or chronic and may be due to either primary or secondary causes. Primary causes affect the valve directly. Malformation of the mitral valve, degenerative mitral valve conditions such as myxomatous degeneration, infective endocarditis, certain drugs, mitral annular calcifications, and rheumatic carditis are all examples of primary causes. Secondary causes, also known as functional causes, are those that affect a different part of the heart and indirectly cause the valve to not close properly. Dilated cardiomyopathy and hypertrophic cardiomyopathy cause impaired closure of the mitral valves secondary to left ventricle dilatation or impaired left ventricle contractility, respectively. Left ventricular pacing may cause or enhance an existing murmur due to asynchrony caused by the pacing. Coronary heart disease is a major cause of secondary mitral regurgitation. If the papillary muscles are affected by myocardial infarction, closure will be impaired. The murmur can be either chronic, caused by ischemia, or acute, caused by the rupture of papillary muscles. Rupture of the papillary muscles can be a life-threatening condition. The diagnosis is made through echocardiogram or angiogram. Patients may be asymptomatic if the heart is compensating. Mitral valve prolapse can lead to decreased ejection fraction and eventual heart failure leaving patients hemodynamically unstable. Acutely, patients may become hypotensive and pulmonary edema and cardiogenic shock may be present. Symptoms may include dyspnea on exertion, orthopnea, or palpitations. Atrial fibrillation is a common finding as well. Surgical correction can be done either to repair or replace the valve should there be progression of the condition. Patients should be made aware of their condition and continued follow-up to evaluate for progression is recommended. Aortic stenosis (A) is a harsh crescendo-decrescendo midsystolic murmur, which radiates to carotid arteries. A bicuspid aortic valve or obstruction can cause this musical murmur, which is seen in older patients with calcification of the leaflets. Tricuspid regurgitation (C) is a blowing holosystolic murmur best heard at the right or left mid sternal border or at the subxiphoid area and increases with inspiration. It is associated with infective endocarditis in the case of intravenous drug abuse. Ventricular septal defect (D) is a harsh, blowing holosystolic murmur that may be associated with a thrill. As the condition progresses, the pressures in the ventricles equalize, known as Eisenmenger complex. At this stage, there will be no audible murmur.

A 34-year-old G1P0 at 26 weeks gestation presents to the office with worsening dyspnea with activity. She has a history of rheumatic heart disease as a child and has been monitored for mild mitral stenosis since the age of 12. Which of the following physical exam findings is most likely to be present? A) A blowing, high-pitched harsh systolic murmur B) A harsh, crescendo-decrescendo systolic murmur C) A high-pitched decrescendo diastolic murmur D) A loud S1 heart sound, a normal S2 sound, followed by an opening snap and a low-pitched diastolic rumbling murmur

Explanation: Mitral stenosis has many causes, including history of rheumatic heart disease (most common cause), congenital structural abnormalities, and mitral valve calcification (more common in the elderly). The normal diameter of the mitral valve is around 4-6 cm2 and as the valve becomes more stenosed, the pressure across the mitral valve increases to ensure adequate flow. Left atrial pressure increases, leading to complications such as accumulation of fluid in the lungs, right ventricular dilation, and pulmonary hypertension. Women are more likely than men to develop rheumatic mitral stenosis, and the onset of symptoms occurs between the third and fourth decade of life. In mild to moderate cases of mitral stenosis, patients are asymptomatic early on, but those who are symptomatic present with progressive exertional dyspnea that may be triggered by any event that causes the heart rate to be elevated (e.g., fever, severe anemia, exercise, pregnancy). In severe cases, marked pulmonary hypertension develops, causing symptoms of right heart failure and low cardiac output. Most patients will eventually develop atrial fibrillation. Characteristic physical exam findings is a loud S1, with an opening snap following a normal S2 heart sound (due to the stiff mitral valve). A low-pitched diastolic rumbling murmur will be present, best heard with the bell of the stethoscope at the apex when the patient is in the left lateral decubitus position. Patients with severe disease may also present with a malar flush (pinkish-purplish patches on the cheeks) and jugular venous distension due to pulmonary hypertension and right heart failure. Echocardiography is the tool of choice to diagnose and assess the severity of the disease. Percutaneous balloon valvuloplasty is preferred over surgical valve replacement in patients with symptoms or evidence of pulmonary hypertension or pulmonary edema. No intervention is required in asymptomatic patients.

Which of the following accurately describes mitral valve prolapse? A) Mitral valve prolapse is a rare cause of mitral regurgitation B) Mitral valve prolapse is usually caused by Marfan syndrome C) Most patients with mitral valve prolapse have minimal or no mitral regurgitation D) Most patients with mitral valve prolapse have severe mitral regurgitation that will eventually require surgical management

Explanation: Mitral valve prolapse (MVP) occurs when the leaflets of the mitral valve prolapse into the left atrium during ventricular contraction. It is the main cause of severe organic mitral regurgitation (backwards flow of blood from the left ventricle to the left atrium) in the Western world. Mitral valve prolapse can be classified according to the etiology as primary or secondary. Primary mitral valve prolapse is typically sporadic (not familial) and refers to degenerative disease in the absence of identifiable connective tissue disease. Secondary mitral valve prolapse is associated with an identifiable disorder, such as Marfan syndrome and Ehler-Danlos syndrome. However, most patients with mitral valve prolapse have mild, trace, or no mitral regurgitation. Symptoms do not reliably suggest mitral valve prolapse. Therefore, the key clinical manifestations are identified with auscultation and are then further evaluated by echocardiography. The most common auscultation finding is a non-ejection systolic click. If mitral regurgitation is present, then a holosystolic heart murmur is another common finding during auscultation. A snapping of the chordae tendineae during systole as the valve prolapses into the atrium is thought to cause the click. An ejection click occurs at the point of maximal opening of the aortic or pulmonic wave, which occurs in early systole (i.e., right after the first heart sound). In contrast, a non-ejection click occurs in the middle or later part of systole. The diagnosis of mitral valve prolapse is confirmed with echocardiography. Most patients with mitral valve prolapse have minimal, trace, or no mitral regurgitation and require no treatment. If complications occur, such as severe mitral regurgitation, ruptured chordae tendineae, or infective endocarditis, then the appropriate treatment for these conditions should be performed. Patients with severe mitral regurgitation may require surgical mitral valve repair or replacement. Mitral valve prolapse is a rare cause of mitral regurgitation (A) is incorrect because mitral valve prolapse is the most common cause of isolated organic mitral valve regurgitation in the Western world. Mitral valve prolapse is usually caused by Marfan syndrome (B) is incorrect because most cases of mitral valve prolapse are sporadic primary cases. Secondary cases due to connective tissue diseases, such as Marfan syndrome, are less common. Most patients with mitral valve prolapse have severe mitral regurgitation that will eventually require surgical management (D) is incorrect because most patients with mitral valve prolapse have minimal, trace, or no mitral regurgitation.

Which of the following medical conditions is most likely to increase the risk of multifocal atrial tachycardia? A) Acute pancreatitis B) Chronic obstructive pulmonary disease C) Hyperlipidemia D) Hypothyroidism

Explanation: Multifocal atrial tachycardia is an irregularly irregular tachydysrhythmia defined by having three distinct p-waves on ECG with a ventricular rate above 100 beats per minute. Wandering atrial pacemaker is a similar dysrhythmia with three distinct p waves but a ventricular rate between 60 and 100. Multifocal atrial tachycardia occurs most commonly in the setting of advanced or decompensated pulmonary or cardiac disease. Common examples of pulmonary diseases provoking multifocal atrial tachycardia are chronic obstructive pulmonary disease and pneumonia. These pulmonary conditions can cause hypoxia, hypercapnia, and acidosis, which are all triggers of ectopic atrial activity. This leads to multiple atrial foci firing and creating three or more p-waves with distinct shapes seen on ECG. The clinical manifestations usually are symptoms of the underlying medical condition and are not related to the tachydysrhythmia itself. Since these symptoms are commonly related to severe pulmonary disease, they may include shortness of breath, wheezing, and productive cough. It is rare for patients to have palpitations, presyncope, or syncope. The diagnosis is suggested by an irregular pulse on exam. It is confirmed by an ECG. The treatment is aimed at the underlying condition. As the pulmonary or cardiac condition improves, multifocal atrial tachycardia typically also improves. Treatment of the tachycardia is rarely necessary because it usually does not cause symptoms. If rate control treatment were used, it would be with a beta-blocker or non-dihydropyridine calcium channel blocker, such as verapamil. Electrolytes, including potassium and magnesium, should be monitored and maintained within reference range. Acute pancreatitis (A) is not known to be a precipitant of multifocal atrial tachycardia. Hyperlipidemia (C) is a risk factor for acute coronary syndrome, but hyperlipidemia itself does not increase the risk of multifocal atrial tachycardia. Hypothyroidism (D) is also not a known precipitant of multifocal atrial tachycardia.

Which of the following is a difference between the treatment of right ventricular myocardial infarctions and the standard treatment of myocardial infarctions? A) Dual oral antiplatelet therapy should not be used in patients with right ventricular myocardial infarctions B) Nitroglycerin use should be avoided in patients with right ventricular myocardial infarctions C) Percutaneous coronary intervention should not be used in patients with right ventricular myocardial infarctions D) Statin therapy should not be used in patients with right ventricular myocardial infarctions

Explanation: Myocardial infarction (MI) is defined as a clinical or pathologic event caused by myocardial ischemia in which there is evidence of myocardial injury or necrosis. Chest pain or discomfort is the most common symptom of acute myocardial infarction. The classic chest pain of a myocardial infarction is described as a substernal tightness or discomfort that radiates to the left arm or jaw and is worse with exertion. The diagnosis is confirmed with electrocardiogram findings and cardiac biomarkers, such as troponin. The immediate management of acute myocardial infarction generally includes relieving ischemic chest pain, antiplatelet therapy, and urgent revascularization. The standard initial medical management of acute myocardial infarction includes oxygen, nitroglycerin, beta-blockers, and aspirin. However, there are some important differences in the management of patients with a myocardial infarction involving the right ventricle (right ventricular myocardial infarction). Concomitant right ventricular myocardial infarctions should be suspected in patients with myocardial infarctions involving the inferior part of the left ventricle, which is more commonly referred to as an inferior myocardial infarction. An inferior wall myocardial infarction is suspected by ST-segment, T wave, and Q wave changes involving leads II, III, and aVF on an electrocardiogram. Reciprocal changes, such as ST-segment depression, may also be seen in leads aVL, I, V5, and V6. About 33% of patients with an inferior wall myocardial infarction also have a right ventricular myocardial infarction. It is rare for patients to have a right ventricular infarction without also having an inferior myocardial infarction. Therefore, all patients with an inferior myocardial infarction should be assessed for the presence of a right ventricular infarction by looking for signs of right heart failure including hypotension, jugular venous distention (distended neck veins), and checking for ST-segment elevation in lead V4R (this is checked by obtaining a right-sided ECG). Right ventricular infarctions decrease the blood pumped from the right ventricle and result in decreased preload and cardiac output from the left ventricle. This decreased cardiac output can manifest as hypotension and cardiogenic shock. Therefore, interventions that could potentially further decrease preload and cardiac output, such as nitroglycerin, diuretics, and beta-blockers, are generally avoided in patients with right ventricular myocardial infarction. In addition, opioid drugs may lower preload and therefore should also be avoided. Intravenous fluids should be administered carefully to increase preload in patients with right ventricular infarctions. Vasopressors, such as dopamine, are the next step in management for patients with severe hypotension despite intravenous fluids. The standard interventions for myocardial infarctions that do not decrease preload, such as dual antiplatelet therapy, anticoagulation, and statin therapy, are still used in right ventricular myocardial infarctions. In addition, urgent revascularization, particularly with percutaneous coronary intervention (PCI), is still the most important step in the management of right ventricular infarctions. Dual oral antiplatelet therapy should not be used in patients with right ventricular myocardial infarctions (A) is incorrect. Since antiplatelet therapy does not further decrease preload, it is recommended in patients with all myocardial infarctions including right ventricular myocardial infarctions. Percutaneous coronary intervention should not be used in patients with right ventricular myocardial infarctions (C) is incorrect because reperfusion is still the most important intervention. Statin therapy should not be used in patients with right ventricular myocardial infarctions (D) is incorrect because statins do not further decrease preload.

What is the laboratory study of choice to confirm acute myocardial infarction? A) Creatinine kinase MB B) D-dimer C) Myoglobin D) Troponin

Explanation: Myocardial infarction is defined as myocardial injury caused by myocardial ischemia. Myocardial ischemia is most commonly secondary to plaque in a coronary artery causing decreased blood flow to myocardial tissue. Coronary heart disease, which leads to myocardial infarction, is common in the United States. Major risk factors include age, hypertension, dyslipidemia, smoking, and family history. Patients having a myocardial infarction generally present with substernal, crushing chest pain, diaphoresis, weakness, fatigue, nausea, vomiting, syncope, sense of impending doom, and shortness-of-breath. Patients may also present with sudden cardiac death secondary to ventricular fibrillation (Vfib). Some patients such as diabetics, elderly and women may present with painless infarcts or more atypical symptoms such as abdominal pain, nausea, or heartburn. To meet diagnostic criteria for acute myocardial infarction, the patient must exhibit elevation of cardiac biomarkers and have typical symptoms such as chest pain, abnormal ECG, or imaging showing loss of wall motion or viable myocardium. Abnormal ECG findings include ST segment elevation, depression, and T-wave abnormalities. The cardiac biomarker of choice to diagnose a myocardial infarction is troponins (Troponin I and T). Cardiac troponin biomarkers are highly sensitive and specific in diagnosing cardiac injury. Contemporary troponin assays can generally diagnose myocardial ischemia within two to three hours of presentation. Patients presenting early after onset of symptoms, however, may still have a negative troponin even with true cardiac ischemia due to a delayed rise. Most patients can be excluded from having a myocardial infarction within six hours of presentation, however, patients with highly suspicious histories should be tested again at 12 hours. ECG, echocardiogram, and cardiac catheterization are also mainstays in the evaluation and treatment of myocardial infarction. Once the patient is diagnosed with myocardial infarction, a cardiologist should be consulted immediately for consideration of revascularization. Patients should be given supplemental oxygen and started on heparin and aspirin to prevent further cardiac injury. Measures must also be taken to relieve the patient's chest pain, such as nitroglycerine, morphine. Complications of acute myocardial infarction include rupture of the left ventricular free wall and interventricular septum, mitral regurgitation, congestive heart failure, bradyarrhythmias, atrioventricular block, pericarditis and pericardial effusion. After myocardial infarction, all patients should be started on aspirin, beta blocker, ACE inhibitor, and statin to reduce mortality and risk of further coronary events. Creatinine kinase MB (CK-MB) (A) is a nonspecific biomarker that is no longer recommended as first-line testing due to the development of highly specific troponin biomarkers. CK-MB can also be released from skeletal muscle. Its use is only recommended in facilities where contemporary troponin assays are not available. D-dimer (B) is a measure of fibrin degradation products and is neither sensitive nor specific in diagnosing myocardial infarction. D-dimer is generally used to aid in the diagnosis of deep vein thrombosis, pulmonary embolism, and disseminated intravascular coagulation. Myoglobin (C) was historically used to diagnose myocardial infarction because of its rapid release from damaged cells. Contemporary troponin assays are elevated before myoglobin so it is generally not recommended as first-line testing unless an insensitive troponin assay is being used.

A 50-year-old man with a history of hypertension and diabetes mellitus presents to the emergency department with a myocardial infarction. Which of the following medications will reduce this patient's mortality after a myocardial infarction? A) Captopril B) Ibuprofen C) Morphine D) Nitrates

Explanation: Myocardial infarction should be considered in patients with risk factors for cardiovascular disease, such as advanced age, hypertension, hyperlipidemia, and diabetes mellitus, presenting with chest discomfort or shortness of breath. The diagnosis is confirmed with electrocardiogram (ECG) findings and cardiac biomarkers, such as troponin. The acute management for patients with acute myocardial infarction consists of antiplatelet therapy (aspirin), relieving ischemic chest pain, and emergency reperfusion. Sublingual nitroglycerin should be administered to patients presenting with ischemic chest pain, and intravenous nitroglycerin can be used in patients with persistent chest pain after three sublingual nitroglycerin tablets. However, nitroglycerin should be avoided in patients with hypotension, in patients with a right ventricular infarction, and in patients who have taken a phosphodiesterase inhibitor, such as vardenafil, within 24 hours. Morphine should be avoided in patients with acute myocardial infarction, except in cases of intolerable pain, because studies have shown morphine use is associated with adverse outcomes. Cardioselective beta-blockers, such as metoprolol or atenolol, should be started within 24 hours of a myocardial infarction and continued indefinitely in all patients who have had a myocardial infarction and do not have any contraindications. Contraindications to beta-blockers include heart failure, evidence of a low cardiac output state, high risk for cardiogenic shock, bradycardia, heart block, or reactive airway disease. High-intensity statin therapy should also be initiated as early as possible following myocardial infarction. Examples of high-intensity statin medication and dosages are atorvastatin dosed at 80 milligrams and rosuvastatin dosed between 20-40 milligrams daily. Antiplatelet therapy, including aspirin, is recommended indefinitely in patients following myocardial infarction without any contraindications. In many patients, dual antiplatelet therapy, such as aspirin and clopidogrel, is recommended in patients following myocardial infarction. The P2Y12 receptor inhibitor (clopidogrel, prasugrel, or ticagrelor) should be continued at least one year following acute myocardial infarction. Aspirin is the pharmacologic intervention with the greatest efficacy in reducing mortality following an acute myocardial infarction. An angiotensin-converting enzyme inhibitor, such as captopril, is indicated following myocardial infarction in patients with hypertension, diabetes mellitus, or heart failure with a left ventricular ejection fraction less than 40%. Anticoagulant therapy is also recommended immediately following a myocardial infarction regardless of the reperfusion approach. The evidence regarding which anticoagulant to use is less established, but unfractionated heparin, fondaparinux, and enoxaparin are common choices. Supplemental oxygen may also be used in patients presenting with acute myocardial infarction. Indications for supplemental oxygen use include arterial hemoglobin saturation less than 90% and respiratory distress. Aspirin, beta-blockers, and angiotensin-converting enzyme (ACE) inhibitors are pharmacologic interventions that reduce mortality following myocardial infarction. There are similarities and differences in the management of patients with ST elevation myocardial infarction (STEMI) and non-ST elevation myocardial infarction (NSTEMI). Oxygen therapy, nitroglycerin, beta-blockers, aspirin, morphine, and angiotensin-converting enzyme inhibitors are used similarly in both conditions. However, there are some differences in the use of reperfusion therapy. Reperfusion for acute ST-elevation myocardial infarction can be performed with fibrinolytic therapy (alteplase) or percutaneous coronary intervention (PCI). High-quality percutaneous coronary intervention is preferred over fibrinolysis because it enhances survival, reduces the risk of intracerebral hemorrhage, and reduces the risk of recurrent myocardial infarction. The timing of percutaneous coronary intervention is important to have good outcomes. The goal is a first medical contact time to percutaneous coronary intervention time of less than 90 minutes for patients who present to facilities that perform percutaneous coronary intervention. Patients who present to a facility that does not have capabilities to perform percutaneous coronary intervention have a goal time of 120 minutes from first medical contact to percutaneous coronary intervention (at the facility the patient was transferred to). Patients who arrive to a medical facility with acute ST-elevation myocardial infarction within 12 hours of symptom onset who cannot receive percutaneous coronary intervention within 120 minutes should be treated with fibrinolytic therapy, such as alteplase. Fibrinolytic therapy has not shown benefit in patients with acute ST-elevation myocardial infarction who present more than 12 hours after the onset of symptoms. Reperfusion therapy is sometimes used in patients with non-ST elevation myocardial infarction who are considered to be at high risk for an adverse cardiovascular outcome. Indications for reperfusion therapy with non-ST elevation myocardial infarction include hemodynamic instability or cardiogenic shock, severe left ventricular dysfunction or heart failure, recurrent or persistent angina despite intensive medical therapy, sustained ventricular tachycardia, or new or worsening mitral valve regurgitation. Furthermore, patients with a non-ST elevation myocardial infarction and high TIMI score may receive percutaneous coronary intervention. Fibrinolytic therapy is not used in patients with non-ST elevation myocardial infarction. Ibuprofen (B) is a nonsteroidal anti-inflammatory drug (NSAID). Nonsteroidal anti-inflammatory drugs should be discontinued immediately, with the exception of aspirin, in patients with an acute myocardial infarction, because they are associated with an increased risk of cardiovascular events. Morphine (C) may be used to control unacceptable levels of pain in patients with an acute myocardial infarction. However, it is preferred to avoid morphine use, because morphine is associated with worse outcomes. Nitrates (D) are used in the management of acute myocardial infarction to relieve ischemic chest pain, but they are not associated with a reduction in mortality.

A 45-year-old man has been managed for heart failure for the last two months. He has undergone a series of tests to identify the cause of his heart failure. Most recently, an endomyocardial biopsy has come back positive for parvovirus 19, confirming viral myocarditis. Which of the following problems associated with viral myocarditis is most likely causing his symptoms of heart failure? A) Acute rheumatic fever B) Dilated cardiomyopathy C) Giant cell myocarditis D) Vaccine-mediated myopericarditis

Explanation: Myocarditis is an inflammation of cardiac muscle most commonly associated with viral infection (Coxsackie B), though it can also have other infectious and noninfectious etiologies. Not all patients are symptomatic, as symptoms depend on the degree of myocardium involvement. When symptomatic, patients with myocarditis typically present with symptoms of heart failure, although symptoms concerning for myocardial infarction or arrhythmia may also be noted. Patients with myocarditis caused by a viral infection are at increased risk for dilated cardiomyopathy. The path to developing dilated cardiomyopathy is understood in three stages. First, viral infection causes myocyte death which exposes myocyte proteins to an immune response. Next, the immune response is activated; and finally, the antibody response leads to chronic myocardial inflammation, necrosis, apoptosis, and fibrosis. Evidence of myocarditis may be seen on ECG, chest X-ray (cardiomegaly), and cardiac biomarkers, but definitive diagnosis of myocarditis requires endomyocardial biopsy. Management of myocarditis usually begins with medical management of heart failure and dysrhythmia. Ultimately, mechanical circulatory support or transplantation may be indicated. Acute rheumatic fever (A) is associated with group A streptococcus infection not a viral infection. Giant cell myocarditis (C) is an autoimmune disease that can lead to myocarditis but is not associated with viral infection. Vaccine-mediated myopericarditis (D) is a cause of heart failure that is a known complication of smallpox vaccination specifically. Given that smallpox is now eradicated and smallpox vaccines are no longer given, this is an unlikely complication.

A 62-year-old man presents to the ED with acute onset of fever and sharp, pleuritic chest pain. He notes the pain is improved when he sits up and leans forward. On physical exam, a scratchy, squeaking sound can be appreciated over the left sternal border. Acute pericarditis is suspected, possibly with pericardial effusion. Initial tests are ordered including an ECG, a chest X-ray, blood tests including a troponin level, and echocardiography. Which of these studies will best support diagnosis of a pericardial effusion? A) Chest X-ray B) ECG C) Echocardiogram D) Troponin level

Explanation: Normally, a thin layer of fluid lubricates the space between the pericardium and the heart. When the amount of fluid in this space exceeds what is normally present, a pericardial effusion exists. An effusion may be a result of any pericardial problem but is most commonly noted in cases of acute pericarditis, autoimmune disease, trauma including surgery or myocardial infarction, and malignancy. The physical signs of a developing pericardial effusion are generally nonspecific (muffled heart sounds, soft PMI, dullness at left lung base, pericardial friction rub) and insensitive to trigger a diagnostic workup, but in the context of a cardiac workup or pericardial disease, pericardial effusion should be suspected. Diagnosing an effusion and following its progress is important because the buildup of fluid in this space can lead to an increase in pericardial pressure and ultimately cardiac tamponade compromising cardiac function. All patients with suspected pericardial disease should be evaluated with echocardiography, which has the sensitivity and specificity to establish the diagnosis (can show as little as 20 mL of fluid), in addition to evaluating any associated hemodynamic sequelae. Based on this evaluation, an appropriate management plan can be established. If an effusion is not hemodynamically significant and the patient is asymptomatic, the focus should turn to diagnosis and treatment of the underlying disease accompanied by close monitoring of the effusion (repeat echocardiogram in one to two weeks). For more significant effusions with evidence of cardiac tamponade, drainage of pericardial fluid by pericardiocentesis or open surgical drainage may be indicated. Pericardial fluid analysis may clarify the cause of the effusion. Chest X-ray (A) can support the diagnosis of pericardial effusion when an enlarged cardiac silhouette ("water bottle" appearance) with clear lung fields are appreciated on the study. CXR shows an enlargement of cardiac silhouette when >250 mL of fluid has accumulated. However, this finding has poor specificity for pericardial effusion and is not diagnostic as an isolated finding. Findings frequently noted on ECG (B) when a pericardial effusion is present include sinus tachycardia, low QRS voltages, T-wave flattening, and electrical alternans. The combination of sinus tachycardia and electrical alternans is highly specific to pericardial effusion but only modestly sensitive. Thus, absence of these findings does not rule out the possibility of pericardial effusion, and ECG is inferior to echocardiogram for definitive diagnosis. Increased troponin level (D) is associated with acute pericarditis, but as it is a marker of myocardial injury, its role in the diagnosis of pericardial effusion is primarily to establish the cause of the effusion.

A 22-year-old woman presents with a syncopal episode. She ran a marathon and was sitting down resting after the event. When she went to stand up, she "blacked out." She is starting to feel better now and denies chest pain, abdominal pain, palpitations, or shortness of breath. Her vital signs are within normal limits. Which of the following is the most likely diagnosis? A) Absence seizure B) Atrioventricular nodal reentrant tachycardia C) Heat stroke D) Orthostatic hypotension

Explanation: Orthostatic hypotension is characterized by a marked decrease in blood pressure with positional changes, most commonly standing. The most common causes of orthostatic hypotension are volume depletion and autonomic dysfunction. Medications such as antihypertensives can also cause orthostatic hypotension. Symptoms are secondary to cerebral hypoperfusion and include dizziness, weakness, visual blurring, and in severe cases, syncope, angina, and stroke. The diagnosis is made by documenting a fall in at least 20 mm Hg systolic or 10 mm Hg diastolic blood pressure upon standing for two minutes after having been supine for at least five minutes. Treatment consists of volume repletion and possible medication discontinuation. Pharmacologic treatment may include fludrocortisone, midodrine, and supplementary agents such as erythropoietin, caffeine, pyridostigmine, and NSAIDs. Absence seizures (A) are most common in childhood and cause sudden staring and altered consciousness lasting five to 10 seconds. Atrioventricular nodal reentrant tachycardia (B) is a supraventricular tachycardia that commonly causes dizziness, palpitations, dyspnea, and sometimes syncope but is not generally associated with postural change. Heat stroke (C) can present with syncope and is associated with exertion, however, patients will also have a significantly elevated core body temperature.

Which of the following pathologic mechanisms most commonly causes paroxysmal supraventricular tachycardia? A) Accessory electrical pathway B) Excessive catecholamine release C) Sinoatrial node dysfunction D) Structural heart disease

Explanation: Paroxysmal supraventricular tachycardia is a narrow QRS complex tachycardia that abruptly begins and ends and maintains a regular ventricular response. Reentry via an accessory electrical pathway is the most common mechanism of paroxysmal supraventricular tachycardia and is caused primarily by either atrioventricular nodal reentrant tachycardia or atrioventricular reentrant tachycardia. Atrioventricular nodal reentrant tachycardia is characterized by dual pathways for electrical signal within the atrioventricular node or perinodal atrial tissue. This pathological reentry pathway accounts for the majority of paroxysmal supraventricular tachycardia. The other predominant reentry pathway is atrioventricular reentrant tachycardia, which is comprised of an extranodal pathway connecting the atrial and ventricular chambers of the heart. Finding delta waves on ECG indicate anterograde conduction via the accessory pathway and is diagnosed as Wolff-Parkinson-White syndrome. The prevalence of paroxysmal supraventricular tachycardia is slightly higher in men and increases with age. Patients may be asymptomatic or complain of palpitations, mild chest pain, shortness of breath, diaphoresis, or faintness. The ECG reveals a heart rate between 140 and 280 beats/min with a regular rhythm. Treatment strategies vary depending on the hemodynamic stability of the patient, with hypotension, altered mental status, and signs of hypoperfusion (diaphoresis, pale, clammy skin) indicating hemodynamic instability. Unstable patients with paroxysmal supraventricular tachycardia findings require urgent electrical cardioversion. In stable patients, initial measures include valsalva maneuver and facial contact with cold water (the diving reflex). Carotid sinus massage is an effective technique in increasing vagal tone, thereby abating the rapid heart rate, however, caution should be taken to rule out carotid bruits prior to initiating carotid massage. First-line medication management in narrow complex, stable, tachycardia involves adenosine 6 mg IV bolus followed by 12 mg IV bolus if not successful. In the overall treatment of SVT, once the patient is stable it is important to diagnose and treat any underlying causes, such as dehydration, electrolyte abnormalities, myocardial ischemia, and anemia. Long-term prevention can be accomplished via catheter ablation of accessory pathways or use of atrioventricular nodal blocking agents (e.g., verapamil, diltiazem, and metoprolol). While excessive catecholamine release (B) could contribute to an elevated heart rate, it is not the mechanism by which paroxysmal supraventricular tachycardia occurs. Pheochromocytomas induce autonomic dysfunction via secretion of excessive catecholamines with the characteristic finding being treatment-resistant hypertension. Sinoatrial node dysfunction (C) is not responsible for paroxysmal supraventricular tachycardia. Supraventricular dysrhythmias, such as atrial fibrillation, are caused by dysfunction of the primary pacemaker. Structural heart disease (D) can play a role in a select group of paroxysmal supraventricular tachycardia patients, however, it is not the primary cause of this condition.

What is the best antiplatelet regimen for patients undergoing urgent primary percutaneous coronary intervention? A) Aspirin B) Aspirin and abciximab C) Aspirin and prasugrel D) Clopidogrel

Explanation: Percutaneous coronary intervention (PCI), formerly known as angioplasty with stenting, is the revascularization method of choice for most patients with a myocardial infarction. Percutaneous coronary intervention is the recommended treatment for all patients with acute ST elevation myocardial infarction who have access to it in a timely manner. The management goal for a patient initially arriving to a facility than can perform percutaneous coronary intervention is a first contact to percutaneous coronary intervention time of less than 90 minutes. Reperfusion therapy is sometimes used in patients with non-ST elevation myocardial infarction who are considered to be at high risk for an adverse cardiovascular outcome. Indications for reperfusion therapy with non-ST elevation myocardial infarction include hemodynamic instability or cardiogenic shock, severe left ventricular dysfunction or heart failure, recurrent or persistent angina despite intensive medical therapy, sustained ventricular tachycardia, or new or worsening mitral valve regurgitation. Furthermore, patients with a non-ST elevation myocardial infarction and high TIMI score may receive percutaneous coronary intervention. Percutaneous coronary intervention can also be performed electively for some patients with stable coronary artery disease. Percutaneous coronary intervention uses a catheter to go through the radial or femoral artery to place a stent in a coronary blood vessel that has been narrowed by plaque. Antithrombotic therapy is used during the periprocedural period of coronary arteriography and percutaneous coronary intervention because studies have shown that it improves outcomes. The recommended antithrombotic regimen varies between patients having elective percutaneous coronary intervention and those having urgent percutaneous coronary intervention. Patients with scheduled elective percutaneous coronary intervention for stable coronary artery disease should be taking aspirin daily. In addition, patients get a higher loading dose of aspirin (162-325 milligrams) preoperatively. In addition, these patients are given clopidogrel, which is a P2Y12 receptor inhibitor, and unfractionated heparin, which is an anticoagulant. Heparin is an anticoagulant agent and aspirin and clopidogrel are antiplatelet agents. Unfractionated heparin is continued during the procedure. The duration of clopidogrel usage following stent placement varies depending on the type of stent used, but it is typically used for at least six to 12 months. The same regimen of aspirin, a P2Y12 receptor inhibitor, and unfractionated heparin is used in patients requiring urgent primary percutaneous coronary intervention in the setting of acute coronary syndrome, however, either ticagrelor or prasugrel, rather than clopidogrel, is preferred as the P2Y12 receptor inhibitor agent. In addition to the antithrombotic regimen, beta-blockers should be used in all stable patients undergoing percutaneous coronary intervention. Aspirin (A) monotherapy is incorrect. Studies show that dual antiplatelet therapy improves outcomes compared to aspirin monotherapy. Aspirin and abciximab (B) is incorrect. Abciximab is a glycoprotein IIb/IIIa inhibitor. Glycoprotein IIb/IIIa inhibitors are not routinely used in patients who receive aspirin and a P2Y12 receptor inhibitor. Clopidogrel (D) monotherapy is incorrect because all patients with acute myocardial infarction who do not have any absolute contraindications should be given aspirin.

Which of the following conditions is shown in the rhythm strip above? A) Atrial fibrillation B) Atrial flutter C) Premature atrial complex D) Sinus arrhythmia

Explanation: Premature complexes are the group of dysrhythmias that occur with either supraventricular or ventricular impulses that discharge prior to their expected timing within the cardiac electrical cycle. These complexes can be divided into supraventricular premature beats (e.g., premature atrial complexes) and premature ventricular complexes. Supraventricular premature beats can originate in the atria from ectopic foci or in the atrioventricular node. A variety of mechanisms trigger premature atrial complexes, the most common form of supraventricular premature beats, which may be present in normal hearts as well as those with structural heart disease. Ventricular premature complexes are triggered from ventricular myocardial activity and arise due to reentry, enhanced or abnormal automaticity, and triggered activity, with reentry being the most common mechanism. These complexes are present in more than 80% of apparently healthy individuals studied with 24-hour ambulatory monitoring. Premature atrial complexes may be precipitated by smoking, alcohol, and caffeine. Other concurrent risk factors and comorbidities include acute myocardial infarction, coronary heart disease, mitral valve disease, chronic obstructive pulmonary disease, and conditions such as cardiomyopathy that increase pressure or cause dilatation of the atria. Frequent premature atrial complexes have been associated with increased cardiovascular mortality and may also predict new atrial fibrillation or cardiovascular events. Isolated events of premature atrial complexes have not been associated with sudden cardiac death, however, in combination with ventricular premature complexes, patients' risk for sudden cardiac death is increased. Simple ventricular premature complexes in apparently healthy individuals have revealed increased mortality. These complexes are also associated with a worse prognosis in patients with prior myocardial infarction but not those with heart failure or cardiomyopathy. Premature complexes of either variety are asymptomatic in the majority of patients. If symptomatic, patients complain of "skipped beats", palpitations, lightheadedness, dizziness, and presyncope. Physical exam may reveal an irregular pulse if observed during a premature complex. Chest auscultation may also detect early heart sounds or pauses. An ECG is the definitive diagnostic tool used to identify premature complexes. Ambulatory ECG monitoring (e.g., holter monitor) can aid in the diagnosis of premature complexes as it provides information over 24-48 hours. Premature atrial complexes are indicated on ECG as a P wave that occurs relatively early in the cardiac cycle with a differing morphology and axis from the normal sinus P wave in other cardiac cycles. This early electrical impulse serves to reset the cardiac cycle and the subsequent normal sinus rhythm will resume based on the initiation of the premature atrial complex. Premature ventricular contractions are indicated on ECG by wide QRS complex, bizarre morphology, and a T wave in the opposite direction of the main QRS vector. In contrast to premature atrial contractions, premature ventricular contractions initiate a full compensatory pause, with the P to P interval surrounding the extra beat being twice the normal sinus P to P interval. Treatment is not necessary for asymptomatic patients with infrequent premature complexes and education and reassurance should be provided for these relatively benign conditions. Patients should be advised to avoid suspected or known triggers and may be treated with a beta-blocker (e.g., metoprolol). Calcium channel blockers (e.g., verapamil) are also used as first-line therapy for premature ventricular complexes. Radiofrequency catheter ablation may be necessary for those with premature complexes refractory to antiarrhythmic medications. Atrial fibrillation (A) is caused by ectopic foci, commonly originating from the pulmonary veins, which cause uncoordinated atrial contraction. This dysrhythmia presents on ECG as an irregularly irregular rhythm with indistinguishable P waves, which is unlike the single, early P wave identified on the ECG of premature atrial contractions. Atrial flutter (B) is the result of reentry circuits that cause rapid, regular atrial contractions at a characteristic rate of 300 beats per minute. The ECG findings of this condition differ from premature atrial contractions by the presence of "sawtooth" P wave morphology and rapid rate. Sinus arrhythmia (D) is a normal variation found on ECG as increased rate with inhalation and decreased rate with exhalation. Findings on ECG vary from those of premature atrial contraction by the absence of a reset cardiac cycle following an early P wave and QRS complex.

Which of the following lifestyle modifications is used in the treatment of primary hypertension? A) Dietary potassium restriction B) Dietary salt restriction C) Increasing alcohol intake D) Sedentary lifestyle

Explanation: Primary (essential) hypertension is associated with an increased risk of heart failure, myocardial infarction, ischemic stroke, intracerebral hemorrhage, and chronic kidney disease. Therefore, it is important to lower blood pressure in patients with hypertension. Blood pressure can be lowered with lifestyle modifications and pharmacologic therapy. All patients with elevated blood pressure or hypertension should undergo lifestyle modifications. Lifestyle modifications include dietary salt restriction, potassium supplementation (preferably by dietary modification), weight loss, Dietary Approach to Stopping Hypertension (DASH) diet, exercise, and limited alcohol intake. Less than 2.3 grams of sodium per day is a reasonable goal for dietary salt restriction. Potassium supplementation is associated with a reduction in blood pressure, particularly in African Americans. Weight loss in overweight or obese individuals can significantly lower blood pressure. The Dietary Approach to Stopping Hypertension (DASH) is a diet that is high in vegetables, fruits, low-fat dairy products, whole grains, poultry, fish, and nuts. Aerobic exercise and resistance training are associated with lowering systolic and diastolic blood pressure. Most studies that have demonstrated a reduction in blood pressure due to aerobic exercise were done with patients performing moderate-intensity exercise for 40 minutes three to four times per week. Alcohol intake should be limited in patients with hypertension to no more than two alcoholic drinks per day in men, and no more than one alcoholic drink per day in women. Dietary potassium restriction (A) would not be recommended in patients with primary hypertension. Potassium supplementation, preferably via increased potassium in the diet, is associated with lowering blood pressure. Increasing alcohol intake (C) is contraindicated in patients with primary hypertension. Alcohol intake should be limited to no more than two alcoholic drinks daily in men, and no more than one alcoholic drink daily in women with hypertension. Sedentary lifestyle (D) is incorrect because primary hypertension is improved with physical activity. Sedentary lifestyle is a risk factor for primary hypertension.

A 53-year-old man with a history of hyperlipidemia presents to clinic for his annual checkup. He states he is feeling well, but his blood pressure is noted to be 146/92 mm Hg. He says that he checks his blood pressure at home and it is consistently in the 140s/90s mm Hg. The accuracy of the device he uses to measure his blood pressure at home was verified at today's visit. Which of the following is the most likely cause of his hypertension? A) Obstructive sleep apnea B) Primary hypertension C) Primary renal disease D) Renal artery stenosis

Explanation: Primary (essential) hypertension is hypertension in which the underlying cause cannot be identified. More than 90% of cases of hypertension are primary hypertension. Therefore, in this vignette, primary hypertension is the most likely answer due to its higher prevalence and because there are not any obvious indicators of secondary hypertension. Findings that suggest secondary causes of hypertension include new onset at an especially young or old age, abrupt onset with previously normal blood pressure, drug-resistant hypertension, or clinical clues for specific secondary causes (e.g., low potassium might suggest primary aldosteronism). The definition of hypertension according to the American Heart Association as of 2017 is blood pressure ≥ 130/≥ 80 mm Hg. Hypertension is ideally diagnosed with multiple out-of-office measurements taken over several weeks using proper technique with a device that has been validated in-office to measure blood pressure. There are two exceptions in which hypertension can be diagnosed with a single measurement. These exceptions are patients with blood pressure ≥ 180/≥ 120 mm Hg and patients with blood pressure ≥ 160/≥ 100 mm Hg who also have target end-organ damage (e.g., left ventricular hypertrophy, hypertensive retinopathy). Primary hypertension is treated with lifestyle modification and pharmacologic treatment. Common pharmacologic classes used include thiazide diuretics, long-acting calcium channel blockers, angiotensin converting enzyme (ACE) inhibitors, and angiotensin II receptor blockers (ARBs). ACE inhibitors and ARBs should not be used together. Obstructive sleep apnea (A) is not the most likely cause of the hypertension given the information provided in the vignette. Obstructive sleep apnea is more common in overweight or obese men. It can cause hypertension that is resistant to treatment. Common clinical findings include loud snoring, witnessed apneic episodes during the night, and daytime somnolence. Primary renal disease (C) can cause hypertension but is a much less common cause and was not indicated by any findings in the vignette. Primary renal disease can cause hypertension, edema, and abnormal findings on urinalysis, such as hematuria or proteinuria. Elevated creatinine would be a common laboratory finding. Renal artery stenosis (D) is another possible cause for this patient, especially given the hyperlipidemia, but is much less likely than primary hypertension. Renal artery stenosis is the most common cause of secondary hypertension and is often due to fibromuscular dysplasia in younger patients and to atherosclerosis in older patients. Abdominal bruits may be heard on physical exam during auscultation. In summary, primary hypertension is by far the most likely cause with the given information. Secondary causes could be suspected if this patient had clinical clues or laboratory findings suggesting a secondary etiology or if the patient is later resistant to treatment.

An African-American man with no other medical conditions is being started on pharmacologic therapy for primary hypertension. Which of the following is the most appropriate initial antihypertensive medication for this patient? A) Amlodipine B) Lisinopril C) Losartan D) Metoprolol

Explanation: Primary (essential) hypertension is hypertension in which the underlying cause cannot be identified. Primary hypertension makes up more than 90% of cases of hypertension. The pathogenesis is poorly understood, but there are genetic and environmental components. Risk factors include advanced age, obesity or weight gain, family history, African American race, high sodium diet, physical inactivity, and excessive alcohol consumption. Patients with primary hypertension are most commonly asymptomatic. However, over time, hypertension significantly increases the risk of heart failure, myocardial infarction, and cerebrovascular accidents. The definition of hypertension according to the American Heart Association and American College of Cardiology in 2017 is blood pressure ≥ 130/≥ 80 mm Hg. Primary hypertension is treated with lifestyle modifications and pharmacologic treatment. Lifestyle modifications should be encouraged in all patients with primary hypertension and include dietary salt restriction, weight loss, dietary approach to stop hypertension (DASH) diet, exercise, and limiting alcohol intake. Pharmacologic treatment is recommended for all patients with out-of-office daytime blood pressure ≥ 135 mm Hg systolic or ≥ 85 mm Hg diastolic. The threshold to use pharmacologic therapy drops to ≥ 130/≥ 80 mm Hg for patients with established clinical cardiovascular disease, diabetes, chronic kidney disease, and adults over age 65. Initial pharmacologic therapy should be chosen from the following four classes of medication: thiazide diuretics, long-acting calcium channel blockers, angiotensin converting enzyme (ACE) inhibitors, and angiotensin II receptor blockers (ARBs). Thiazide or long-acting calcium channel blockers are preferred in African American patients, while an ACE inhibitor or ARB should be used in patients with diabetic nephropathy or nondiabetic chronic kidney disease. Patients whose blood pressure remains uncontrolled despite antihypertensive monotherapy can use a combination of these four classes of medications, but ACE inhibitors and ARBs should not be used together. Amlodipine is a long-acting calcium channel blocker and therefore is the most appropriate of the choices above for the initial treatment of an African American patient without other medical comorbidities. If there are no specific indications for a particular medication based upon comorbidities, most guidelines and recommendations, including the 2017 ACC/AHA guidelines, recommend that initial therapy be chosen from among the following four classes of medications: thiazide diuretics, long-acting calcium channel blockers (most often a dihydropyridine such as amlodipine), angiotensin-converting enzyme (ACE) inhibitors, angiotensin II receptor blockers (ARBs). Additional considerations in choice of initial therapy include a thiazide-like diuretic or long-acting dihydropyridine calcium channel blocker should be used as initial monotherapy in African-American patients. An ACE inhibitor or ARB should be used for initial monotherapy in patients who have diabetic nephropathy or nondiabetic chronic kidney disease complicated by proteinuria. Beta-blockers are no longer recommended as initial monotherapy in the absence of a specific (compelling) indication for their use, such as ischemic heart disease or heart failure with decreased ejection fraction. Lisinopril (B) is an ACE inhibitor. African American patients have a smaller reduction than white patients in response to ACE inhibitors used as monotherapy for hypertension. Studies have shown African Americans have more blood pressure reduction when long-acting calcium channel blockers or thiazide diuretics are used as initial monotherapy. Losartan (C) is an angiotensin II receptor blocker and is not the preferred initial agent for similar reasons to lisinopril. Metoprolol (D) is a beta-blocker. Beta-blockers are not used as the initial pharmacologic agent for any patients with primary hypertension unless the patient also has a comorbid condition that requires a beta-blocker, such as essential tremors or stable angina.

Which of the following is the best initial screening test to diagnose primary aldosteronism? A) Adrenal CT scan B) Aldosterone suppression test C) Aldosterone to renin ratio D) Serum sodium

Explanation: Primary aldosteronism is a condition defined by nonsuppressible hypersecretion of aldosterone. This is an underdiagnosed cause of secondary hypertension. Aldosterone-producing adenomas found in the adrenal glands and bilateral adrenal hyperplasia are the two main subtypes of primary aldosteronism. Aldosterone physiologically causes reabsorption of sodium and secretion of potassium. The retained sodium increases plasma volume and therefore also increases blood pressure. As a result, patients with primary aldosteronism have hypertension mostly due to expanded plasma volume and some patients have hypokalemia. These are the two most common clinical findings. Primary aldosteronism should be suspected in patients with hypertension and hypokalemia, severe or drug-resistant hypertension, hypertension with an adrenal incidentaloma, and hypertension in a patient with a first-degree relative with primary aldosteronism. The initial screening test involves comparing the ratio of plasma aldosterone concentration to plasma renin activity. The aldosterone is elevated and renin is usually suppressed. If this ratio is above 20, then confirmatory testing should be done. The confirmatory test is an aldosterone suppression test, which can be done by administering oral sodium and then measuring the urine aldosterone excretion. Once the diagnosis is confirmed, an adrenal CT is done to determine the subtype and exclude adrenal carcinoma. The treatment options include surgery or medical management. Surgical adrenalectomy is recommended for surgical candidates with unilateral disease (e.g., adrenal adenoma or adrenal carcinoma). Medical management is done for patients with bilateral adrenal hyperplasia. This involves mineralocorticoid receptor antagonists such as spironolactone or eplerenone. Adrenal CT scan (A) is part of the workup, but is not done until the diagnosis of primary aldosteronism is confirmed. The adrenal CT is used to differentiate between the subtypes and assess the risk of adrenal cancer. Aldosterone suppression test (B) is the test used to confirm the diagnosis of primary aldosteronism. Many endocrine conditions have initial screening tests that are more sensitive followed by confirming tests that are more specific (i.e., less false positives). Serum sodium (D) may be slightly elevated in primary aldosteronism but does not play a role in the diagnosis.

A 43-year-old unresponsive man is brought to the emergency department via ambulance. His vitals reveal BP 50/30 mm Hg, RR 8, and HR 63 beats per minute via ECG tracing, shown above. A pulse is not palpable on physical exam. What is the most appropriate treatment for this patient? A) Amiodarone B) Atropine C) Cardiopulmonary resuscitation D) Electrical cardioversion

Explanation: Pulseless electrical activity is defined as the presence of organized electrical activity as demonstrated on ECG with insufficient or absent mechanical contraction. This dysrhythmia fails to perfuse the myocardium and vital organs. The incidence of pulseless electrical activity has increased in the past two decades up to approximately 35-40% of cardiac arrest events. Significant hypoxia, profound acidosis, severe hypovolemia, tension pneumothorax, electrolyte imbalance, drug overdose, sepsis, large myocardial infarction, massive pulmonary embolism, cardiac tamponade, hypoglycemia, hypothermia, and trauma are primary causes of pulseless electrical activity. This condition is also the most common rhythm seen with a pulmonary embolism. Patients may present initially with signs and symptoms pertaining to the underlying etiology that causes their eventual deterioration into pulseless electrical activity. Often, patients present in cardiac arrest and are unresponsive. These patients necessitate immediate activation of advanced cardiac life support, an algorithm and guideline created jointly by the European Resuscitation Council and the American Heart Association. Primary survey includes assessment of airway, breathing, and circulation. Patients with pulseless electrical activity will have an absent pulse on physical examination and require immediate initiation of cardiopulmonary resuscitation with excellent quality chest compressions and ventilation. Cardiac monitoring using ECG is used to both initially diagnose this dysrhythmia and determine changes in rhythm. True pulseless electrical activity is characteristically defined on ECG by abnormal automaticity, which is demonstrated as a slow ventricular rate with wide QRS complex. Rapid, narrow-complex pulseless electrical activity results from tissue hypoxia and can be indicative of an etiology, such as hypovolemia, or obstructions to circulation, such as cardiac tamponade, pulmonary embolism, or tension pneumothorax. This variation has been termed pseudo-pulseless electrical activity and has been associated with better outcomes in successful resuscitation. Pulseless electrical activity is a nonshockable rhythm, and thus, electrical cardioversion is not recommended to treat this condition. Successful treatment of pulseless electrical activity depends on careful evaluation and appropriate treatment of underlying etiology along with excellent chest compressions and ventilation during cardiopulmonary resuscitation. Epinephrine 1 mg IV push should be administered as soon as possible and every three to five minutes thereafter in conjunction with ongoing cardiopulmonary resuscitation. The survival to discharge rate for patients who present to the hospital with pulseless electrical activity ranges from 2-5%. If this dysrhythmia occurs after electrical cardioversion in the treatment of ventricular fibrillation, the prognosis worsens to 0-2% of patients surviving to discharge. Amiodarone (A) is an antiarrhythmic medication used primarily in the treatment of atrial fibrillation. This medication can also be used according to advanced cardiac life support guidelines alternating with epinephrine and between electrical cardioversion shocks in shockable rhythms, such as ventricular fibrillation or ventricular tachycardia. Atropine (B) is part of the advanced cardiac life support guidelines as treatment for symptomatic bradycardic rhythms. While the rhythm strip displays a bradycardic rhythm, the absence of a pulse makes this dysrhythmia pulseless electrical activity, which does not respond to atropine. Electrical cardioversion (D) is not appropriate for treating pulseless electrical activity as it is a nonshockable rhythm. Shockable cardiac arrest rhythms include ventricular fibrillation and pulseless ventricular tachycardia.

On physical exam, which of the following would you expect to find with a diagnosis of hypertrophic cardiomyopathy? A) Austin Flint murmur B) Pulsus bisferiens C) Pulsus paradoxus D) Pulsus parvus et tardus

Explanation: Pulsus bisferiens is an arterial phenomenon seen in hypertrophic cardiomyopathy. It is characterized by the presence of two distinct peaks of the arterial pulse during systole. While the etiology of pulsus bisferiens is not clear, it is thought to be related to the blockage of the left ventricular outflow tract. It can also be detected in patients with aortic regurgitation. Hypertrophic cardiomyopathy (HCM) is an autosomal dominant condition characterized by thickening of one or more areas of the left ventricle wall or interventricular septum leading to filling defects and diastolic dysfunction. Interventricular septal hypertrophy in particular can lead to left ventricular outflow tract (LVOT) obstruction. Obstructive HCM is more common (70% of cases) than non-obstructive HCM. HCM is often diagnosed via routine or family screening with echocardiography, and most patients are diagnosed in late-childhood or adolescence. Symptomatic patients typically present with activity-associated symptomatology including fatigue, dyspnea, syncope, palpitations and chest pain. Physical exam findings in HCM include a harsh crescendo-decrescendo systolic ejection murmur that decreases with squatting and increases with the valsalva maneuver, loud S4 heart sound, a laterally-displaced left ventricular apical impulse, and a parasternal lift associated with mitral regurgitation. An echocardiography is the diagnostic modality of choice for HCM, and cardiac catheterization may also aid in diagnosis. Echocardiogram findings include ventricular or interventricular septal hypertrophy (most common), the degree of LVOT obstruction, and possibly systolic anterior motion of the mitral valve which may contribute to the degree of LVOT obstruction. Cardiac catheterization allows for measurement of intracardiac pressures and provides a value for the LVOT gradient. In general, the larger the LVOT gradient the more symptomatic the patient. Medical management includes the use of beta-blockers and nondihydropyridine calcium channel blockers which slow the heart rate and prolong diastole allowing more time for ventricular filling. Diuretics are avoided in patients with HCM as they can reduce the preload and worsen LVOT obstruction. Patients with HCM are predisposed to ventricular dysrhythmias and at risk for sudden death, so patients should avoid strenuous exercise. Patients at particular risk for arrhythmias should be considered for placement of intracardiac defibrillator (ICD). Given that there is a genetic component to this disease, all first-degree relatives should undergo screening evaluation, including a physical exam, electrocardiography, and echocardiography.

Which of the following best describes the pathophysiology responsible for causing restrictive cardiomyopathy? A) Decreased cardiac contractility due to infiltration of the ventricular walls B) Decreased diastolic filling due to hypertrophy of the ventricular walls C) Decreased diastolic filling due to rigidity of the ventricular walls D) Increased left ventricular systolic pressure due to a narrowed outflow tract

Explanation: Restrictive cardiomyopathy is a cardiac condition where stiffened, noncompliant ventricles reduce diastolic filling, which results in diastolic dysfunction. The most common cause of restrictive cardiomyopathy is infiltrative disease (e.g., amyloidosis, sarcoidosis). This infiltration of the cardiac musculature causes decreased diastolic filling due to rigidity of the ventricular walls with subsequent low preload and high filling pressures. To compensate, the atria dilate and atrial pressures are elevated, causing characteristic signs and symptoms of right heart failure, such as dyspnea and peripheral edema. The reduced compliance of the ventricles does not inhibit their contractility, and systolic function is preserved in most cases of early disease. Restrictive cardiomyopathy can be classified based on the underlying pathophysiologic process as either hypertrophic (e.g., hypertension, aortic stenosis), deposition (including infiltrative disease and storage disorders, such as hemochromatosis), inflammatory (e.g., postirradiation), or primary (e.g., diabetic, familial, idiopathic). Restrictive cardiomyopathy is a frequent cause of death in Africa, India, South and Central America, and Asia due to the high incidence of endomyocardial fibrosis in these areas. The nature of the disease process is slowly progressive, however, once signs and symptoms are apparent, the disease is already advanced and heart failure progresses rapidly. Systemic venous congestion secondary to decreased compliance of the right ventricle causes peripheral edema, ascites, and hepatic and bowel congestion, which primarily manifests as loss of appetite. Patients also complain of palpitations, fatigue, weakness, and exercise intolerance. Physical exam reveals elevated jugular venous pressure with Kussmaul sign, which is an increase in jugular venous distention with inspiration. Pulse may be normal or tachycardic, indicating low stroke volume. Cardiac auscultation reveals a third heart sound (S3 gallop) due to abrupt cessation of rapid ventricular filling. Findings on ECG are nonspecific and may include atrial fibrillation, ST-T wave abnormalities, premature atrial and ventricular beats, atrioventricular block, and intraventricular conduction delay. Chest radiograph reveals cardiomegaly due to atrial enlargement and pulmonary venous congestion. Pleural effusions may be present. Plasma brain natriuretic peptide can be useful in differentiating restrictive cardiomyopathy from the similar presentation of constrictive pericarditis, with levels ≥ 400 pg/mL suggestive of restrictive cardiomyopathy. Further evaluation using echocardiography can help establish the diagnosis. All forms of restrictive cardiomyopathy share commonalities on echocardiography: biatrial enlargement, normal or small left ventricular cavity size with preserved left ventricular systolic function, and abnormal diastolic function. In infiltrative disease, left ventricular wall motion abnormalities and thickened ventricular walls may be apparent. Cardiac magnetic resonance imaging can provide evidence of fibrosis, infiltration, necrosis, and scar tissue that can be correlated with disease processes that cause the condition to further the diagnostic evaluation of restrictive cardiomyopathy. If workup is inconclusive, endomyocardial biopsy is used to identify specific secondary causes, such as amyloidosis, sarcoidosis, and hemochromatosis. As most causes of restrictive cardiomyopathy are not curable, patients are symptomatically treated with the goal of reducing pulmonary and systemic congestion. Loop diuretics (e.g., furosemide), calcium channel blockers (e.g., verapamil), and angiotensin-converting enzyme inhibitors (e.g., enalapril) are standard treatments to ameliorate congestive symptoms. Cardiac transplantation can be a definitive cure but underlying etiologies may prevent this option. Atrial fibrillation, a common complication secondary to atrial dilation, can predispose patients to thromboembolism and stroke and should be treated with anticoagulation (e.g., warfarin, dabigatran). Due to the rapid onset and progression of heart failure, the majority of patients die within a few years following diagnosis. Decreased cardiac contractility due to infiltration of the ventricular walls (A) is not a component of restrictive cardiomyopathy pathophysiology. While infiltration of the cardiac musculature is a common cause of restrictive cardiomyopathy, cardiac contractility is relatively preserved in these patients. Hypertrophic cardiomyopathy results in decreased diastolic filling due to hypertrophy of the ventricular walls (B). This inherited genetic disorder presents with diastolic dysfunction secondary to impaired ventricular relaxation and decreased filling volume due to thickened walls. It can also present with increased left ventricular systolic pressure due to a narrowed outflow tract (D). The systolic anterior motion of the mitral valve, in conjunction with hypertrophy of the septum, causes a smaller passageway for blood to exit the ventricle, thereby resulting in a greater systolic pressure within the left ventricle during systole. Hypertrophic cardiomyopathy is an important cause of sudden cardiac death and its murmur is distinguished from the similar murmur of aortic stenosis by its increase in intensity during valsalva and standing.

45-year-old man with a history of hypertension presents to his primary care provider for follow-up. He states that he was unable to refill his prescription for hypertension medications. He states he is feeling well. His blood pressure is 182/122 mm Hg. Which of the following is the most likely diagnosis? A) Hypertensive emergency B) Hypertensive urgency C) Resistant hypertension D) White coat hypertension

Explanation: Severe hypertension, hypertension with a systolic pressure of ≥ 180 mm Hg or a diastolic pressure of ≥ 120 mm Hg, can be either symptomatic with signs of acute end organ damage or asymptomatic with no end organ damage present. If asymptomatic, it is known as a hypertensive urgency. Risk factors include hypertension that is uncontrolled or patients who are noncompliant with their hypertension medications. Noncompliance to a low sodium diet can also lead to a severe elevation in blood pressure regardless of compliance to hypertension medications. If left untreated, the condition can progress to target organ damage and become a hypertensive emergency. Patients with hypertensive urgency may be completely asymptomatic, however, many will present with a mild headache. Vital signs will show an elevation of the systolic pressure to ≥ 180 mm Hg or a diastolic pressure of ≥ 120 mm Hg; both may be elevated. Diagnosis is based on the elevation of blood pressure and the exclusion of end organ damage. Treatment of the condition is important in preventing the progression to hypertensive emergency. The blood pressure should be reduced within 24 hours, longer in the elderly or in those at risk for ischemia should the blood pressure drop too rapidly. The initial target is a blood pressure of < 160/< 100 mm Hg. During the first two hours, the blood pressure should only be reduced by 25 to 30%, which may prolong the length of time it takes to reach the initial target. This slow reduction will allow for adequate perfusion of the tissues, which will lower the risk of cerebral ischemia, myocardial infarction, or acute kidney injury. Short-acting oral medications, such as captopril or clonidine, or long-acting oral medications, such as amlodipine and chlorthalidone, are commonly used to reach the initial target. Long-term target blood pressure is < 130/< 80 mm Hg. Patient management will depend on the circumstances. Restarting medications or increasing the dose and encouraging compliance may be sufficient in some cases. Adding a diuretic may be helpful in those who are not adhering to a low sodium diet. Patients who have previously untreated hypertension should be placed on a combination therapy of two medications. Patients should be followed closely to best monitor blood pressure and maintain target goals. They should be advised to adhere to a low sodium diet and comply with their hypertension medications to prevent further hypertensive urgencies. A hypertensive emergency (A) is defined as a severe hypertension that is associated with acute end organ damage. In this case, hypertensive encephalopathy, retinal hemorrhages, papilledema, or acute and subacute kidney injury would be present. This is a life-threatening condition. Resistant hypertension (C) is an elevated blood pressure reading that remains elevated despite combination treatment with at least three medications. White coat hypertension (D) is seen in a patient who has normal blood pressure readings when monitored outside of the office while the readings in office are elevated to ≥ 130/≥ 80 mm Hg. These patients should have their out of office blood pressure monitored annually as they are at risk of developing hypertension.

An 80-year-old woman presents to the emergency department with chest pain and shortness of breath. Vital signs are temperature 37.5°C, heart rate 115 beats per minute, and blood pressure 75/40 mm Hg. On exam, extremities are cool and pale. Jugular venous distension is present. Auscultation of the chest reveals tachycardia with crackles in the lung bases bilaterally. Electrocardiogram shows ST elevation in leads II, III, and avF. Which of the following is the most likely mechanism of this patient's shock? A) Anaphylactic shock B) Cardiogenic shock C) Hypovolemic shock D) Septic shock

Explanation: Shock is a condition related to insufficient oxygen delivery that results in cellular and tissue hypoxemia. Factors that ensure adequate tissue perfusion are cardiac output (composed of stroke volume and heart rate) and systemic vascular resistance (determined by vessel length, tone, and blood viscosity). If any of these elements are compromised, a patient can develop shock. When this occurs secondary to an intracardiac disorder, it is termed cardiogenic shock. Disorders that lead to cardiogenic shock can be divided into three categories: cardiomyopathic, arrhythmic, and mechanical. Cardiomyopathic causes are those that affect the heart muscle itself such as acute congestive heart failure or acute myocardial infarction that causes stunning/death to part of the heart muscle. Atrial arrhythmias, ventricular arrhythmias, and heart blocks can also lead to cardiogenic shock. Mechanical causes of cardiogenic shock include valvular disorders such as severe aortic regurgitation and mitral regurgitation. Other etiologies include severe ventricular septal defects, interventricular septum rupture, or ventricular free wall rupture. Clinical presentation may include altered mental status, chest pain, and shortness of breath. On physical exam, patients are typically afebrile with tachycardia, hypotension, tachypnea, and have cool, pale extremities. Patients may also demonstrate signs of fluid overload and pulmonary edema, including jugular venous distention and crackles in the lungs. Laboratory studies include an elevated lactic acid level which is produced from increased anaerobic metabolism due to low oxygen delivery to the tissues. Brain natriuretic peptide, a hormone released by cardiac cells when the heart is stretched, will also be elevated. Cardiac enzymes, such as troponin I or T, are checked to rule out myocardial infarction. Electrocardiogram should be obtained to evaluate for arrhythmia or acute myocardial infarction, as in the case above. Chest radiograph typically reveals pulmonary edema. Diagnosis of cardiogenic shock is suspected based on presentation and physical exam. Echocardiogram aids in diagnosis by showing a reduced left ventricular ejection fraction and potentially the cause for the cardiogenic shock. Treatment of cardiogenic shock includes identifying and treating underlying cause, initiating vasopressors or inotropes (dopamine, norepinephrine, dobutamine), and diuretics. Anaphylactic shock (A) is a type of distributive shock resulting from a life-threatening allergic response to food, medication, or an insect sting. Allergic reactions occur when a patient is exposed to an antigen that results in an immune-mediated release of inflammatory cells which leads to symptoms such as pruritus and hives. In anaphylaxis, a massive inflammatory response occurs, resulting in bronchospasm, airway constriction, and shock. Clinical presentation includes hives, edema of the face or oral cavity, and stridor. Patients are often tachycardic and hypotensive as well. Recent exposure to a known allergen is also common. Prompt recognition of this form of shock is crucial, and epinephrine should be administered immediately. Hypovolemic shock (C) occurs as a result of overall volume depletion within the vascular system. Hypovolemic shock can be divided into two categories: hemorrhagic and nonhemorrhagic. In hemorrhagic shock, acute blood loss leads to a low volume state with resulting shock. Non-hemorrhagic hypovolemic shock results from fluid loss other than blood. This type of shock is often accompanied by a history of poor oral intake, large urinary output, or large gastric output. Other examples include prolonged heat exposure and burns. Patients also present with tachycardia and hypotension, however, they do not have signs of pulmonary edema or an abnormal electrocardiogram. Septic shock (D) is the most common type of distributive shock and results from a widespread, inflammatory response to infection. This leads to global vasodilation, cellular injury, and organ failure. Similar to cardiogenic shock, clinical presentation may include altered mental status with abnormal vital signs such as tachypnea and tachycardia, but patients are commonly febrile. On exam, patients may have warm extremities initially and develop cool and mottled extremities as the condition progresses. Patients do not typically show signs of pulmonary edema or fluid overload as in the case above.

Which of the following should be considered a differential diagnosis for sick sinus syndrome? A) Cocaine usage B) Hypothyroidism C) Pheochromocytoma D) Sepsis

Explanation: Sick sinus syndrome (SSS) is characterized by clinical signs and symptoms including dizziness, fatigue, dyspnea, palpitations, presyncope and syncope, and electrocardiogram (ECG) abnormalities such as bradycardia, sinus pauses, or sinus arrest. It is thought to be attributable to dysfunction of the sinus node, which acts as the intrinsic pacemaker of the heart. SSS is usually seen in the older population and is considered a diagnosis of exclusion after all other potentially reversible causes of bradycardia or sinus node dysfunction have been ruled out. This includes medication use such as beta-blockers, calcium channel blockers, or other antiarrhythmics, or other potential causes, including myocardial ischemia, atrioventricular node dysfunction including heart blocks, other neurogenic or cardiogenic causes of syncope, and hypothyroidism. The diagnosis of SSS is made when there is no evidence for reversible causes of the symptoms or with lack of improvement in symptoms after reversible conditions are addressed. Immediate management of symptomatic bradycardic patients follows the advanced cardiac life support (ACLS) protocol. Long-term management of stable patients involves consideration of pacemaker placement. Cocaine use (A), pheochromocytoma (C), and sepsis (D) all present with tachycardia. Cocaine use should be considered in any patient presenting with tachycardia and is easily identifiable on urine toxicology screening. Cocaine can lead to coronary vasospasm and ischemia, and thus it is important to consider in tachycardic patients or patients presenting with chest pain or dyspnea. Pheochromocytoma is a neuroendocrine tumor of the adrenal glands characterized by abnormally high levels of catecholamines norepinephrine and epinephrine. While rare, it is an important potentially-reversible cause of tachycardia and hypertension. Pheochromocytoma is diagnosed via 24-hour urine catecholamine and metanephrine collection and is treated with surgical excision of the tumor. Sepsis can present with signs of hemodynamic instability and shock including hypotension, hypo- or hyperthermia, and tachycardia. Septic patients appear clinically ill and should undergo aggressive fluid resuscitation as well as blood cultures and lactate measurement prior to initiation of antibiotics.

A morbidly obese, 60-year-old woman presents to the emergency department with the complaint of left leg pain. On physical exam, an area of edema above the left medial malleolus that extends coronally is visible. The area is tender to palpation. A palpable cord is present. Bilateral varicose veins are visible. A duplex ultrasound reveals vein wall thickening, subcutaneous edema, and thrombus occluding the great saphenous vein. Which of the following is the most likely diagnosis for this condition? A) Deep vein thrombosis B) Phlebitis C) Superficial thrombophlebitis D) Superficial vein thrombosis

Explanation: Superficial vein thrombosis of the lower extremity is the occlusion of an axial vein, either the great saphenous vein or small saphenous vein. This condition is more common in women. Risk factors include varicose veins, obesity, injury, intravenous drug use, and hypercoagulability due to pregnancy, malignancy, or other condition. Presentation of superficial vein thrombosis includes erythema and pain along the inner medial thigh following the course of the vein. Both superficial thromboembolism and phlebitis are most likely to occur in varicose veins. A palpable cord due to the occlusion of the vein will be present but may be more difficult to discern in an obese patient as the cord location may be several centimeters below the skin surface. It is important to diagnose the location of the thrombus as occlusion of the axial veins can lead to thrombus in the deep veins. Duplex ultrasound is used to make the diagnosis. Treatment depends on the cause of the thrombus. Most cases of superficial vein thrombosis should be treated with anticoagulation for 45 days as the condition can progress to deep vein thrombosis. D-dimer should be performed prior to initiating anticoagulation and after 45 days to confirm thrombus resolution. Should there be a contraindication to anticoagulation, serial duplex ultrasounds are indicated and ligation of the saphenofemoral junction may be considered. If there is recurrence of the thrombus in the same vein despite conservative therapy, ligation or excision should be considered. Complete resolution of the thrombus usually takes a week, but the cord can be present for several months. Antibiotic therapy should be administered if there is concern of suppurative thrombophlebitis. To prevent future episodes of thrombus and enhance recovery, patients should be advised to wear compression stockings.

A 2-day-old female newborn presents to her pediatrician for her first visit after birth. When she cries, the pediatrician notices her nail beds and lips have a bluish color. She is sent for echocardiogram to confirm the pediatrician's suspicions of Tetralogy of Fallot. Which anatomic findings on echogram confirm the diagnosis? A) Aorta rising from the right ventricle and pulmonary artery arising from the left ventricle B) Tricuspid atresia and ventricular septal defect C) Ventricular septal defect and a single great vessel arising from the heart D) Ventricular septal defect and overriding aorta

Explanation: Tetralogy of Fallot (TOF) is a congenital heart condition characterized by four primary features: (1) pulmonary artery stenosis, (2) ventricular septal defect, (3) overriding aorta, and (4) right ventricular hypertrophy. Anatomically, it is the large ventricular septal defect and overriding aorta which seem to lead to the development of right ventricular hypertrophy and pulmonary stenosis. TOF is associated with some genetic syndromes, such as Down syndrome, DiGeorge syndrome, and Alagille syndrome, but it can also be noted in infants without other syndromic anomalies. Depending on the degree of right ventricular outflow obstruction and pulmonary flow, infants with TOF may present with or without cyanosis. Cyanosis may only be appreciated during hypercyanotic spells, known as "tet" spells, more commonly seen when infants are agitated. In some cases, a murmur reflecting the obstruction (as opposed to the ventricular septal defect), may be appreciated, which typically presents as a systolic crescendo-decrescendo murmur with a harsh ejection quality along the left mid to upper sternal border with posterior radiation. Pulse oximetry may also flag some newborns for further evaluation. Definitive diagnosis is obtained through echocardiography, which enables assessment of all features of TOF both for diagnosis and preoperative evaluation. Chest x-ray classically shows boot-shaped heart. Almost all infants with TOF will require surgical repair before they are 12 months of age. An aorta rising from the right ventricle and pulmonary artery arising from the left ventricle (A) describes transposition of the great arteries, which leads to a parallel circulation causing newborn cyanosis. Tricuspid atresia (B) is one of several types of tricuspid valve abnormalities that can lead to newborn cyanosis. It is often accompanied by a ventricular septal defect. In tricuspid atresia, there is a complete lack of communication between the right atrium and right ventricle, leading to a right-to-left shunt and newborn cyanosis. Tricuspid stenosis and Ebstein's anomaly are other tricuspid defects that can cause newborn cyanosis. A ventricular septal defect with a single great vessel arising from the heart (C) describes truncus arteriosus, another common cause of cyanosis in a newborn. Because the aorta receives all the output from both the right and left ventricles, cyanosis results.

Which of the following blood pressure readings, when confirmed with repetitive measurements, is consistent with stage I hypertension according to the 2017 American College of Cardiology and American Heart Association guidelines? A) 110/78 mm Hg B) 118/82 mm Hg C) 129/79 mm Hg D) 143/91 mm Hg

Explanation: The American College of Cardiology (ACC) and American Heart Association (AHA) released new blood pressure guidelines in 2017. These guidelines changed the definition of elevated blood pressure, stage I hypertension, and stage II hypertension. In addition, the blood pressure threshold for initiating antihypertensive pharmacologic therapy also changed. Normal blood pressure is still defined as ≤ 120/80 mm Hg. Prehypertension was eliminated and replaced by the term elevated blood pressure. Elevated blood pressure is defined as a systolic blood pressure from 121-129 mm Hg. The threshold for stage I hypertension has been lowered to a systolic blood pressure between 130-139 mm Hg or a diastolic blood pressure ranging from 81-89 mm Hg. The threshold for stage II hypertension has been lowered to systolic blood pressure ≥ 140 mm Hg or diastolic blood pressure ≥ 90 mm Hg. 118/82 mm Hg is within the range for stage I hypertension. Prior to diagnosing a patient with hypertension, at least two blood pressure measurements on at least two occasions are required. Out-of-office blood pressure measurements are preferred to confirm the diagnosis of hypertension and to titrate antihypertensive pharmacologic therapy. The benefit of pharmacologic therapy for hypertension depends on the patient's overall risk of cardiovascular disease (CVD). Patients with a history of cardiovascular disease, such as a myocardial infarction (MI) or cerebrovascular accident (CVA), should be started on blood pressure reducing pharmacologic therapy for secondary prevention of cardiovascular disease if their blood pressure is consistently ≥ 130/80 mm Hg. The decision to treat patients with no history of cardiovascular disease for blood pressure reduction depends in part on the patient's overall risk for future cardiovascular disease. This can be determined by calculating the 10-year atherosclerotic cardiovascular disease (ASCVD) risk for a patient. If the 10-year risk of atherosclerotic cardiovascular disease is ≥ 10% in a patient without previous cardiovascular disease, then pharmacologic therapy is also recommended for blood pressure reduction in patients with consistent blood pressure ≥ 130/80 mm Hg. Patients without prior cardiovascular disease at < 10% risk of cardiovascular disease in the next 10 years should be treated with pharmacologic therapy for hypertension when blood pressure is consistently ≥ 140/90 mm Hg. Patients treated with pharmacologic therapy for blood pressure should have a treatment goal of < 130/80 mm Hg. The first-line pharmacologic medications for blood pressure reduction in the absence of specific indications for a medication are angiotensin-converting enzyme inhibitors (ACEI), angiotensin II receptor blockers (ARBs), thiazide diuretics, and calcium channel blockers. 110/78 mm Hg (A) is a normal blood pressure reading. 129/79 mm Hg (C) falls within the new elevated blood pressure range because the systolic blood pressure is between 120 and 129 mm Hg. 143/91 mm Hg (D) falls within the range of stage II hypertension because the systolic and diastolic blood pressure readings are ≥ 140/90 mm Hg.

Which of the following medications lowers patient mortality when used in the treatment of heart failure? A) Digoxin B) Furosemide C) Hydrochlorothiazide D) Metoprolol

Explanation: The goals of treating heart failure with pharmacologic therapy are to improve symptoms, to reverse or slow deterioration in myocardial function, and to decrease mortality. Improvement in symptoms can be achieved by diuretics, beta-blockers, angiotensin-converting enzyme inhibitors (ACEIs), angiotensin II receptor blockers, hydralazine, nitrates, digoxin, and mineralocorticoid receptor antagonists. Mortality is also decreased with these medications, with the exception of loop diuretics and digoxin, which improve symptoms but do not decrease mortality. Beta-blockers, such as metoprolol, are an example of a medication class that lowers patient mortality when used in the treatment of post-MI heart failure. Carvedilol, metoprolol, and bisoprolol are the three beta-blockers that have been approved for the treatment of heart failure. Beta-blockers should not be initiated during acute exacerbations of heart failure symptoms. Loop diuretics, such as furosemide, are used to just improve symptoms in patients with volume overload due to heart failure with reduced ejection fraction (systolic heart failure). Symptomatic improvement, such as improved dyspnea or peripheral edema, often occurs within hours or days of beginning loop diuretics. Angiotensin-converting enzyme inhibitors are recommended in patients with left ventricular systolic dysfunction. This drug class works by dilating blood vessels and decreasing the production of aldosterone. ACE inhibitors reduce mortality, prolong survival, and alleviate symptoms. Angiotensin II receptor blockers are used in patients with systolic heart failure who cannot tolerate angiotensin-converting enzyme inhibitors due to cough or angioedema. Mineralocorticoid receptor antagonists, such as spironolactone, are recommended for patients with more advanced stages of heart failure and limitation in physical activity. Hydralazine and oral nitrates are recommended for African American patients with systolic heart failure (have been shown to improve mortality in African Americans) and moderate to severe symptoms despite therapy with diuretics, angiotensin-converting enzyme inhibitors, and beta-blockers. Hydralazine and nitrate therapy can also be used in patients who cannot tolerate angiotensin-converting enzyme inhibitors or angiotensin II receptor antagonists. Ivabradine is a sinus node inhibitor that is recommended in patients with systolic heart failure and a heart rate above 70 beats per minute despite either already taking a beta-blocker or being intolerant to beta-blockers. Ivabradine reduces the risk of hospitalization in patients with chronic heart failure with a heart rate above 70 beats per minute. Digoxin is rarely used to improve symptoms in patients with systolic heart failure, but it does not improve mortality. Standard treatment of heart failure usually includes beta blocker, ACE inhibitor, and a loop diuretic. Other medications such as digoxin, hydralazine, nitrate, spironolactone may be added depending on severity of heart failure.

A 65-year-old man is admitted with pneumonia. You are called to see the patient for sudden-onset dizziness and chest pain. On exam, the patient is awake and alert, has a rapid pulse, and blood pressure is 110/60 mm Hg. Electrocardiogram is consistent with torsades de pointes. Which of the following is the most appropriate initial treatment? A) Amiodarone 300 mg IV B) Emergent cardiac defibrillation C) Epinephrine 1 mg IV D) Magnesium sulfate 2 g IV

Explanation: Torsades de pointes is a form of polymorphic ventricular tachycardia that results from a prolonged QT interval. The condition may be congenital or acquired. Electrocardiogram reveals a wide complex, polymorphic ventricular tachycardia with a gradual change in the amplitude of the QRS complexes that twist around the isoelectric line. Other findings include a prolonged QT interval and a ventricular rate can range between 150-300. Patients may complain of chest pain, palpitations, and dizziness. Torsades de pointes may be short and self-limiting, however, these episodes can recur multiple times in a row and progress ventricular fibrillation. First-line treatment of torsades de pointes in a hemodynamically stable patient is magnesium sulfate 2 g IV. Additional measures include correcting other electrolyte derangements and discontinuing current medications that prolong the QT interval. Amiodarone 300 mg IV (A) is not an appropriate choice for the treatment of torsades de pointes. Amiodarone is a class III antiarrhythmic agent that is used for the treatment of ventricular arrhythmias, however, it should not be used in patients with torsades de pointes as it can cause QT prolongation. Emergent cardiac defibrillation (B) is an appropriate treatment intervention for a patient with torsades de pointes who is unresponsive or hemodynamically unstable, which was not the case with the patient above. Epinephrine 1 mg IV (C), also known as adrenaline, is an alpha- and beta-adrenergic receptor agonist that is used in the treatment of anaphylaxis, shock, and for patients with pulseless ventricular tachycardia, ventricular fibrillation, and asystole. It is not indicated for the treatment of torsades de pointes unless the patient progresses to pulseless ventricular tachycardia or ventricular fibrillation.

Which of the following electrolyte abnormalities is associated with the rhythm seen on the above electrocardiogram? A) Hypercalcemia B) Hyperkalemia C) Hypomagnesemia D) Hyponatremia

Explanation: Torsades de pointes is a type of polymorphic ventricular tachycardia associated with a prolonged QT interval. Hypomagnesemia, hypokalemia, and hypocalcemia can all predispose patients to the development of Torsades via acquired prolonged QT interval. Medications are one of the most common causes of acquired prolonged QT interval including certain antiarrhythmics, macrolide and fluoroquinolone antibiotics, antifungals, psychotropic medications, and gastrointestinal motility agents. Patients can also have congenital prolonged QT syndrome, which places them at higher risk for Torsades. Torsades de pointes is initially treated with intravenous magnesium sulfate, which prevents further prolonged QT-triggered premature ventricular beats from triggering further torsades de pointes. Patients with congenital prolonged QT interval may be managed with beta-blockers to shorten the QT interval. Hypercalcemia (A) causes a shortening of the QT interval and can be caused by primary hyperparathyroidism or malignancy. Hyperkalemia (B) is a plasma potassium level > 5.0 mEq/L and is associated with loss of P waves, peaked T waves, and wide QRS complexes seen on ECG. Treatment includes myocyte stabilization with calcium gluconate followed by insulin and dextrose intravenously to drive the excess potassium into the cells or furosemide to increase potassium excretion in the urine. Hyponatremia (D) is a plasma sodium level of < 135 mEq/L and is associated with headaches, nausea, vomiting, lethargy, confusion, seizures, and altered mental status in advanced stages. It is not associated with any cardiac conduction abnormalities. Hyponatremia can be divided into hypovolemic, euvolemic, or hypervolemic hyponatremia, and management consists of either fluid restriction or sodium chloride supplementation depending on the cause.

A 56-year-old man with a history of asthma, hypercholesterolemia, hypertension, and left-sided heart failure presents for a routine visit to have his prescriptions refilled. On exam, a new murmur is appreciated, a 2/6 holosystolic murmur best heard at the right sternal border without radiation. An echocardiogram confirms new pathology of tricuspid regurgitation. What element of his past medical history is a significant risk factor for developing this problem? A) Asthma B) Hypercholesterolemia C) Hypertension D) Left-sided heart failure

Explanation: Tricuspid regurgitation is a normal physiologic finding in many adults. In most cases, patients are asymptomatic and the valve problem is detected incidentally on echocardiogram or CT. Abnormal degrees of regurgitation are a relatively common cardiac abnormality most commonly associated with right-sided heart problems, specifically right atrium dilatation and right ventricle and pulmonary hypertension, which increases right-sided pressures. Disease processes that lead to right-sided heart dysfunction include left-sided heart failure (the most common cause), mitral or pulmonic valve disease, primary pulmonary disease, pulmonary artery stenosis, left-to-right cardiac volume shunting, and hyperthyroidism. Primary causes of tricuspid regurgitation are far less common but include rheumatic heart disease, Epstein anomaly, infectious endocarditis (IV drug users), trauma, and connective tissue disorders. In cases of moderate to severe tricuspid regurgitation, a blowing holosystolic murmur may be appreciated at the right or left mid-sternal border without radiation. Other physical findings associated with severe tricuspid regurgitation include jugular venous distention, prominent V waves in jugular venous pulse with rapid descent, pulsatile liver, peripheral edema, ascites, and hepatomegaly. Atrial fibrillation (Afib) is usually present. Echocardiography is the primary test used to evaluate the extent of disease and follow its progress. ECG may reveal right ventricular and right atrial enlargement. Treatment is based on severity and may include lifestyle changes, medical management, and surgery. Asthma (A) is not a known risk factor for tricuspid regurgitation or other types of murmurs. Hypercholesterolemia (B) increases risk for aortic and aortic valve stenosis, usually presenting as a harsh crescendo-decrescendo systolic ejection murmur heard best in second right intercostal space and radiating to carotid arteries. Hypertension (C) is not directly associated with any heart murmurs but may contribute to heart failure, which is associated with multiple abnormal heart sounds.

A 32-year-old woman presents to the clinic for a follow-up visit. She was seen three months ago for bothersome varicosities of the left lower extremity. She has used compression stockings every day for the past three months, has begun exercising daily, elevates her leg whenever possible, and quit smoking. However, the varicose veins are still painful and cosmetically troublesome. A venous Doppler of the lower extremities reveals saphenous vein reflux on the left and no evidence of deep or superficial venous thrombosis. Which of the following is the next best step in treating this patient's varicosities? A) Ablation of the perforator veins B) Ablation of the saphenous vein C) Sclerotherapy of the varicose veins D) Surgical excision of the varicose veins

Explanation: Varicose veins are enlarged, tortuous superficial veins resulting from venous hypertension. Most commonly, valvular insufficiency is to blame, although decreased function of the venous pump, venous wall deformities, and venous thrombosis are also causative factors. Risk factors for the development of varicose veins include pregnancy, obesity, sedentary lifestyle, prolonged standing, family history of varicosities, genetic syndromes, smoking, advanced age, trauma, and previous superficial or deep vein thrombosis. Varicose veins can be diagnosed clinically, although Doppler ultrasound of the affected extremity is useful to rule out thrombosis and to guide treatment decisions. First-line therapy for varicose veins includes compression stockings and lifestyle modifications, such as smoking cessation, exercise, and elevation of the affected limb. After three months of conservative therapy, more invasive therapy can be considered, especially for patients who continue to have pain or are troubled by the cosmetic appearance of the varicosities. Sclerotherapy of superficial veins involves injecting a sclerosing agent directly into the varicose vein. Thermal vein ablation uses laser or radiofrequency energy to destroy the vein walls. Surgical excision of the varicose veins can also be performed. In patients with superficial varicosities that are the result of deep vein insufficiency and reflux, ablation of the deep vein should be performed first (or may be performed concomitantly) before superficial veins are treated. This treatment will result in a lower incidence of recurrence of the superficial varicose veins. Ablation of the perforator veins (A) is a good approach for the treatment of varicose veins when deep veins are not affected. This patient has reflux of the saphenous vein, so ablation of the deeper vein (the saphenous) is a better choice. Sclerotherapy of the varicose veins (C) can be performed in this patient and will most likely need to be performed at some point. However, ablation of the deeper refluxing vein first will increase the likelihood that sclerotherapy of the varicosities will be curative. Surgical excision of the varicose veins (D) is a more invasive option, involving more risks than thermal ablation or sclerotherapy. Also, the deeper venous reflux should be addressed prior to superficial treatments.

Which of the following best describes the primary physiologic dysfunction associated with varicose veins of the lower extremities? A) Chronic release of inflammatory mediators B) Diminished venous pump activity C) Valvular incompetence D) Venous wall deformities

Explanation: Varicose veins are most common in the lower extremities and are the result of venous hypertension. The primary physiologic dysfunction leading to venous hypertension is valvular incompetence. When valves of the venous system become incompetent, the blood is directed in a reverse fashion from the deep venous system to the superficial venous system. This redirection leads to superficial venous congestion and propagates more valvular incompetence in those veins. Valvular incompetence leads to congestion, stasis, and tortuous, visible, and sometimes painful superficial veins. Risk factors for the development of varicose veins include pregnancy, obesity, sedentary lifestyle, prolonged standing, family history of varicosities, certain genetic syndromes, lax ligaments, smoking, advanced age, trauma, and previous superficial or deep vein thrombosis. The diagnosis of varicose veins is made clinically, although signs and symptoms of deep vein thrombosis, such as acute onset of unilateral lower extremity pain and edema and a positive Homan sign, should be evaluated with Doppler ultrasound. First-line treatment of varicose veins involves lifestyle modifications and pressure stockings. If these measures fail, venoactive substances, such as flavonoid supplements, may be of some benefit. Rheologic agents, such as aspirin and pentoxifylline, improve healing of stasis ulcers caused by venous insufficiency. Injection of sclerosing agents into varicose veins can be performed as an elective procedure for those with cosmetically troublesome varicosities or for patients with painful varicosities that are inadequately treated with conservative measures. Chronic release of inflammatory mediators (A) is part of the physiologic cascade leading to venous stasis ulcers in those with venous hypertension. Venous hypertension produces stress on the vein walls, which leads to the release of inflammatory molecules into the surrounding tissues. The result can be edema or lipodystrophic changes to the extremity. Lipodermatosclerosis describes epidermal and dermal layers that are adhered to the subcutaneous tissue via inflammatory mediators. This tissue has reduced vascularity and an increased risk of ulceration. Diminished venous pump activity (B) secondary to weak leg muscles, sedentary lifestyle, inactivity, or obesity can lead to venous hypertension and is a cause of varicose veins, although valvular incompetence is more common. Venous wall deformities (D) occur after venous hypertension has already begun. Valves become incompetent, which leads to increased venous pressure on the vein walls. As the vein walls become deformed, more valvular incompetence is propagated.

Which of the following is the first-line pharmacologic therapy for preventing symptomatic episodes in patients with vasospastic angina? A) Nitroglycerin B) Propranolol C) Sumatriptan D) Verapamil

Explanation: Vasospastic angina is a condition that causes myocardial ischemia due to transient, abrupt spasm of a coronary artery. Vasospastic angina was previously called Prinzmetal and variant angina. The presentation is similar to anginal episodes in patients with obstructive coronary artery disease with a few important differences. Episodes in vasospastic angina occur predominantly at rest and frequently occur between midnight and the early morning. This is in contrast to obstructive coronary artery disease, such as stable angina, when the pain usually occurs with exertion. Vasospastic angina episodes typically last five to 15 minutes. Patients with vasospastic angina may be younger and have less cardiovascular disease risk factors. The similarities between the two types of angina include the location, quality, and associated symptoms of the pain. During episodes of vasospastic angina (chest pain), these patients will have typical ECG changes associated with transient myocardial ischemia, such as ST depression or ST elevation. These ECG changes usually last about 15 minutes. Stress ECG is usually normal in patients with vasospastic angina, although sometimes the exercise induces an episode of vasospasm, which would cause ST changes during the stress test. The workup of a patient with angina should focus on ruling out obstructive coronary artery disease since this is a more common condition than vasospastic angina. Therefore, coronary arteriography is often needed to rule out obstructive coronary artery disease before vasospastic angina can be diagnosed. Treatment of vasospastic angina is aimed at reducing the symptomatic episodes and complications including dysrhythmias. Sublingual nitroglycerin is used as needed at the start of symptomatic episodes to reduce the duration of ischemia. Calcium channel blockers, such as verapamil, are the first-line daily therapy to prevent symptomatic episo

A 2-week-old female newborn presents to her pediatrician for fussiness and poor feeding. On presentation, she is noted to be afebrile and well-appearing but tachypneic and tachycardic. Her physical exam is significant for a grade 2/6 blowing holosystolic murmur best heard at the left mid to lower sternal border. An echocardiogram is ordered due to concern for ventricular septal defect. Which of the following diagnoses should also be considered based on her presentation? A) Bronchiolitis B) Pneumonia C) Transposition of the great arteries D) Tricuspid regurgitation

Explanation: Ventricular septal defect (VSD) is the most common congenital heart condition. It is essentially a "hole" in the septum between the two ventricles. VSDs are described by location, size, and significance of cardiac shunt. These characteristics are associated with varying degrees of symptoms, predict the likelihood of spontaneous VSD closure, and also suggest optimal management. Patients with a small VSD may only present with an isolated systolic murmur. Moderate to large VSDs may lead to signs of heart failure in the first month of life, including hepatomegaly, tachypnea, tachycardia, pallor, poor feeding, poor weight gain, and respiratory distress, including grunting, retractions, and rales. Cardiac exam for these patients includes precordial findings on palpation consistent with a left-to-right shunt. The classic systolic murmur of VSD is harsh, blowing holosystolic murmur with thrill best heard at the left sternal border, however, in infants with a large VSD, a murmur may not be appreciated due to increased right ventricular pressures. Infants with VSDs may also present with diastolic murmurs indicating left-to-right shunting. Presentation with symptoms of heart failure requires consideration of pulmonary causes of illness, but the presence of a systolic murmur can redirect the differential back to cardiac causes, providing the rest of the exam and workup support a non-respiratory illness. Other congenital cardiac defects that may present without cyanosis and should be considered include tricuspid regurgitation and mitral regurgitation. Definitive diagnosis should be obtained by echocardiography. Small, asymptomatic VSDs generally do not require special treatment. Larger VSDs may require medical, nutritional, or surgical intervention. Infants with bronchiolitis (A) are likely to present with tachypnea and tachycardia but usually give a history of upper respiratory congestion with or without fever. Systolic murmurs are not associated with this respiratory condition. Likewise, pneumonia (B) would not present with a systolic murmur and exam should additionally note diminished breath sounds and history of fever. Transposition of the great arteries (C) presents with tachypnea, but cyanosis and diminished pulses would be expected as well. Murmurs are not typically associated with this condition except in cases where a septal defect or left ventricular outflow tract obstruction is also present.

Which of the following diagnoses best corresponds with the rhythm strip shown above? A) Atrial fibrillation B) Multifocal atrial tachycardia C) Sinus arrhythmia D) Wandering atrial pacemaker

Explanation: Wandering atrial pacemaker, also known as multifocal atrial rhythm, is an atrial dysrhythmia caused by three or more ectopic atrial foci. The sinoatrial node is the primary pacemaker of the heart and initiates the cardiac cycle with atrial stimulation. Electrical impulse then travels through the atrioventricular node and the bundle of His. Impulses split between the right and left branches of the bundle of His and extend through myocardium of the ventricles via the Purkinje fiber system. The sinoatrial node has spontaneous automaticity, meaning it generates electrical impulses without stimulus. The intrinsic rate of the sinoatrial node varies from 60-100 beats per minute. Within the atria also exist ectopic foci, which are cells with pacemaker potential. In the wandering atrial pacemaker condition, three or more atrial foci discharge randomly (wandering), creating their own P wave tracings on ECG. Wandering atrial pacemaker can originate secondary to enhanced automaticity of these foci or can be triggered by a normal physiologic stimulus, causing excessive excitation if occurring after the refractory period. Patients with pulmonary disease, most commonly chronic obstructive pulmonary disease, are at an increased risk for developing wandering atrial pacemaker. Cardiac disease, including coronary, valvular, and hypertensive conditions, can also predispose this dysrhythmia. Other conditions with a known correlation to wandering atrial pacemaker include hypokalemia, hypomagnesemia, medications (isoproterenol, aminophylline, theophylline), chronic renal failure, and sepsis. Patients are typically asymptomatic from this dysrhythmia itself and present with symptoms pertaining to their underlying etiology, such as shortness of breath or wheezing in pulmonary disease patients. Wandering atrial pacemaker is diagnosed on ECG as variable P wave morphology and PR interval duration with an irregularly irregular rhythm. Often noted incidentally on ECG, the dysrhythmia does not require specific treatment and management should instead focus on the effective treatment of the underlying condition. Atrial fibrillation (A) is also characterized by an irregularly irregular rhythm, however, the distinguishing difference lies in the multiple distinctive P wave morphologies seen on wandering atrial pacemaker ECG tracings as opposed to indistinguishable P waves on atrial fibrillation ECG tracings. Multifocal atrial tachycardia (B) differs from wandering atrial pacemaker in that the heart rate is tachycardic (i.e., greater than 100 beats per minute). Sinus arrhythmia (C) may also be irregularly irregular but displays one P wave morphology. A 65-year-old man presents to your clinic after laboratory testing revealed elevated total cholesterol and low density lipoprotein-cholesterol. After discussion, statin therapy is prescribed.

While evaluating an ECG of a patient with palpitations, you note delta waves. What condition does this patient most likely have? A) Atrial flutter B) Hypothermia C) Right bundle branch block D) Wolff-Parkinson-White syndrome

Explanation: Wolff-Parkinson-White syndrome is a pre-excitation syndrome caused by an accessory pathway from atria to ventricles. It causes premature ventricular excitation because it lacks the delay seen in the AV node. Wolff-Parkinson-White syndrome is relatively rare, occurring in less than 1% of the general population. Of that small percentage, only about 10% will ever exhibit any symptoms. Signs and symptoms of Wolff-Parkinson-White syndrome include palpitations, lightheadedness, chest pain, and sudden cardiac death. These symptoms are most commonly a result of atrioventricular nodal reentrant tachycardia but can result from atrial fibrillation, atrial flutter, ventricular tachycardia, or ventricular fibrillation. Characteristic ECG findings in Wolff-Parkinson-White syndrome are a narrow complex tachycardia, short PR interval resulting from a rapid conduction down the accessory pathway and delta waves, which appear as a slurred QRS upstroke. This delta wave finding is a result of slowed ventricular activation. Treatment of Wolff-Parkinson-White syndrome consists of catheter ablation, surgical ablation, or medical therapy (procainamide or quinidine). Care must be taken to avoid AV nodal blocking agents such as adenosine, beta blockers, calcium channel blockers, and digoxin, as they may increase the risk of ventricular fibrillation. Atrial flutter (A) is characterized by a sawtooth pattern on an ECG. Although Wolff-Parkinson-White syndrome can sometimes result in atrial flutter, it does not exhibit a delta wave on the ECG. Hypothermia (B) if severe enough, can produce a characteristic J or Osborn wave. This is characterized by a J point elevation most prominent in leads V2 through V5. Right bundle branch block (C) is characterized by a QRS duration longer than 120 ms and Rsr', rsR', or rSR' in leads V1 or V2. Because there is no preexcitation pathway, a delta wave will not be observed.


Related study sets

Differentiation in the classroom

View Set

English as a Second Language Supplemental (154)

View Set

Nuclear Energy Assignment and quiz

View Set